LSAT PREP TEST QUESTIONS Flashcards

1
Q

LSAT Logical Reasoning Prep Test 52 Question #1 Certain companies require their managers to rank
workers in the groups they supervise from best to
worst, giving each worker a unique ranking based on
job performance. The top 10 percent of the workers
in each group are rewarded and the bottom 10
percent are penalized or fired. But this system is
unfair to workers. Good workers could receive low
rankings merely because they belong to groups of
exceptionally good workers. Furthermore, managers
often give the highest rankings to workers who share
the manager’s interests outside of work.
Which one of the following most accurately expresses
the conclusion drawn in the argument?
(A) Some companies require their managers to give
unique rankings to the workers they supervise.
(B) Under the ranking system, the top 10 percent of
the workers in each group are rewarded and the
bottom 10 percent are penalized or fired.
(C) The ranking system is not a fair way to determine
penalties or rewards for workers.
(D) Workers in exceptionally strong work groups are
unfairly penalized under the ranking system.
(E) Managers often give the highest rankings to
workers who share the manager’s outside
interests.

A

Correct Answer: C

C Main Point

This argument disagrees with the compensation system companies use requiring managers to first rank their workers from best to worst, and then reward the top 10 percent in each group and penalize or fire the workers in the bottom 10 percent. The argument concludes that this system is unfair to workers. The premises are that the rankings depend too much on the quality of the workers with whom each worker is grouped. Also, managers often rank workers for the wrong reasons, such as affinity.

A. No. This is a premise.

B. No. This is a premise.

C. Yes. This is a good restatement of the disagreement.

D. No. This is a premise.

E. No. This is a premise.

How well did you know this?
1
Not at all
2
3
4
5
Perfectly
2
Q

LSAT Logical Reasoning Prep Test 52 Question #2 Psychologist: A study of 436 university students
found that those who took short naps
throughout the day suffered from insomnia
more frequently than those who did not.
Moreover, people who work on commercial
fishing vessels often have irregular sleep
patterns that include frequent napping, and
they also suffer from insomnia. So it is very
likely that napping tends to cause insomnia.
The reasoning in the psychologist’s argument is most
vulnerable to criticism on the grounds that the
argument
(A) presumes, without providing justification, that
university students suffer from insomnia
more frequently than do members of the
general population
(B) presumes that all instances of insomnia have
the same cause
(C) fails to provide a scientifically respectable
definition for the term “napping”
(D) fails to consider the possibility that frequent
daytime napping is an effect rather than a
cause of insomnia
(E) presumes, without providing justification, that
there is such a thing as a regular sleep pattern
for someone working on a commercial fishing
vessel

A

Correct Answer: D

D Flaw

The argument presents a causal interpretation of evidence. The psychologist concludes that napping is likely to cause insomnia because two groups of people who nap more suffer more from insomnia.

A. No. The argument does not compare university students to the general population, but rather university students who nap to those who do not.

B. No. The argument does not assume that napping is the only cause of insomnia, just that it tends to cause insomnia.

C. No. The argument uses the term “napping” consistently and does not need to define it.

D. Yes. The argument fails to rule out the possibility that the causal direction might be reversed.

E. No. This is not essential to the argument.

How well did you know this?
1
Not at all
2
3
4
5
Perfectly
3
Q

LSAT Logical Reasoning Prep Test 52 Question #3 Whenever Joe’s car is vacuumed, the employees of
K & L Auto vacuum it; they are the only people who
ever vacuum Joe’s car. If the employees of K & L Auto
vacuumed Joe’s car, then Joe took his car to K & L
Auto to be fixed. Joe’s car was recently vacuumed.
Therefore, Joe took his car to K & L Auto to be fixed.
The pattern of reasoning exhibited by the argument
above is most similar to that exhibited by which one
of the following?
(A) Emily’s water glass is wet and it would be wet
only if she drank water from it this morning.
Since the only time she drinks water in the
morning is when she takes her medication,
Emily took her medication this morning.
(B) Lisa went to the hair salon today since either
she went to the hair salon today or she went
to the bank this morning, but Lisa did not go
to the bank this morning.
(C) There are no bills on John’s kitchen table. Since
John gets at least one bill per day and he
always puts his bills on his kitchen table,
someone else must have checked John’s mail
today.
(D) Linda is grumpy only if she does not have her
coffee in the morning, and Linda does not
have her coffee in the morning only if she
runs out of coffee. Therefore, Linda runs out
of coffee only on days that she is grumpy.
(E) Jeff had to choose either a grapefruit or cereal
for breakfast this morning. Given that Jeff is
allergic to grapefruit, Jeff must have had
cereal for breakfast this morning.

A

Correct Answer: A

A Parallel

Diagram the argument. Joe’s car is vacuumed → K & L vacuumed it. K & L vacuumed it → Joe took it to K & L to be fixed. The argument demonstrates that the sufficient factor (Joe’s car is vacuumed) is valid. Therefore, the result (Joe took his car to K & L to be fixed) must also be true.

A. Yes. Emily’s glass is wet → she drank water from it this morning → she took her medication. Emily’s glass is wet, so she must have taken her medication.

B. No. There is no either/or option in the original argument.

C. No. This answer choice does not match the structure of the original argument.

D. No. This answer choice demonstrates flawed reasoning and the original argument does not.

E. No. There is no either/or option in the original argument.

How well did you know this?
1
Not at all
2
3
4
5
Perfectly
4
Q

LSAT Logical Reasoning Prep Test 52 Question #4 Editorialist: In a large corporation, one of the
functions of the corporation’s president is to
promote the key interests of the shareholders.
Therefore, the president has a duty to keep the
corporation’s profits high.
Which one of the following, if true, would most
strengthen the editorialist’s argument?
(A) Shareholders sometimes will be satisfied even
if dividends paid to them from company
profits are not high.
(B) The president and the board of directors of a
corporation are jointly responsible for
advancing the key interests of the
shareholders.
(C) Keeping a corporation’s profits high is likely to
advance the important interests of the
corporation’s shareholders.
(D) In considering where to invest, most potential
shareholders are interested in more than just
the profitability of a corporation.
(E) The president of a corporation has many
functions besides advancing the important
interests of the corporation’s shareholders.

A

Correct Answer: C

C Strengthen

The editorialist concludes that the president has a duty to keep the corporation’s profits high. The premise is that one of the functions of the president is to promote the key interests of the stockholders. The argument would benefit from a connection between the key interests of the stockholders and high profits.

A. No. This answer choice would, if anything, weaken the argument.

B. No. The argument is concerned with the president and what he/she should do, not the board of directors.

C. Yes. This answer choice provides a strong connection between the key interests of the shareholders and keeping the profits high.

D. No. The argument does not imply that profitability is the only interest of the shareholders.

E. No. Like (D), the argument does not imply that advancing the important interests of shareholders is the president’s only responsibility.

How well did you know this?
1
Not at all
2
3
4
5
Perfectly
5
Q

LSAT Logical Reasoning Prep Test 52 Question #5 Everyone in Biba’s neighborhood is permitted to
swim at Barton Pool at some time during each day
that it is open. No children under the age of 6 are
permitted to swim at Barton Pool between noon and
5 P.M. From 5 P.M. until closing, Barton Pool is
reserved for adults only.
If all the sentences above are true, then which one of
the following must be true?
(A) Few children under the age of 6 live in Biba’s
neighborhood.
(B) If Biba’s next-door neighbor has a child under
the age of 6, then Barton Pool is open before
noon.
(C) If most children who swim in Barton Pool
swim in the afternoon, then the pool is
generally less crowded after 5 P.M.
(D) On days when Barton Pool is open, at least
some children swim there in the afternoon.
(E) Any child swimming in Barton Pool before
5 P.M. must be breaking Barton Pool rules.

A

Correct Answer: B

B Inference

This is a connect-the-facts inference with conditionals. Live in Biba’s neighborhood → permitted to swim at some time each day. Under 6 → can’t swim between noon and 5 P.M. Child → can’t swim from 5 P.M. until closing.

A. No. You can’t prove how many, if any, children live in Biba’s neighborhood.

B. Yes. Children under 6 can’t swim in the pool from noon until closing. Yet, as the passage says, everyone that lives in the neighborhood must be able to swim at some time during the day. This child would have to be able to swim before noon.

C. No. The argument does not provide any information to determine how crowded the pool will be.

D. No. The argument does not explain who lives in Biba’s neighborhood. It could be a neighborhood with no children.

E. No. The pool could be open before noon. Also, it’s only children under 6 that are not allowed to swim between noon and 5 P.M.

How well did you know this?
1
Not at all
2
3
4
5
Perfectly
6
Q

LSAT Logical Reasoning Prep Test 52 Question #6 Beck: Our computer program estimates municipal
automotive use based on weekly data. Some
staff question the accuracy of the program’s
estimates. But because the figures it provides
are remarkably consistent from week to week,
we can be confident of its accuracy.
The reasoning in Beck’s argument is flawed in that it
(A) fails to establish that consistency is a more
important consideration than accuracy
(B) fails to consider the program’s accuracy in
other tasks that it may perform
(C) takes for granted that the program’s output
would be consistent even if its estimates were
inaccurate
(D) regards accuracy as the sole criterion for
judging the program’s value
(E) fails to consider that the program could
produce consistent but inaccurate output

A

Correct Answer: E

E Flaw

The purpose of this argument is to disagree with a claim. Beck concludes that, despite what some of the staff says, they can be confident of their computer program’s accuracy because the figures that the program provides are consistent from week to week. Beck equates consistency with accuracy.

A. No. The argument doesn’t claim that consistency is more important than accuracy. It equates the two.

B. No. The argument is concerned with only one task: estimating the municipal automotive use.

C. No. The argument takes for granted that, because the output is consistent, the program must be accurate.

D. No. The argument is concerned with only the accuracy of the program. It doesn’t make any claims about the program’s value in general.

E. Yes. The argument assumes that, because the output is consistent, the program must be accurate.

How well did you know this?
1
Not at all
2
3
4
5
Perfectly
7
Q

LSAT Logical Reasoning Prep Test 52 Question #7 Inertia affects the flow of water pumped through a
closed system of pipes. When the pump is first
switched on, the water, which has mass, takes time to
reach full speed. When the pump is switched off,
inertia causes the decrease in the water flow to be
gradual. The effects of inductance in electrical
circuits are similar to the effects of inertia in water
pipes.
The information above provides the most support
for which one of the following?
(A) The rate at which electrical current flows is
affected by inductance.
(B) The flow of electrical current in a circuit
requires inertia.
(C) Inertia in the flow of water pumped by an
electrically powered pump is caused by
inductance in the pump’s circuits.
(D) Electrical engineers try to minimize the effects
of inductance in electrical circuits.
(E) When a water pump is switched off it
continues to pump water for a second or two

A

Correct Answer: A

A Inference

The argument describes how inertia affects the flow of water pumped through a closed system of pipes. It then equates the effects of inductance in electrical circuits with the effects of inertia in water pipes.

A. Yes. If the effects of inductance are similar to the effects of inertia, and inertia affects the flow of water, then inductance will affect the flow of electrical current.

B. No. Inertia refers to water and pipes, not the flow of electrical current.

C. No. You cannot prove how inductance affects inertia from the information provided.

D. No. You cannot prove anything about electrical engineers from the information provided.

E. No. This choice is too strong. All you can prove is that the inertia causes the decrease in the water flow to be gradual. You cannot prove how long this takes.

How well did you know this?
1
Not at all
2
3
4
5
Perfectly
8
Q

LSAT Logical Reasoning Prep Test 52 Question #8 Journalist: To reconcile the need for profits sufficient
to support new drug research with the moral
imperative to provide medicines to those who
most need them but cannot afford them, some
pharmaceutical companies feel justified in
selling a drug in rich nations at one price and
in poor nations at a much lower price. But this
practice is unjustified. A nation with a low
average income may still have a substantial
middle class better able to pay for new drugs
than are many of the poorer citizens of an
overall wealthier nation.
Which one of the following principles, if valid, most
helps to justify the journalist’s reasoning?
(A) People who are ill deserve more consideration
than do healthy people, regardless of their
relative socioeconomic positions.
(B) Wealthy institutions have an obligation to
expend at least some of their resources to
assist those incapable of assisting themselves.
(C) Whether one deserves special consideration
depends on one’s needs rather than on
characteristics of the society to which one
belongs.
(D) The people in wealthy nations should not have
better access to health care than do the people
in poorer nations.
(E) Unequal access to health care is more unfair
than an unequal distribution of wealth.

A

Correct Answer: C

C Principle Strengthen

The purpose of this argument is to disagree with a claim. The journalist thinks that the pharmaceutical companies are not justified in selling a drug in rich nations at one price and in poor nations at another price. The journalist’s premise is that many individuals in poorer nations might be better able to pay for new drugs than poorer individuals in nations with higher overall wealth.

A. No. The argument doesn’t compare ill people to healthy people.

B. No. This answer choice supports the position that the argument is designed to disagree with.

C. Yes. This answer choice points out that special consideration should be provided to individuals when individuals within a society might not have the same amount of resources as the society does, on average.

D. No. This answer choice is not relevant to the argument.

E. No. The argument does not discuss the fairness of unequal distribution of wealth.

How well did you know this?
1
Not at all
2
3
4
5
Perfectly
9
Q

LSAT Logical Reasoning Prep Test 52 Question #9 9. Robert: The school board is considering adopting a
year-round academic schedule that eliminates
the traditional three-month summer vacation.
This schedule should be adopted, since
teachers need to cover more new material
during the school year than they do now.
Samantha: The proposed schedule will not permit
teachers to cover more new material. Even
though the schedule eliminates summer
vacation, it adds six new two-week breaks, so
the total number of school days will be about
the same as before.
Which one of the following, if true, is a response
Robert could make that would counter Samantha’s
argument?
(A) Teachers would be willing to accept
elimination of the traditional three-month
summer vacation as long as the total vacation
time they are entitled to each year is not
reduced.
(B) Most parents who work outside the home find
it difficult to arrange adequate supervision
for their school-age children over the
traditional three-month summer vacation.
(C) In school districts that have adopted a yearround schedule that increases the number of
school days per year, students show a deeper
understanding and better retention of new
material.
(D) Teachers spend no more than a day of class
time reviewing old material when students
have been away from school for only a few
weeks, but have to spend up to a month of
class time reviewing after a three-month
summer vacation.
(E) Students prefer taking a long vacation from
school during the summer to taking more
frequent but shorter vacations spread
throughout the year.

A

Correct Answer: D

D Weaken

Robert is trying to solve a problem and Samantha claims that he hasn’t solved it. Robert concludes that the school board should adopt a year-round academic schedule because teachers need to cover more new material during the school year than they do now. Samantha claims that the new school schedule won’t permit the teachers to cover any more new material because the amount of vacation will be the same as before, just in a different configuration. The students will have six two-week breaks, instead of one three-month break.

A. No. This doesn’t address the problem of needing to cover more new material.

B. No. This answer choice does not address the problem of needing to cover more new material.

C. No. It’s nice that students show a deeper understanding of the material in year-round schools, but the answer choice does not address the issue at hand: needing to cover more new material.

D. Yes. If the teachers have to spend significantly less class time reviewing material after the short breaks in the year-round schedule than after the long break in the traditional schedule, then teachers will have more class days to cover new material.

E. No. Student preference is not relevant. The disagreement is about the amount of new material that could be covered.

How well did you know this?
1
Not at all
2
3
4
5
Perfectly
10
Q

LSAT Logical Reasoning Prep Test 52 Question #10 In order to reduce traffic congestion and raise
revenue for the city, the mayor plans to implement a
charge of $10 per day for driving in the downtown
area. Payment of this charge will be enforced using a
highly sophisticated system that employs digital
cameras and computerized automobile registration.
This system will not be ready until the end of next
year. Without this system, however, mass evasion of
the charge will result. Therefore, when the mayor’s
plan is first implemented, payment of the charge will
not be effectively enforced.
Which one of the following is an assumption on
which the argument depends for its conclusion to be
properly drawn?
(A) The mayor’s plan to charge for driving
downtown will be implemented before the
end of next year.
(B) The city will incur a budget deficit if it does
not receive the revenue it expects to raise
from the charge for driving downtown.
(C) The plan to charge for driving downtown
should be implemented as soon as payment of
the charge can be effectively enforced.
(D) Raising revenue is a more important
consideration for the city than is reducing
traffic congestion.
(E) A daily charge for driving downtown is the
most effective way to reduce traffic
congestion.

A

Correct Answer: A

A Necessary Assumption

The argument concludes that the mayor’s plan to reduce congestion and raise revenue by charging $10 per day for driving in the downtown area will not be effectively enforced when it is first implemented. The premises are that payment will be enforced by a highly sophisticated system that will not be ready until the end of next year, and that many people will avoid paying the charge without this system in place.

A. Yes. It is essential to the argument that the plan be in place before the system is ready at the end of next year.

B. No. The argument does not discuss the possibility of a budget deficit.

C. No. The argument is concerned with whether the plan will be effectively enforced when it is first implemented, not when it should be implemented.

D. No. The argument does not compare the importance of raising revenue versus reducing traffic congestion.

E. No. Too strong. The argument doesn’t need to say that a daily charge is the most effective way to reduce traffic congestion.

How well did you know this?
1
Not at all
2
3
4
5
Perfectly
11
Q

LSAT Logical Reasoning Prep Test 52 Question #11 A recent study revealed that the percentage of people
treated at large, urban hospitals who recover from
their illnesses is lower than the percentage for people
treated at smaller, rural hospitals.
Each of the following, if true, contributes to an
explanation of the difference in recovery rates
EXCEPT:
(A) Because there are fewer patients to feed,
nutritionists at small hospitals are better able
to tailor meals to the dietary needs of each
patient.
(B) The less friendly, more impersonal atmosphere
of large hospitals can be a source of stress for
patients at those hospitals.
(C) Although large hospitals tend to draw doctors
trained at the more prestigious schools, no
correlation has been found between the
prestige of a doctor’s school and patients’
recovery rate.
(D) Because space is relatively scarce in large
hospitals, doctors are encouraged to minimize
the length of time that patients are held for
observation following a medical procedure.
(E) Doctors at large hospitals tend to have a
greater number of patients and consequently
less time to explain to staff and to patients
how medications are to be administered.

A

Correct Answer: C

C Resolve/Explain

There is a discrepancy in recovery rates between people treated at large, urban hospitals and people treated at smaller, rural hospitals. People treated at smaller, rural hospitals have a higher rate of recovery. Eliminate any answer choices that help resolve the discrepancy.

A. No. If the patients at smaller hospitals are more likely to get fed according to their dietary needs, they might be more likely to recover from their illnesses.

B. No. If patients at larger hospitals are more stressed than patients at smaller hospitals, that might adversely affect their recovery rate.

C. Yes. This answer choice states that there has been no correlation found between the prestige of a doctor’s school and patients’ recovery rates. In no way is this relevant to explaining the differing recovery rates of patients treated at larger or smaller hospitals.

D. No. If patients are not observed for as long, there is a greater chance that an unobserved complication might occur, thus adversely affecting the recovery rate for patients treated at larger hospitals.

E. No. If the staff and patients do not get explanations about the administration of their medications, improper administration of these medications is more likely to occur. This would adversely affect the recovery rates of patients treated at larger hospitals.

How well did you know this?
1
Not at all
2
3
4
5
Perfectly
12
Q

LSAT Logical Reasoning Prep Test 52 Question #12 12. Perry: Worker-owned businesses require workers to
spend time on management decision-making
and investment strategy, tasks that are not
directly productive. Also, such businesses have
less extensive divisions of labor than do
investor-owned businesses. Such inefficiencies
can lead to low profitability, and thus increase
the risk for lenders. Therefore, lenders seeking
to reduce their risk should not make loans to
worker-owned businesses.
Which one of the following, if true, most seriously
weakens Perry’s argument?
(A) Businesses with the most extensive divisions of
labor sometimes fail to make the fullest use of
their most versatile employees’ potential.
(B) Lenders who specialize in high-risk loans are
the largest source of loans for worker-owned
businesses.
(C) Investor-owned businesses are more likely
than worker-owned businesses are to receive
start-up loans.
(D) Worker-owned businesses have traditionally
obtained loans from cooperative lending
institutions established by coalitions of
worker-owned businesses.
(E) In most worker-owned businesses, workers
compensate for inefficiencies by working
longer hours than do workers in investorowned businesses.

A

Correct Answer: E

E Weaken

Perry concludes that lenders who are seeking to reduce their risk should not make loans to worker-owned businesses. This is because worker-owned businesses require workers to spend time on management and investment, which are not directly productive. Worker-owned businesses also have less extensive divisions of labor than do investor-owned businesses. These inefficiencies can lead to low profitability, which would increase risk for lenders. An issue with Perry’s argument is that, just because inefficiencies can lead to low profitability, it doesn’t mean that they will. Maybe there is something unique about worker-owned businesses that will overcome this problem.

A. No. Businesses with the most extensive divisions of labor can sometimes fail to make the fullest use of their most versatile employees’ potential and still be more efficient, on average, than the worker-owned businesses and still be, on average, the safer investment.

B. No. This answer choice is a description of the lenders who do make loans to worker-owned businesses. It does not address the warning against lending to worker-owned businesses.

C. No. The argument is not concerned with start-up loans, nor is it concerned with who actually gets loans.

D. No. This answer choice does not give a reason as to why worker-owned businesses might be a less risky investment than the argument claims.

E. Yes. If the workers work longer hours, the inefficiencies might be compensated for, which means that the inefficiencies won’t necessarily lead to low profitability. If they don’t lead to low profitability, then the risk will not necessarily increase for lenders.

How well did you know this?
1
Not at all
2
3
4
5
Perfectly
13
Q

LSAT Logical Reasoning Prep Test 52 Question #13 13. Some paleontologists believe that certain species of
dinosaurs guarded their young in protective nests
long after the young hatched. As evidence, they cite
the discovery of fossilized hadrosaur babies and
adolescents in carefully designed nests. But similar
nests for hatchlings and adolescents are constructed
by modern crocodiles, even though crocodiles guard
their young only for a very brief time after they
hatch. Hence, .
Which one of the following most logically completes
the argument?
(A) paleontologists who believe that hadrosaurs
guarded their young long after the young
hatched have no evidence to support this
belief
(B) we will never be able to know the extent to
which hadrosaurs guarded their young
(C) hadrosaurs guarded their young for at most
very brief periods after hatching
(D) it is unclear whether what we learn about
hadrosaurs from their fossilized remains tells
us anything about other dinosaurs
(E) the construction of nests for hatchlings and
adolescents is not strong evidence for the
paleontologists’ belief

A

Correct Answer: E

E Main Point

This argument is designed to disagree with a claim. The argument disagrees with some paleontologists, who believe that certain dinosaurs guarded their young in protective nests long after the young hatched. The evidence cited for the paleontologists’ claim is the discovery of fossilized hadrosaur babies and adolescents in carefully designed nests. In disagreeing with these paleontologists, the argument notes that modern crocodiles construct similar nests, even though these crocodiles don’t guard their young for long. The argument seems to be leading to the conclusion that the evidence of fossilized nests is not enough to claim that the dinosaurs guarded their young long after the young hatched.

A. No. This answer choice is too strong. The paleontologists do have some evidence, just not enough to fully support their conclusion.

B. No. This answer choice is too strong. The evidence cited is not strong enough for the paleontologists’ conclusion. That’s not to say that we will never know the extent to which hadrosaurs guarded their young.

C. No. This answer choice is too strong. There is not enough evidence to know that they guarded their young for large periods of time. That’s not to say that hadrosaurs couldn’t have actually guarded their young for large periods of time.

D. No. This is not the disagreement. The paleontologists are making a claim about a certain species of dinosaurs, not all dinosaurs.

E. Yes. The argument disagrees with the paleontologists as to the strength of the evidence that supports their belief.

How well did you know this?
1
Not at all
2
3
4
5
Perfectly
14
Q

LSAT Logical Reasoning Prep Test 52 Question #14 14. For one academic year all the students at a high
school were observed. The aim was to test the
hypothesis that studying more increased a student’s
chances of earning a higher grade. It turned out that
the students who spent the most time studying did
not earn grades as high as did many students who
studied less. Nonetheless, the researchers concluded
that the results of the observation supported the
initial hypothesis.
Which one of the following, if true, most helps to
explain why the researchers drew the conclusion
described above?
(A) The students who spent the most time
studying earned higher grades than did some
students who studied for less time than the
average.
(B) The students tended to get slightly lower
grades as the academic year progressed.
(C) In each course, the more a student studied, the
better his or her grade was in that course.
(D) The students who spent the least time studying
tended to be students with no more than
average involvement in extracurricular
activities.
(E) Students who spent more time studying
understood the course material better than
other students did.

A

Correct Answer: C

C Resolve/Explain

The apparent paradox is that when researchers tested the hypothesis that studying more increased a student’s chances of earning a higher grade, the students who spent the most time studying did not earn grades as high as did many students who studied less. Yet, the researchers concluded that the results supported their hypothesis.

A. No. This doesn’t resolve the apparent paradox. The passage says that many students who studied less earned higher grades. This still leaves room for some students who studied less to get lower grades, while still maintaining the seeming paradox.

B. No. If all the students tended to get slightly lower grades as the year progressed, the problem still remains that many students who studied less will have higher grades than the students who spent the most time studying.

C. Yes. This resolves the apparent paradox. If each individual student does better in a given class if he or she studies, then it looks like studying more will increase a student’s change of getting a higher grade, even if the students who study the most get lower grades than many who don’t study as much. The hypothesis was about an individual’s grades, while the statistics were about the students considered in groups.

D. No. This does not resolve the apparent paradox.

E. No. This does not resolve the apparent paradox as well as (C) because it doesn’t directly connect studying with an individual’s chances of getting a better grade in a given class.

How well did you know this?
1
Not at all
2
3
4
5
Perfectly
15
Q

LSAT Logical Reasoning Prep Test 52 Question #15 15. Researchers had three groups of professional cyclists
cycle for one hour at different levels of intensity.
Members of groups A, B, and C cycled at rates that
sustained, for an hour, pulses of about 60 percent, 70
percent, and 85 percent, respectively, of the
recommended maximum pulse rate for recreational
cyclists. Most members of Group A reported being
less depressed and angry afterward. Most members of
Group B did not report these benefits. Most
members of Group C reported feeling worse in these
respects than before the exercise.
Which one of the following is most strongly
supported by the information above?
(A) The higher the pulse rate attained in sustained
exercise, the less psychological benefit the
exercise tends to produce.
(B) The effect that a period of cycling has on the
mood of professional cyclists tends to depend
at least in part on how intense the cycling is.
(C) For professional cyclists, the best exercise from
the point of view of improving mood is
cycling that pushes the pulse no higher than
60 percent of the maximum pulse rate.
(D) Physical factors, including pulse rate,
contribute as much to depression as do
psychological factors.
(E) Moderate cycling tends to benefit professional
cyclists physically as much or more than
intense cycling

A

Correct Answer: B

B Inference

Find the answer choice supported by the passage.

A. No. This answer choice is too strong. The argument talks only about cycling, not about exercise in general. In addition, you can’t prove that in every instance the higher the pulse rate, the less psychological benefit produced. What if having a pulse rate of 60 percent of the recommended maximum pulse rate leads to greater benefits than having a pulse rate of 40 percent of the maximum?

B. Yes. The argument demonstrates a correlation between the mood of professional cyclists and how intense the cycling is. When the cycling is at 60 percent of the recommended maximum pulse rate for recreational bikers, the professional cyclists reported being less depressed and angry. Those cycling at 85 percent, on the other hand, reported feeling more depressed and angry.

C. No. This answer choice is too strong. You can’t prove anything about pulse rates higher than 85 percent so it is possible that something higher could also improve mood.

D. No. This answer choice is too strong. You can’t prove that physical factors contribute as much as psychological factors contribute. The passage does not include information about any psychological factors that may have been at play so you can’t make that comparison.

E. No. You can’t prove whether moderate cycling benefits professional cyclists physically as much or more than intense cycling.

How well did you know this?
1
Not at all
2
3
4
5
Perfectly
16
Q

LSAT Logical Reasoning Prep Test 52 Question #16 16. Anyone who believes in extraterrestrials believes in
UFOs. But the existence of UFOs has been
conclusively refuted. Therefore a belief in
extraterrestrials is false as well.
Which one of the following arguments contains
flawed reasoning most similar to that in the
argument above?
(A) Anyone who believes in unicorns believes in
centaurs. But it has been demonstrated that
there are no centaurs, so there are no
unicorns either.
(B) Anyone who believes in unicorns believes in
centaurs. But you do not believe in centaurs,
so you do not believe in unicorns either.
(C) Anyone who believes in unicorns believes in
centaurs. But you do not believe in unicorns,
so you do not believe in centaurs either.
(D) Anyone who believes in unicorns believes in
centaurs. But there is no good reason to
believe in centaurs, so a belief in unicorns is
unjustified as well.
(E) Anyone who believes in unicorns believes in
centaurs. But it has been conclusively proven
that there is no such thing as a unicorn, so a
belief in centaurs is mistaken as well.

A

Correct Answer: A

A Parallel Flaw

This argument is diagrammable: Believe in existence of ETs → believe in existence of UFOs. UFOs don’t exist → ETs don’t exist. The argument does not support the assumption that the existence of ETs depends on the existence of UFOs.

A. Yes. Believe in existence of unicorns → believe in existence of centaurs. Centaurs don’t exist → unicorns can’t exist. This is the same argument structure and the same flaw.

B. No. This argument is not flawed. Believe in unicorns → believe in centaurs. Don’t believe in centaurs → don’t believe in unicorns. The second conditional is the contrapositive of the first.

C. No. This answer choice does not make the switch from belief to actual existence.

D. No. This answer choice does not make the switch from belief to actual existence.

E. No. This answer choice does make the switch from belief to actual existence but its second premise starts with the non-existence of unicorns, which were, in fact, the sufficient condition in the first premise. This does not match the structure of the original argument.

How well did you know this?
1
Not at all
2
3
4
5
Perfectly
17
Q

LSAT Logical Reasoning Prep Test 52 Question #17 17. People want to be instantly and intuitively liked.
Those persons who are perceived as forming
opinions of others only after cautiously gathering
and weighing the evidence are generally resented.
Thus, it is imprudent to appear prudent.
Which one of the following, if assumed, enables the
argument’s conclusion to be properly drawn?
(A) People who act spontaneously are well liked.
(B) Imprudent people act instantly and intuitively.
(C) People resent those less prudent than
themselves.
(D) People who are intuitive know instantly when
they like someone.
(E) It is imprudent to cause people to resent you.

A

Correct Answer: E

E Suffiicient Assumption

The conclusion of the argument is that it is imprudent to appear prudent. The first premise is that people want to be instantly and intuitively liked. The second premise is that people who are perceived to form opinions about others non-spontaneously are generally resented. The argument shifts language from the premise, which discusses behavior that causes resentment, to the conclusion, which discusses prudence. You need an answer that proves the conclusion by tying it to resentment.

A. No. The conclusion isn’t about how people need to act in order to be well liked. It is a judgment about appearing prudent, not about spontaneity.

B. No. You need to know why it is imprudent to appear prudent, not how imprudent people generally act.

C. No. If anything, this argument seems to claim that people resent those more prudent than themselves.

D. No. The conclusion is concerned with prudence and imprudence, not about intuitive people.

E. Yes. This gives the argument the connection it needs between imprudence and resentment.

How well did you know this?
1
Not at all
2
3
4
5
Perfectly
18
Q

LSAT Logical Reasoning Prep Test 52 Question #18 18. Journalist: Recent studies have demonstrated that a
regular smoker who has just smoked a
cigarette will typically display significantly
better short-term memory skills than a
nonsmoker, whether or not the nonsmoker has
also just smoked a cigarette for the purposes of
the study. Moreover, the majority of those
smokers who exhibit this superiority in shortterm memory skills will do so for at least eight
hours after having last smoked.
If the journalist’s statements are true, then each of
the following could be true EXCEPT:
(A) The short-term memory skills exhibited by a
nonsmoker who has just smoked a cigarette
are usually substantially worse than the shortterm memory skills exhibited by a nonsmoker
who has not recently smoked a cigarette.
(B) The short-term memory skills exhibited by a
nonsmoker who has just smoked a cigarette
are typically superior to those exhibited by a
regular smoker who has just smoked a
cigarette.
(C) The short-term memory skills exhibited by a
nonsmoker who has just smoked a cigarette
are typically superior to those exhibited by a
regular smoker who has not smoked for more
than eight hours.
(D) A regular smoker who, immediately after
smoking a cigarette, exhibits short-term
memory skills no better than those typically
exhibited by a nonsmoker is nevertheless
likely to exhibit superior short-term memory
skills in the hours following a period of heavy
smoking.
(E) The short-term memory skills exhibited by a
regular smoker who last smoked a cigarette
five hours ago are typically superior to those
exhibited by a regular smoker who has just
smoked a cigarette.

A

Correct Answer: B

B Inference

Find the answer choice that contradicts evidence in the argument.

A. No. This answer choice could be true. The argument doesn’t compare the difference in memory between nonsmokers who have just smoked a cigarette and nonsmokers who haven’t recently smoked a cigarette.

B. Yes. The argument states that the short-term memory skills of a nonsmoker who has just smoked are typically significantly worse than those of a smoker who has just smoked. So, this answer choice directly contradicts the argument.

C. No. The argument doesn’t compare the memory skills of nonsmokers who have just smoked and smokers who haven’t smoked in over eight hours. This answer choice could be true.

D. No. This answer choice could be true. The argument doesn’t say anything about periods of heavy smoking. The answer choice is also comparing two individuals, not the typical results.

E. No. The argument doesn’t compare the memory skills of smokers who last smoked five hours ago and smokers who have just smoked. This answer choice could be true.

How well did you know this?
1
Not at all
2
3
4
5
Perfectly
19
Q

LSAT Logical Reasoning Prep Test 52 Question #19 19. Educator: It has been argued that our professional
organization should make decisions about
important issues—such as raising dues and
taking political stands—by a direct vote of all
members rather than by having members vote
for officers who in turn make the decisions.
This would not, however, be the right way to
decide these matters, for the vote of any given
individual is much more likely to determine
organizational policy by influencing the
election of an officer than by influencing the
result of a direct vote on a single issue.
Which one of the following principles would, if valid,
most help to justify the educator’s reasoning?
(A) No procedure for making organizational
decisions should allow one individual’s vote
to weigh more than that of another.
(B) Outcomes of organizational elections should
be evaluated according to their benefit to the
organization as a whole, not according to the
fairness of the methods by which they are
produced.
(C) Important issues facing organizations should
be decided by people who can devote their
full time to mastering the information
relevant to the issues.
(D) An officer of an organization should not make
a particular decision on an issue unless a
majority of the organization’s members
would approve of that decision.
(E) An organization’s procedures for making
organizational decisions should maximize the
power of each member of the org

A

Correct Answer: E

E Principle Strengthen

This argument is designed to disagree with a claim. The educator is against deciding matters in his professional organization by a direct vote instead of having matter decided by officers who are elected by direct vote. The premise is that organizational policy will be more influenced by individuals voting for officers rather than individuals directly voting on issues. A principle that would strengthen it would connect procedures for making organizational decisions with the amount of influence each member has on these decisions.

A. No. This principle would weaken the argument, if anything.

B. No. This principle does not connect the procedures for making decisions with the amount of influence of each member on these decisions.

C. No. This answer choice does not tell us that it would be the officers that would have this time, so it doesn’t help the argument.

D. No. This might be true but it does not strengthen the educator’s claim that voting to elect officers will give each individual more influence in organizational policy.

E. Yes. This principle strengthens the argument by relating procedures for making decisions and the maximization of the power of each individual to influence the decisions.

How well did you know this?
1
Not at all
2
3
4
5
Perfectly
20
Q

LSAT Logical Reasoning Prep Test 52 Question #20 20. Neural connections carrying signals from the cortex
(the brain region responsible for thought) down to
the amygdala (a brain region crucial for emotions)
are less well developed than connections carrying
signals from the amygdala up to the cortex. Thus, the
amygdala exerts a greater influence on the cortex
than vice versa.
The argument’s conclusion follows logically if which
one of the following is assumed?
(A) The influence that the amygdala exerts on the
rest of the brain is dependent on the influence
that the cortex exerts on the rest of the brain.
(B) No other brain region exerts more influence
on the cortex than does the amygdala.
(C) The region of the brain that has the most
influence on the cortex is the one that has the
most highly developed neural connections to
the cortex.
(D) The amygdala is not itself controlled by one or
more other regions of the brain.
(E) The degree of development of a set of neural
connections is directly proportional to the
influence transmitted across those
connections.

A

Correct Answer: E

E Sufficient Assumption

The conclusion of the argument is that the amygdala exerts a greater influence on the cortex than vice versa. The premise for this conclusion is that the neural connections that carry signals from the cortex to the amygdala are less well developed than the connections carrying signals the other way around. The argument equates how developed the connections carrying signals from one part of the brain to the other are to the amount of influence one part of the brain has on the other.

A. No. The argument is concerned only with the influence the amygdala exerts on the cortex, not the rest of the brain.

B. No. Other brain regions are not relevant and the assumption needs to equate how developed the connections are to the amount of influence.

C. No. This answer choice is too general. The region of the brain that has the most highly developed neural connections to the cortex might be something other than the amygdala.

D. No. The argument is concerned with the influence that the amygdala has on the cortex. It doesn’t matter whether some other region controls it.

E. Yes. This connects the degree of development of neural connections with the degree of influence one part of the brain has on another.

How well did you know this?
1
Not at all
2
3
4
5
Perfectly
21
Q

LSAT Logical Reasoning Prep Test 52 Question #21 21. The Iliad and the Odyssey were both attributed to
Homer in ancient times. But these two poems differ
greatly in tone and vocabulary and in certain details
of the fictional world they depict. So they are almost
certainly not the work of the same poet.
Which one of the following statements, if true, most
weakens the reasoning above?
(A) Several hymns that were also attributed to
Homer in ancient times differ more from the
Iliad in the respects mentioned than does the
Odyssey.
(B) Both the Iliad and the Odyssey have come
down to us in manuscripts that have suffered
from minor copying errors and other textual
corruptions.
(C) Works known to have been written by the
same modern writer are as different from
each other in the respects mentioned as are
the Iliad and the Odyssey.
(D) Neither the Iliad nor the Odyssey taken by
itself is completely consistent in all of the
respects mentioned.
(E) Both the Iliad and the Odyssey were the result
of an extended process of oral composition in
which many poets were involved.

A

Correct Answer: C

C Weaken

This argument disagrees with the claim that the difference in vocabulary, tone, and details of the fictional world depicted in the Iliad and the Odyssey imply that they could almost certainly not be the work of the same poet. A good answer choice will demonstrate how even if two works are different in many ways, the same person may have written both of them.

A. No. Homer might not have actually written the hymns either.

B. No. This doesn’t go far enough. If the manuscripts have suffered only minor copying errors and other textual corruptions, the corruptions won’t explain away all of the stylistic differences between the two.

C. Yes. You know that the modern writer actually wrote the works described. This counterexample lends plausibility to the possibility that Homer wrote both the Iliad and the Odyssey despite their many differences.

D. No. The argument rests on comparing the Iliad with the Odyssey, not looking at each by itself.

E. No. This would support the claim that Homer didn’t write both.

How well did you know this?
1
Not at all
2
3
4
5
Perfectly
22
Q

LSAT Logical Reasoning Prep Test 52 Question #22 22. Moralist: A statement is wholly truthful only if it is
true and made without intended deception. A
statement is a lie if it is intended to deceive or
if its speaker, upon learning that the statement
was misinterpreted, refrains from clarifying it.
Which one of the following judgments most closely
conforms to the principles stated by the moralist?
(A) Ted’s statement to the investigator that he had
been abducted by extraterrestrial beings was
wholly truthful even though no one has ever
been abducted by extraterrestrial beings.
After all, Ted was not trying to deceive the
investigator.
(B) Tony was not lying when he told his
granddaughter that he did not wear dentures,
for even though Tony meant to deceive his
granddaughter, she made it clear to Tony that
she did not believe him.
(C) Siobhan did not tell a lie when she told her
supervisor that she was ill and hence would
not be able to come to work for an important
presentation. However, even though her
statement was true, it was not wholly truthful.
(D) Walter’s claim to a potential employer that he
had done volunteer work was a lie. Even
though Walter had worked without pay in his
father’s factory, he used the phrase “volunteer
work” in an attempt to deceive the interviewer
into thinking he had worked for a socially
beneficial cause.
(E) The tour guide intended to deceive the tourists
when he told them that the cabin they were
looking at was centuries old. Still, his
statement about the cabin’s age was not a lie,
for if he thought that this statement had been
misinterpreted, he would have tried to
clarify it.

A

Correct Answer: D

D Principle Match

Diagram the moralist’s two principles. 1. Statement wholly truthful → it’s true and made without intended deception. Contrapositive: ~true or made with intended deception → ~not wholly truthful. 2. Intended to deceive or doesn’t clarify misinterpretation → lie. Contrapositive: ~lie → ~intended to deceive and clarifies misinterpretation. The best answer choice will provide an example in which at least one of the two principles is fulfilled.

A. No. Neither principle gives criteria to determine that a statement is wholly truthful.

B. No. Neither principle gives criteria to determine that a statement is not a lie, only when it is a lie.

C. No. You don’t know whether Siobhan intended to deceive, nor do you know whether she is actually sick. This doesn’t fit either principle.

D. Yes. If a statement is intended to deceive, it is a lie. Walter intended to deceive, so he lied.

E. No. If the statement is intended to deceive OR the person doesn’t clarify a misinterpretation, the statement is a lie. So the tour guide DID lie, according to the second principle.

How well did you know this?
1
Not at all
2
3
4
5
Perfectly
23
Q

LSAT Logical Reasoning Prep Test 52 Question #23 23. Principle: It is healthy for children to engage in an
activity that promotes their intellectual
development only if engaging in that activity
does not detract from their social
development.
Application: Although Megan’s frequent reading
stimulates her intellectually, it reduces the
amount of time she spends interacting with
other people. Therefore, it is not healthy for
her to read as much as she does.
The application of the principle is most vulnerable to
criticism on which one of the following grounds?
(A) It misinterprets the principle as a universal
claim intended to hold in all cases without
exception, rather than as a mere
generalization.
(B) It overlooks the possibility that the benefits of
a given activity may sometimes be important
enough to outweigh the adverse health effects.
(C) It misinterprets the principle to be, at least in
part, a claim about what is unhealthy, rather
than solely a claim about what is healthy.
(D) It takes for granted that any decrease in the
amount of time a child spends interacting
with others detracts from that child’s social
development.
(E) It takes a necessary condition for an activity’s
being healthy as a sufficient condition for its
being so.

A

Correct Answer: D

D Principle Match

This argument contains a principle that states the following: healthy to engage in intellectual development → engaging in that activity does not detract from social development. The argument then presents an application in the evidence about Megan. It draws the conclusion that Megan’s amount of reading is not healthy because it reduces the amount of time she spends interacting with other people. The flaw is that the argument equates interacting with others with social development.

A. No. The principle is a universal claim.

B. No. The argument does not discuss health effects.

C. No. As the principle is a conditional statement, it has a contrapositive. The contrapositive does make a claim about what is unhealthy.

D. Yes. The argument equates interacting with other people and social development. It might well be that the plot lines in the books that she reads help with her social development.

E. No. This argument does not contain a necessary/sufficient flaw.

How well did you know this?
1
Not at all
2
3
4
5
Perfectly
24
Q

LSAT Logical Reasoning Prep Test 52 Question #24 In response to several bacterial infections traced to
its apple juice, McElligott now flash pasteurizes its
apple juice by quickly heating and immediately
rechilling it. Intensive pasteurization, in which juice
is heated for an hour, eliminates bacteria more
effectively than does any other method, but is likely
to destroy the original flavor. However, because
McElligott’s citrus juices have not been linked to any
bacterial infections, they remain unpasteurized.
The statements above, if true, provide the most
support for which one of the following claims?
(A) McElligott’s citrus juices contain fewer
infectious bacteria than do citrus juices
produced by other companies.
(B) McElligott’s apple juice is less likely to contain
infectious bacteria than are McElligott’s citrus
juices.
(C) McElligott’s citrus juices retain more of the
juices’ original flavor than do any pasteurized
citrus juices.
(D) The most effective method for eliminating
bacteria from juice is also the method most
likely to destroy flavor.
(E) Apple juice that undergoes intensive
pasteurization is less likely than McElligott’s
apple juice is to contain bacteria.

A

Correct Answer: E

E Inference

Find the answer choice supported by the passage.

A. No. The passage does not provide information about the amount of bacteria in other companies’ juices.

B. No. The passage doesn’t have any comparison between the amounts of bacteria in the apple juice versus the citrus juices. You can’t prove which juice is less likely to contain infectious bacteria.

C. No. Intensive pasteurization is likely to destroy the original flavor, but whether other types of pasteurization do this is not discussed in the passage. So there is not enough information to determine whether McElligott’s unpasteurized citrus juices retain more of the original flavor than do any pasteurized citrus juices.

D. No. This answer choice sounds good but is too strong. Intensive pasteurization is the most effective method for eliminating bacteria from juice. Intensive pasteurization is also likely to destroy the original flavor of the juice. The passage does not support, however, that intensive pasteurization is the method most likely to destroy flavor. Another method of eliminating bacteria might be even more likely to destroy the flavor.

E. Yes. McElligott’s juice did not undergo intensive pasteurization because it was flash pasteurized. It is also stated that intensive pasteurization is the most effective way to eliminate bacteria from the juice, so a juice that undergoes intensive pasteurization is less likely to contain bacteria than McElligott’s apple juice.

How well did you know this?
1
Not at all
2
3
4
5
Perfectly
25
Q

LSAT Logical Reasoning Prep Test 52 Question #25 25. Sociologist: Widespread acceptance of the idea that
individuals are incapable of looking after their
own welfare is injurious to a democracy. So
legislators who value democracy should not
propose any law prohibiting behavior that is
not harmful to anyone besides the person
engaging in it. After all, the assumptions that
appear to guide legislators will often become
widely accepted.
The sociologist’s argument requires the assumption
that
(A) democratically elected legislators invariably
have favorable attitudes toward the
preservation of democracy
(B) people tend to believe what is believed by
those who are prominent and powerful
(C) legislators often seem to be guided by the
assumption that individuals are incapable of
looking after their own welfare, even though
these legislators also seem to value democracy
(D) in most cases, behavior that is harmful to the
person who engages in it is harmful to no one
else
(E) a legislator proposing a law prohibiting an act
that can harm only the person performing the
act will seem to be assuming that individuals
are incapable of looking after their own
welfare

A

Correct Answer: E

E Necessary Assumption

The sociologist is solving a problem. The problem is that widespread acceptance of the idea that individuals are incapable of looking after their own welfare is injurious to a democracy. The sociologist concludes that legislators who value democracy should not propose any law that prohibits behavior harmful only to the person engaging in that behavior. The premise is that the assumptions that appear to guide legislators will often become widely accepted. There is a gap between discouraging the proposal of laws prohibiting actions harmful only to the person engaging in them and the assumptions that appear to guide these legislators. For the argument to work, the assumptions that appear to guide the legislators must involve the idea that individuals are incapable of looking after themselves.

A. No. The argument explicitly discusses legislators who value democracy, not all democratically elected legislators.

B. No. The sociologist doesn’t care about what the legislators actually believe about whether people are capable of looking after themselves. He/she is concerned with what the legislators appear to believe about this subject.

C. No. This is a description of how legislators often seem to be guided. The argument is a prescription for how legislators who value democracy often seem to be guided.

D. No. This might be true. However, it doesn’t give you the connection you need between what laws legislators should propose and what their underlying assumptions appear to be.

E. Yes. This connects the sociologist’s prescription for proposing laws and the perceived assumptions of the legislators.

How well did you know this?
1
Not at all
2
3
4
5
Perfectly
26
Q

LSAT PrepTest 52 Section #3 Question #1. Any museum that owns the rare stamp that features
an airplane printed upside down should not display
it. Ultraviolet light causes red ink to fade, and a
substantial portion of the stamp is red. If the stamp
is displayed, it will be damaged. It should be kept
safely locked away, even though this will deny the
public the chance to see it.
The reasoning above most closely conforms to which
one of the following principles?
(A) The public should judge the quality of a
museum by the rarity of the objects in its
collection.
(B) Museum display cases should protect their
contents from damage caused by ultraviolet
light.
(C) Red ink should not be used on items that will
not be exposed to ultraviolet light.
(D) A museum piece that would be damaged by
display should not be displayed.
(E) The primary purpose of a museum is to
educate the public.

A

Correct Answer: D

D Principle Match

The passage claims that any museum that owns the rare stamp that features an airplane printed upside down should not display it. This is because a substantial portion of the stamp is red, and ultraviolet light causes red ink to fade. The most important criterion for the conclusion seems to be that if the stamp is displayed, it will be damaged. The author acknowledges that the public will be denied the chance to see the stamp but maintains that the stamp ought to be kept safely locked away.

A. No. The passage does not mention whether the rare objects that a museum possesses should be displayed or not.

B. No. The passage states that if the stamp is displayed, it will be damaged. So, the stamp shouldn’t be displayed. It doesn’t address what features the museum display cases should have.

C. No. Red ink WAS used on this stamp and the stamp WILL be exposed to ultraviolet light if displayed. This principle doesn’t apply.

D. Yes. According to the passage, the stamp would be damaged by display and, therefore, should not be displayed.

E. No. If the primary purpose of a museum is to educate the public, it might be thought that the stamp should be displayed. Yet the passage states that it shouldn’t be, as it would be damaged.

How well did you know this?
1
Not at all
2
3
4
5
Perfectly
27
Q

LSAT PrepTest 52 Section #3 Question #2 Dietitian: Many diet-conscious consumers are excited
about new “fake fat” products designed to give
food the flavor and consistency of fatty foods,
yet without fat’s harmful effects. Consumers
who expect the new fat substitute to help them
lose weight arc likely to be disappointed,
however. Research has shown that when people
knowingly or unknowingly eat foods
containing “fake fat,” they tend to take in at
least as many additional calories as are saved
by eating “fake fat.”
Which one of the following most accurately expresses
the conclusion of the dietitian’s argument?
(A) People tend to take in a certain number of
daily calories, no matter what types of food
they eat
(B) Most consumers who think that foods with
“fake fat” are more nutritious than fatty foods
are destined to be disappointed.
(C) “Fake fat” products are likely to contribute to
obesity more than do other foods.
(D) “Fake fat” in foods is probably not going to
help consumers meet weight loss goals.
(E) “Fake fat” in foods is indistinguishable from
genuine fat by most consumers on the basis of
taste alone.

A

Correct Answer: D

D Main Point

The dietician warns that consumers who see a solution to their weight-loss problems in “fake fat” products are likely to be disappointed. This is because the people who either knowingly or unknowingly eat foods containing “fake fat” tend to take in at least as many additional calories as are saved by eating “fake fat” by eating more of the food.

A. No. This is too general. The dietician does not talk about the number of calories people consume, no matter what type of food they eat. The dietician is concerned primarily with fatty foods and their “fake fat” alternatives.

B. No. The dietician addresses only the number of calories consumed, not overall nutrition.

C. No. This is too strong. The dietician claims only that “fake fat” products are unlikely to help in weight loss, not that they will be more likely to contribute to obesity.

D. Yes. Since consumers of “fake fat” products tend to take in at least as many additional calories as are saved when eating these foods, it is unlikely that these foods will help them lose weight.

E. No. The dietician explains that “fake fat” products are designed to give food the flavor and consistency of fatty foods, but the dietician doesn’t explain whether or not they are successful in doing so.

How well did you know this?
1
Not at all
2
3
4
5
Perfectly
28
Q

LSAT PrepTest 52 Section #3 Question #3 Banking analyst: Banks often offer various services to
new customers at no charge. But this is not an
ideal business practice, since regular, long-term
customers, who make up the bulk of the
business for most banks, are excluded from
these special offers.
Which one of the following, if true, most strengthens
the banking analyst’s argument’?
(A) Most banks have similar charges for most
services and pay similar interest rates on
deposits.
(B) Banks do best when offering special privileges
only to their most loyal customers.
(C) Offering services at no charge to all of its
current customers would be prohibitively
expensive for a bank.
(D) Once they have chosen a bank, people tend to
remain loyal to that bank.
(E) Some banks that offer services at no charge to
new customers are very successful.

A

Correct Answer: B

B Strengthen

The banking analyst concludes that offering no-charge services to new customers is not an ideal business practice. The premise is that regular, long-term customers are excluded from these special offers and it is these customers who make up the bulk of business for most banks.

A. No. This argument is claiming that a no-fee service for new customers is not an ideal business practice. It does not discuss how much the banks charge for the services, normally.

B. Yes. If this is true, banks would do better to give their best offers and services to their long-term customers instead of focusing on new customers.

C. No. The banking analyst discourages offering no-fee services to new customers but does not discuss what banks should do instead.

D. No. If this were true, it would weaken the conclusion.

E. No. If this were true, it would weaken the conclusion.

How well did you know this?
1
Not at all
2
3
4
5
Perfectly
29
Q

LSAT PrepTest 52 Section #3 Question #4 Panelist: Medical research articles cited in popular
newspapers or magazines are more likely than
other medical research articles to be cited in
subsequent medical research. Thus, it appears
that medical researchers’ judgments of the
importance of prior research are strongly
influenced by the publicity received by that
research and do not strongly correspond to the
research’s true importance.
The panelist’s argument is most vulnerable to
criticism on the grounds that it
(A) presents counterarguments to a view that is
not actually held by any medical researcher
(B) fails to consider the possibility that popular
newspapers and magazines do a good job of
identifying the most important medical
research articles
(C) takes for granted that coverage of medical
research in the popular press is more
concerned with the eminence of the scientists
involved than with the content of their
research
(D) fails to consider the possibility that popular
newspapers and magazines are able to review
only a minuscule percentage of medical
research articles
(E) draws a conclusion that is logically equivalent
to its premise

A

Correct Answer: B

B Flaw

The panelist concludes that medical researchers’ judgments of the importance of prior research are strongly influenced by the research appearing in popular magazines and newspapers, and, therefore, the judgments do not strongly correspond to the research’s true importance. The premise is that medical research that is cited in popular magazines or newspapers is more likely to be cited in later medical research. The panelist fails to consider that there might be a good reason, common to both the citations in popular media and to later medical research, that these medical research articles are cited. Maybe it’s the most cutting edge and important medical research that gets cited in popular media, for example.

A. No. There is no view cited in the premises.

B. Yes. This would demonstrate that both the later medical research and the popular media have a good reason to cite this research.

C. No. The argument doesn’t talk about how esteemed or well-known the scientists are who completed the medical research that was cited in popular media.

D. No. The problem isn’t the percentage of medical research articles that the popular media are able to review; it is that the panelist doesn’t see that there might be a good reason that both the popular media and the later medical research are likely to cite the same research.

E. No. This is not a circular argument.

How well did you know this?
1
Not at all
2
3
4
5
Perfectly
30
Q

LSAT PrepTest 52 Section #3 Question #5 Lahar: We must now settle on a procedure for
deciding on meeting agendas. Our club’s
constitution allows three options: unanimous
consent, majority vote, or assigning the task to
a committee. Unanimous consent is unlikely.
Forming a committee has usually led to
factionalism and secret deals. Clearly, we
should subject meeting agendas to majority
vote.
Lahar’s argument does which one of the following?
(A) rejects suggested procedures on constitutional
grounds
(B) claims that one procedure is the appropriate
method for reaching every decision in the
club
(C) suggests a change to a constitution on the basis
of practical considerations
(D) recommends a choice based on the elimination
of alternative options
(E) supports one preference by arguing against
those who have advocated alternatives

A

Correct Answer: D

D Reasoning

Lahar concludes that his club should subject meeting agendas to majority vote. He does this through process of elimination. The club’s constitution allows three ways to decide on meeting agendas. Lahar claims that unanimous consent is unlikely and that forming a committee to decide this has usually led to groups of people pitted against each other and secret deals. The best option, then, is the one that remains: majority vote.

A. No. Lahar considers only the options that are allowed under the club’s constitution.

B. No. This is almost right. However, it’s not the credited response because the answer choice is too strong. He claims that one procedure is the appropriate method for reaching decisions on meeting agendas, not for reaching every decision.

C. No. Lahar does acknowledge practical considerations. However, he does not suggest a change to the constitution.

D. Yes. Lahar eliminates the other two options, leaving only the one that he recommends.

E. No. Lahar argues against alternatives but he does not argue against the people who have advocated alternatives.

How well did you know this?
1
Not at all
2
3
4
5
Perfectly
31
Q

LSAT PrepTest 52 Section #3 Question #6 Mayor: Local antitobacco activists are calling for
expanded antismoking education programs
paid for by revenue from heavily increased
taxes on cigarettes sold in the city. Although
the effectiveness of such education programs is
debatable, there is strong evidence that the
taxes themselves would produce the soughtafter reduction in smoking. Surveys show that
cigarette sales drop substantially in cities that
impose stiff tax increases on cigarettes.
Which one of the following, if true, most undermines
the reasoning in the argument above?
(A) A city-imposed tax on cigarettes will
substantially reduce the amount of smoking
in the city if the tax is burdensome to the
average cigarette consumer.
(B) Consumers are more likely to continue buying
a product if its price increases due to higher
taxes than if its price increases for some other
reason.
(C) Usually, cigarette sales will increase
substantially in the areas surrounding a city
after that city imposes stiff taxes on cigarettes.
(D) People who are well informed about the effects
of long-term tobacco use are significantly less
likely to smoke than are people who are not
informed.
(E) Antismoking education programs that are
funded by taxes on cigarettes will tend to lose
their funding if they are successful.

A

Correct Answer: C

C Weaken

This argument is designed to disagree with a position. The mayor disagrees with local activists calling for expanded antismoking educations programs, which would be paid for by revenue from greatly increased taxes on cigarettes sold in the city. The mayor thinks that these programs are unnecessary, as there is strong evidence that the taxes by themselves would produce the reduction in smoking that the antitobacco activists are seeking. He bases his conclusion on surveys showing that cigarette sales drop substantially in cities that impose these high tax increases on cigarettes.

A. No. This strengthens the argument because it makes a link between high taxes and substantially reduced smoking.

B. No. Consumers might be more likely to continue buying cigarettes if the price increase is due to higher taxes, but as long as less people actually buy and smoke the cigarettes, the mayor’s argument might still be valid.

C. Yes. If people are buying a lot more cigarettes outside of the city, the rate of smoking in that city might not decrease substantially. This answer choice exploits the language shift from cigarette sales to smoking rates.

D. No. This does not address the tax increase issue that is the crux of the mayor’s argument.

E. No. This does not address the mayor’s argument.

How well did you know this?
1
Not at all
2
3
4
5
Perfectly
32
Q

LSAT PrepTest 52 Section #3 Question #8 Caldwell: The government recently demolished a
former naval base. Among the complex’s
facilities were a gymnasium, a swimming pool,
office buildings, gardens, and housing for
hundreds of people. Of course the government
was legally permitted to use these facilities as it
wished. But clearly, using them for the good of
the community would have benefited
everyone, and thus the government’s actions
were not only inefficient but immoral.
Caldwell’s argument is most vulnerable to criticism
on the grounds that it
(A) fails to consider that an action may be morally
permissible even if an alternative course of
action is to everyone’s advantage
(B) presumes, without providing justification, that
the actual consequences of an action are
irrelevant to the action’s moral permissibility
(C) presumes, without providing justification, that
the government never acts in the most
efficient manner
(D) presumes, without providing justification, that
any action that is efficient is also moral
(E) inappropriately treats two possible courses of
action as if they were the only options

A

Correct Answer: A

A Flaw

Caldwell concludes that the government’s actions in tearing down a former naval base were not only inefficient but also immoral. Caldwell concludes this because the former naval base had a large number of facilities, such as a swimming pool, housing, etc., that might have been used for the good of the community, benefitting everyone. Thus, Caldwell equates what is moral with benefiting everyone.

A. Yes. Caldwell doesn’t allow for the possibility that the action of tearing down the base might still be morally permissible even if it doesn’t benefit everyone.

B. No. In fact, Caldwell thinks that the actual consequences are incredibly important to the action’s moral permissibility.

C. No. This is too strong. Caldwell is talking about the demolition of a single base, not the actions of the government in general.

D. No. This one is close. However, Caldwell says that the action was not only inefficient but also immoral. Caldwell isn’t equating being inefficient with being immoral. So Caldwell is also not presuming that any efficient action is also moral.

E. No. This one is tempting, as well. However, Caldwell doesn’t ever claim that tearing down the base or using it for the community are the only options. Other courses of action don’t happen to be relevant to Caldwell’s point.

How well did you know this?
1
Not at all
2
3
4
5
Perfectly
33
Q

LSAT PrepTest 52 Section #3 Question #7 Gotera: Infants lack the motor ability required to
voluntarily produce particular sounds, but
produce various babbling sounds randomly.
Most children are several years old before they
can voluntarily produce most of the vowel and
consonant sounds of their language. We can
conclude that speech acquisition is entirely a
motor control process rather than a process
that is abstract or mental.
Which one of the following is an assumption
required by Gotera’s argument?
(A) Speech acquisition is a function only of one’s
ability to produce the sounds of spoken
language.
(B) During the entire initial babbling stage, infants
cannot intentionally move their tongues while
they are babbling.
(C) The initial babbling stage is completed during
infancy.
(D) The initial babbling stage is the first stage of
the speech acquisition process.
(E) Control of tongue and mouth movements
requires a sophisticated level of mental
development.

A

Correct Answer: A

A Necessary Assumption

Gotera concludes that speech acquisition is entirely a motor control process, not one that is abstract or mental. Gotera bases this conclusion on two facts about infants and children. Infants don’t have the motor control required to voluntarily produce particular sounds. Instead, they produce random babbling sounds. Most children cannot voluntarily produce most of the vowel and consonant sounds of their language until they are several years old. Therefore, if children can’t voluntarily produce the sounds of their language, they can’t really talk. However, that doesn’t mean that motor control is the only process involved in speech acquisition. The conclusion is too strong for the evidence that supports it, so it requires an assumption to fill in the gap.

A. Yes. This eliminates the possibility that there might be some additional process that affects speech acquisition that does not involve motor control. This closes the gap.

B. No. It wouldn’t matter if infants could intentionally move their tongues, as long as they lack the motor ability to intentionally produce particular sounds. The evidence states that they do lack this ability.

C. No. It wouldn’t matter if children babbled until they were 7, as long as they also developed the motor abilities discussed in the argument.

D. No. This might be true. However, it doesn’t address the gap between the evidence and the claim that speech acquisition is entirely a motor control process.

E. No. This would weaken the argument. If this were true, then mental development would factor in to speech acquisition, against Gotera’s argument.

How well did you know this?
1
Not at all
2
3
4
5
Perfectly
34
Q

LSAT PrepTest 52 Section #3 Question #9 Reducing stress lessens a person’s sensitivity to pain.
This is the conclusion reached by researchers who
played extended audiotapes to patients before they
underwent surgery and afterward while they were
recovering. One tape consisted of conversation; the
other consisted of music. Those who listened only to
the latter tape required less anesthesia during surgery
and fewer painkillers afterward than those who
listened only to the former tape.
Which one of the following is an assumption on
which the researchers’ reasoning depends?
(A) All of the patients in the study listened to the
same tape before surgery as they listened to
after surgery.
(B) Anticipating surgery is no less stressful than
recovering from surgery.
(C) Listening to music reduces stress.
(D) The psychological effects of music are not
changed by anesthesia or painkillers.
(E) Both anesthesia and painkillers tend to reduce
stress.

A

Correct Answer: C

C Necessary Assumption

The researchers concluded that reducing stress lessens a person’s sensitivity to pain, based on an experiment where they played audiotapes to patients before and after surgery. The patients who listened only to music required less anesthesia and fewer painkillers as compared to the patients who listened only to conversation. The gap is between reducing stress and music. The music might have caused the patients to require less anesthesia and fewer painkillers for some other reason than because the music reduced stress.

A. No. It is not essential that the patients listened to the same tape before and after surgery, as long as one group listened to conversation and the other group listened to music.

B. No. This is not essential to the argument.

C. Yes. This makes the connection between music and stress reduction. It eliminates other possible reasons as to why the patients listening to music might have required less anesthesia and fewer painkillers.

D. No. This is not essential to the argument.

E. No. This might be true, but it doesn’t give us the connection between music and reducing stress that the argument requires.

How well did you know this?
1
Not at all
2
3
4
5
Perfectly
35
Q

LSAT PrepTest 52 Section #3 Question #10 Samuel: Because communication via computer is
usually conducted privately and anonymously
between people who would otherwise interact in
person, it contributes to the dissolution, not the
creation, of lasting communal bonds.
Tova: You assume that communication via computer
replaces more intimate forms of communication
and interaction, when more often it replaces
asocial or even antisocial behavior.
On the basis of their statements, Samuel and Tova
are committed to disagreeing about which one of the
following?
(A) A general trend of modern life is to dissolve
the social bonds that formerly connected
people.
(B) All purely private behavior contributes to the
dissolution of social bonds.
(C) Face-to-face communication is more likely to
contribute to the creation of social bonds
than is anonymous communication.
(D) It is desirable that new social bonds be created
to replace the ones that have dissolved.
(E) If people were not communicating via
computer, they would most likely be engaged
in activities that create stronger social bonds.

A

Correct Answer: E

E Point at Issue

Samuel concludes that communication via computer contributes to the dissolution of lasting communal bonds. His premise is that communication via computer is usually conducted privately and anonymously between people who would otherwise have conducted the communication in person. Tova disagrees with Samuel’s conclusion because she disagrees with his claim that communication via computer replaces other forms of interaction and communication. She claims, instead, that it replaces asocial or antisocial behavior.

A. No. Neither discusses the dissolution of social bonds as a general trend of modern life.

B. No. This is too strong. Neither talks about all purely private behavior. They are talking about a specific private behavior: communication via computer.

C. No. Tova doesn’t discuss whether face-to-face communication is more likely to contribute to the creation of social bonds.

D. No. Tova doesn’t discuss whether it is desirable to replace social bonds that have dissolved with new ones.

E. Yes. Samuel thinks that communicating via computer replaces social behavior, whereas Tova thinks that communicating via computer doesn’t because it replaces asocial/antisocial behavior.

How well did you know this?
1
Not at all
2
3
4
5
Perfectly
36
Q

LSAT PrepTest 52 Section #3 Question #11 Spreading iron particles over the surface of the
earth’s oceans would lead to an increase in
phytoplankton, decreasing the amount of carbon
dioxide in the atmosphere and thereby counteracting
the greenhouse effect. But while counteracting the
greenhouse effect is important, the side effects of an
iron-seeding strategy have yet to be studied. Since the
oceans represent such an important resource, this
response to the greenhouse effect should not be
implemented immediately.
The reasoning above most closely conforms to which
one of the following principles?
(A) A problem-solving strategy should be
implemented if the side effects of the strategy
are known.
(B) Implementing a problem-solving strategy that
alters an important resource is impermissible
if the consequences are not adequately
understood.
(C) We should not implement a problem-solving
strategy if the consequences of doing so are
more serious than the problem itself.
(D) We should not implement a problem-solving
strategy if that strategy requires altering an
important resource.
(E) As long as there is a possibility that a strategy
for solving a problem may instead exacerbate
that problem, such a solution should not be
adopted.

A

Correct Answer: B

B Principle Match

The passage concludes that we should not immediately spread iron particles over the surface of the ocean in response to the greenhouse effect. Spreading the iron particles would counteract the greenhouse effect by increasing the number of phytoplankton, which would, in turn, decrease the amount of carbon dioxide in the atmosphere. However, the side effects of this strategy haven’t been studied yet, so the author of the argument thinks we should hold off on messing with such an important resource as the ocean.

A. No. The passage doesn’t address strategies with known side effects.

B. Yes. According to the passage, the iron-seeding strategy should not be used yet because the consequences to the ocean, an important resource, are unknown.

C. No. The passage states that the consequences of the iron-seeding strategy are not known, so there is no way to know if those consequences are more serious than the problem of the greenhouse effect.

D. No. This is close, but it states that we should not implement a strategy if it requires altering an important resource. The passage doesn’t go that far. It just says that research into the side effects should be done before implementing the strategy.

E. No. The passage doesn’t concern itself with whether the iron-seeding strategy has a possibility of exacerbating the greenhouse effect. The worry is in how this strategy might alter the oceans.

How well did you know this?
1
Not at all
2
3
4
5
Perfectly
37
Q

LSAT PrepTest 52 Section #3 Question #12 No matter how conscientious they are, historians
always have biases that affect their work. Hence,
rather than trying to interpret historical events,
historians should instead interpret what the people
who participated in historical events thought about
those events.
The reasoning in the argument is most vulnerable to
criticism on the grounds that the argument fails to
consider the possibility that
(A) historians who have different biases often
agree about many aspects of some historical
events
(B) scholars in disciplines other than history also
risk having their biases affect their work
(C) many of the ways in which historians’ biases
affect their work have been identified
(D) not all historians are aware of the effect that
their particular biases have on their work
(E) the proposed shift in focus is unlikely to
eliminate the effect that historians’ biases
have on their work

A

Correct Answer: E

E Flaw

The problem that this argument tries to solve is that historians always have biases that affect their work, which is the interpretation of historical events. The solution that is proposed is that historians should instead interpret what the people who participated in historical events thought about those events. The flaw is that the solution still requires an interpretation that is susceptible to bias.

A. No. This might be true. However, the important point is that historians have biases that affect their work, which is what the proposal claims to avoid.

B. No. Scholars in other disciplines are not relevant to the flaw of this argument.

C. No. The argument is trying to eliminate the biases. It doesn’t matter whether or not these biases have been identified.

D. No. It doesn’t matter, for the purposes of this argument, whether the historians are aware of the effect of their biases. The argument proposes a solution to eliminate the biases whether the historians are aware of them or not.

E. Yes. If historians still have to interpret what the participants thought about the events, there is still the possibility of biased interpretations.

How well did you know this?
1
Not at all
2
3
4
5
Perfectly
38
Q

LSAT PrepTest 52 Section #3 Question #13 Humanitarian considerations aside, sheer economics
dictates that country X should institute, as country Y
has done, a nationwide system of air and ground
transportation for conveying seriously injured
persons to specialized trauma centers. Timely access
to the kind of medical care that only specialized
centers can provide could save the lives of many
people. The earnings of these people would result in
a substantial increase in country X’s gross national
product, and the taxes paid on those earnings would
substantially augment government revenues.
The argument depends on the assumption that
(A) lifetime per-capita income is roughly the same
in country X as it is in country Y
(B) there are no specialized trauma centers in
country X at present
(C) the treatment of seriously injured persons in
trauma centers is not more costly than
treatment elsewhere
(D) there would be a net increase in employment
in country X if more persons survived serious
injury
(E) most people seriously injured in automobile
accidents in country X do not now receive
treatment in specialized trauma centers

A

Correct Answer: D

D Necessary Assumption

The argument concludes that economic considerations dictate that country X should institute a nationwide system of air and ground transportation for conveying the seriously injured to specialized trauma centers. The evidence presented for this conclusion is that timely access to the medical care that can be provided only at trauma centers will save many people’s lives. These people, who are alive as a result of timely access to trauma centers, would be earning money. That these people are earning money is a twofold benefit to country X: The earnings would result in a large increase in X’s GNP, and taxes paid on the earnings would greatly augment government revenues. The argument shifts language from people’s lives being saved to the total increase in X’s GNP and tax revenues. An increase in X’s GNP and tax revenues would require that, as a result of saving these people’s lives, there would be more people working in country X than there are right now. If not, the added cost of instituting the nationwide system might not be economically beneficial.

A. No. Country Y is irrelevant. The argument concentrates on country X.

B. No. The argument is concerned with making sure that people have timely access to specialized trauma centers, wherever they may be located.

C. No. It could be more costly, and yet the people would be alive and working, which would increase country X’s GNP.

D. Yes. The argument assumes that if more people survived serious injury, they would be adding to the workforce, thereby boosting country X’s GNP. If the survivors weren’t able to work, or if they replaced other people when they went back to work, the GNP wouldn’t increase and neither would tax revenue.

E. No. The issue is whether country X should enable people to get timely treatment in specialized trauma centers, not whether more people should go to specialized trauma centers.

How well did you know this?
1
Not at all
2
3
4
5
Perfectly
39
Q

LSAT PrepTest 52 Section #3 Question #14 Early urban societies could not have been
maintained without large-scale farming nearby. This
is because other methods of food acquisition, such as
foraging, cannot support populations as dense as
urban ones. Large-scale farming requires irrigation,
which remained unfeasible in areas far from rivers or
lakes until more recent times.
Which one of the following is most strongly
supported by the information above?
(A) Most peoples who lived in early times lived in
areas near rivers or lakes.
(B) Only if farming is possible in the absence of
irrigation can societies be maintained in areas
far from rivers or lakes.
(C) In early times it was not possible to maintain
urban societies in areas far from rivers or
lakes.
(D) Urban societies with farms near rivers or lakes
do not have to rely upon irrigation to meet
their farming needs.
(E) Early rural societies relied more on foraging
than on agriculture for food.

A

Correct Answer: C

C Inference

Early urban societies → large-scale farming nearby → irrigation → rivers or lakes. The contrapositive is as follows: ~rivers and ~lakes → ~irrigation → ~large-scale farming → ~early urban society.

A. No. The passage talks about early urban societies, but does not say how many peoples lived in them.

B. No. This answer choice can be diagrammed as follows: societies far from rivers or lakes → farming possible in the absence of irrigation. The passage talks only about early urban societies. Also, it doesn’t talk about the possibility of farming without irrigation.

C. Yes. See the contrapositive in the explanation above.

D. No. Even though the passage does not say that urban societies with farms near rivers or lakes necessarily rely upon irrigation, it doesn’t say that they don’t.

E. No. The passage doesn’t mention early rural societies.

How well did you know this?
1
Not at all
2
3
4
5
Perfectly
40
Q

LSAT PrepTest 52 Section #3 Question #15 Economist: A country’s rapid emergence from an
economic recession requires substantial new
investment in that country’s economy. Since
people’s confidence in the economic policies of
their country is a precondition for any new
investment, countries that put collective goals
before individuals’ goals cannot emerge
quickly from an economic recession.
Which one of the following, if assumed, enables the
economist’s conclusion to be properly drawn?
(A) No new investment occurs in any country that
does not emerge quickly from an economic
recession.
(B) Recessions in countries that put collective
goals before individuals’ goals tend not to
affect the country’s people’s support for their
government’s policies.
(C) If the people in a country that puts
individuals’ goals first are willing to make
new investments in their country’s economy,
their country will emerge quickly from an
economic recession.
(D) People in countries that put collective goals
before individuals’ goals lack confidence in
the economic policies of their countries.
(E) A country’s economic policies are the most
significant factor determining whether that
country’s economy will experience a
recession.

A

Correct Answer: D

D Sufficient Assumption

The economist concludes that countries that put collective goals before individual goals cannot emerge quickly from an economic recession because people’s confidence in the economic policies of their country is a precondition for any new investment. This new investment is important because substantial new investment in that country’s economy is required for a country’s rapid emergence from an economic recession. The gap in the argument is between people’s confidence in a country’s economic policies and whether the country puts collective goals before individuals’ goals.

A. No. This is too strong. There might be some new investment, yet a country might still not emerge quickly from a recession. Emerging quickly from an economic recession requires substantial new investment.

B. No. The economist doesn’t discuss whether the recessions themselves affect people’s support for their government’s policies. Also, this answer choice is talking about the government’s policies in general, while the argument focuses on economic policies.

C. No. The economist never claims what would be sufficient for a country to emerge quickly from an economic recession. The economist, instead, tells us that substantial new investments are necessary for a country to emerge from an economic recession and that countries that put collective goals first will fail this necessary condition.

D. Yes. This connects people’s confidence, or lack thereof, in their country’s economic policies and whether the country puts collective goals before individuals’ goals.

E. No. The economist is concerned only with countries that have experienced a recession, not whether they are more likely to experience a recession.

How well did you know this?
1
Not at all
2
3
4
5
Perfectly
41
Q

LSAT PrepTest 52 Section #3 Question #16 The average length of stay for patients at Edgewater
Hospital is four days, compared to six days at
University Hospital. Since studies show that recovery
rates at the two hospitals are similar for patients with
similar illnesses, University Hospital could decrease
its average length of stay without affecting quality of
care.
The reasoning in the argument is most vulnerable to
criticism on the grounds that the argument
(A) equates the quality of care at a hospital with
patients’ average length of stay
(B) treats a condition that will ensure the
preservation of quality of care as a condition
that is required to preserve quality of care
(C) fails to take into account the possibility that
patients at Edgewater Hospital tend to be
treated for different illnesses than patients at
University Hospital
(D) presumes, without providing justification, that
the length of time patients stay in the hospital
is never relevant to the recovery rates of these
patients
(E) fails to take into account the possibility that
patients at University Hospital generally
prefer longer hospital stays

A

Correct Answer: C

C Flaw

The argument concludes that University Hospital could decrease its average length of stay without affecting quality of care. This conclusion is based on a comparison between the average length of stay for patients at Edgewater Hospital and at University Hospital. Studies show that recovery rates for both are similar for patients with similar illnesses. The argument is problematic because similar recovery rates for patients with similar illnesses at these hospitals does not mean that most patients at both hospitals have similar illnesses. University Hospital might specialize in heart attack patients, while Edgewater might specialize in minor injuries and ailments, for example.

A. No. The argument does not say that quality of care and length of stay are the same thing. Based on the evidence concerning recovery rates at both hospitals, it claims that length of stay can be reduced without affecting the quality of care.

B. No. Wrong flaw. This argument does not confuse something sufficient with something necessary.

C. Yes. If the patients at Edgewater tend to be treated for different illnesses, then the recovery rate information does not support the claim that the average length of stay could be reduced.

D. No. The argument allows for the idea that length of stay might be relevant to recovery rates.

E. No. The connection is between average length of stay, recovery rates, and quality of care. Patients’ preferences are irrelevant.

How well did you know this?
1
Not at all
2
3
4
5
Perfectly
42
Q

LSAT PrepTest 52 Section #3 Question #17 Philosopher: Graham argues that since a person is
truly happy only when doing something, the
best life is a life that is full of activity. But we
should not be persuaded by Graham’s
argument. People sleep, and at least sometimes
when sleeping, they are truly happy, even
though they are not doing anything.
Which one of the following most accurately describes
the role played in the philosopher’s argument by the
claim that at least sometimes when sleeping, people
are truly happy, even though they are not doing
anything?
(A) It is a premise of Graham’s argument.
(B) It is an example intended to show that a
premise of Graham’s argument is false.
(C) It is an analogy appealed to by Graham but
that the philosopher rejects.
(D) It is an example intended to disprove the
conclusion of Graham’s argument.
(E) It is the main conclusion of the philosopher’s
argument.

A

Correct Answer: B

B Reasoning

The philosopher disagrees with Graham’s position that the best life is a life that is full of activity because a person is truly happy only when doing something. The philosopher gives the example of people sleeping, during which they are not doing anything but are sometimes truly happy. The role of the example is to speak against Graham’s reason for his conclusion by undermining the claim that a person is truly happy only when doing something.

A. No. It is a premise of the philosopher’s argument.

B. Yes. If the statement is true, Graham cannot claim, as support for his conclusion, that a person is truly happy only when doing something.

C. No. It is not part of Graham’s argument.

D. No. This is close, but the claim is an example intended to disprove a premise of Graham’s argument, not its conclusion. The best life might still be a life that is full of activity; the philosopher just wants to show that Graham’s premise doesn’t actually support his conclusion.

E. No. It is a premise of the philosopher’s argument, not the conclusion.

How well did you know this?
1
Not at all
2
3
4
5
Perfectly
43
Q

LSAT PrepTest 52 Section #3 Question #18 Historian: In rebuttal of my claim that West
influenced Stuart, some people point out that
West’s work is mentioned only once in Stuart’s
diaries. But Stuart’s diaries mention several
meetings with West, and Stuart’s close friend,
Abella, studied under West. Furthermore,
Stuart’s work often uses West’s terminology
which, though now commonplace, none of
Stuart’s contemporaries used.
Which one of the following propositions is most
supported by the historian’s statements, if those
statements are true?
(A) Stuart’s discussions with Abella were one of
the means by which West influenced Stuart.
(B) It is more likely that Stuart influenced West
than that West influenced Stuart.
(C) Stuart’s contemporaries were not influenced
by West.
(D) Stuart’s work was not entirely free from West’s
influence
(E) Because of Stuart’s influence on other people,
West’s terminology is now commonplace.

A

Correct Answer: D

D Principle Match

Find the answer choice supported by the passage.

A. No. The passage does not provide proof whether Stuart had discussions with Abella about West, only that he was friends with Abella, who studied under West.

B. No. The historian is concerned with defending his claim that West influenced Stuart, not the other way around.

C. No. Stuart’s contemporaries didn’t use West’s terminology but there is no proof in the passage that West did or did not otherwise influence them.

D. Yes. Stuart used West’s terminology, which he couldn’t have gotten from his own contemporaries since they did not use the same terminology.

E. No. The passage does not provide any information on why West’s terminology is now commonplace, just that it is.

How well did you know this?
1
Not at all
2
3
4
5
Perfectly
44
Q

LSAT PrepTest 52 Section #3 Question #19 One theory to explain the sudden extinction of all
dinosaurs points to “drug overdoses” as the cause.
Angiosperms, a certain class of plants, first appeared
at the time that dinosaurs became extinct. These
plants produce amino-acid-based alkaloids that are
psychoactive agents. Most plant-eating mammals
avoid these potentially lethal poisons because they
taste bitter. Moreover, mammals have livers that help
detoxify such drugs. However, dinosaurs could
neither taste the bitterness nor detoxify the substance
once it was ingested. This theory receives its strongest
support from the fact that it helps explain why so
many dinosaur fossils are found in unusual and
contorted positions.
Which one of the following, if true, would most
undermine the theory presented above?
(A) Many fossils of large mammals are found in
contorted positions.
(B) Angiosperms provide a great deal of nutrition.
(C) Carnivorous dinosaurs mostly ate other,
vegetarian, dinosaurs that fed on
angiosperms.
(D) Some poisonous plants do not produce aminoacid-based alkaloids.
(E) Mammals sometimes die of drug overdoses
from eating angiosperms.

A

Correct Answer: A

A Weaken

The theory claims that “drug overdoses” were the cause of the sudden extinction of the dinosaurs. This helps explain why so many dinosaur fossils are found in unusual and contorted positions. The evidence is that angiosperms, which contain psychoactive agents, first appeared at the same time that the dinosaurs became extinct. Most plant-eating mammals avoid eating these agents, which are potentially lethal, because they taste bitter. Also, mammals have livers that help detoxify these agents. Dinosaurs couldn’t taste the bitterness, nor could they detoxify the substance. The argument claims that the theory’s strongest support comes from the fact that it helps explain the positions of dinosaur fossils. How does it explain the positions of dinosaur fossils? Might there be other reasons as to why the dinosaur fossils were in these positions?

A. Yes. This widens the gap. If fossils of large mammals are also found in these positions, and mammals were less likely to be poisoned by the psychoactive agents, then a key premise is compromised.

B. No. The argument is about whether dinosaurs were poisoned by angiosperms, not how nutritious angiosperms are.

C. No. This would strengthen the argument. If vegetarian dinosaurs fed on angiosperms and then other dinosaurs ate their angiosperm-polluted bodies, then a lot of dinosaurs would have been in contact with the potentially lethal psychoactive agents in the angiosperms.

D. No. The argument is about whether angiosperms poisoned the dinosaurs, not about other poisonous plants.

E. No. The argument allowed for this possibility. It just implied that it was less likely for them to die from eating angiosperms.

How well did you know this?
1
Not at all
2
3
4
5
Perfectly
45
Q

LSAT PrepTest 52 Section #3 Question #20 There are two ways to manage an existing
transportation infrastructure: continuous
maintenance at adequate levels, and periodic radical
reconstruction. Continuous maintenance dispenses
with the need for radical reconstruction, and radical
reconstruction is necessitated by failing to perform
continuous maintenance. Over the long run,
continuous maintenance is far less expensive;
nevertheless, it almost never happens.
Which one of the following, if true, most contributes
to an explanation of why the first alternative
mentioned is almost never adopted?
(A) Since different parts of the transportation
infrastructure are the responsibility of
different levels of government, radical
reconstruction projects are very difficult to
coordinate efficiently.
(B) When funds for transportation infrastructure
maintenance are scarce, they are typically
distributed in proportion to the amount of
traffic that is borne by different elements of
the infrastructure.
(C) If continuous maintenance is performed at
less-than-adequate levels, the need for radical
reconstruction will often arise later than if
maintenance had been restricted to
responding to emergencies.
(D) Radical reconstruction projects are, in general,
too costly to be paid for from current
revenue.
(E) For long periods, the task of regular
maintenance lacks urgency, since the
consequences of neglecting it are very slow to
manifest themselves.

A

Correct Answer: E

E Resolve/Explain

The discrepancy is that continuous maintenance, which dispenses with the need for radical reconstruction, is far less expensive a way to manage an existing transportation infrastructure in the long run. However, continuous maintenance almost never happens; rather, radical reconstruction, which is necessitated by failing to perform continuous maintenance, is usually the way that an existing transportation infrastructure is managed.

A. No. This would make it more, not less, likely that continuous maintenance would be performed.

B. No. This doesn’t address why continuous maintenance isn’t usually performed, even though it is less expensive in the long run.

C. No. This doesn’t address why continuous adequate maintenance isn’t usually performed.

D. No. This would make it more, not less, likely that continuous maintenance would be performed.

E. Yes. People feel that they can skip maintenance because the problems don’t show up immediately. When they do show up, they are more serious and necessitate radical reconstruction.

How well did you know this?
1
Not at all
2
3
4
5
Perfectly
46
Q

LSAT PrepTest 52 Section #3 Question #21 A good way to get over one’s fear of an activity one
finds terrifying is to do it repeatedly. For instance,
over half of people who have parachuted only once
report being extremely frightened by the experience,
while less than 1 percent of those who have
parachuted ten times or more report being
frightened by it.
The reasoning in the argument is most vulnerable to
criticism on the grounds that the argument
(A) takes for granted that the greater the number
of dangerous activities one engages in the less
one is frightened by any one of them
(B) neglects to consider those people who have
parachuted more than once but fewer than
ten times
(C) takes for granted that people do not know how
frightening something is unless they have
tried it
(D) fails to take into account the possibility that
people would be better off if they did not do
things that terrify them
(E) overlooks the possibility that most people who
have parachuted many times did not find it
frightening initially

A

Correct Answer: E

E Flaw

This argument is designed to solve a problem. The solution proposed for getting over one’s fear of an activity is to do it repeatedly. This is supported by the fact that over 50 percent of the people who have parachuted only once reported being extremely frightened by the experience, while less than 1 percent of people who have parachuted 10 times or more reported being frightened by it. This argument has a sampling problem. Who willingly throws themselves out of an airplane 10 times or more? It could be a higher percentage of people who never found it frightening in the first place.

A. No. The argument addresses overcoming the fear of some particular activity by repeating that same activity, not overcoming the fear of many activities by engaging in lots of frightening activities.

B. No. It’s true that the argument does not address this sample but this is not a flaw of the argument.

C. No. The argument addresses how to get over fears that people already have, no matter how they have acquired this fear.

D. No. What does it mean to be better off? If it means to be less frightened, then the argument does take this into account by saying that, in fact, repeating the activity will make them less frightened of it.

E. Yes. The sample might not be representative because it might include a higher percentage of people who never found it frightening to jump out of a plane in the first place.

How well did you know this?
1
Not at all
2
3
4
5
Perfectly
47
Q

LSAT PrepTest 52 Section #3 Question #22 Most economists believe that reducing the price of
any product generally stimulates demand for it.
However, most wine merchants have found that
reducing the price of domestic wines to make them
more competitive with imported wines with which
they were previously comparably priced is frequently
followed by an increase in sales of those imported
wines.
Which one of the following, if true, most helps to
reconcile the belief of most economists with the
consequences observed by most wine merchants?
(A) Economists’ studies of the prices of grocery
items and their rates of sales rarely cover
alcoholic beverages.
(B) Few merchants of any kind have detailed
knowledge of economic theories about the
relationship between item prices and sales
rates.
(C) Consumers are generally willing to forgo
purchasing other items they desire in order to
purchase a superior wine.
(D) Imported wines in all price ranges are
comparable in quality to domestic wines that
cost less.
(E) An increase in the demand for a consumer
product is compatible with an increase in
demand for a competing product.

A

Correct Answer: E

E Resolve/Explain

The seeming paradox is that although most economists believe that reducing the price of any product generally stimulates demand for it, when most wine merchants reduce the price of domestic wines to make them more competitive with imported wines with which they were previously comparably priced, the sales of the imported wines will increase.

A. No. This still doesn’t explain why, when the general rule is that reducing price stimulates demand, the competing wines sales increased when the prices on the domestic wine were reduced.

B. No. The merchants aren’t the ones making the economic predictions. They are the observers of what actually happened.

C. No. There is no information about which wine is superior. It doesn’t explain why the sales of imported wines increased when domestic wine prices were reduced.

D. No. This does not explain why, if the domestic wines’ prices were reduced, the sales of the imported wines increased.

E. Yes. The argument didn’t say whether the domestic wines’ sales also increased. This allows for the sales of both types of wines to increase, which reconciles the observations with the economists’ beliefs.

How well did you know this?
1
Not at all
2
3
4
5
Perfectly
48
Q

LSAT PrepTest 52 Section #3 Question #23 Certain bacteria that produce hydrogen sulfide as a
waste product would die if directly exposed to
oxygen. The hydrogen sulfide reacts with oxygen,
removing it and so preventing it from harming the
bacteria. Furthermore, the hydrogen sulfide tends to
kill other organisms in the area, thereby providing
the bacteria with a source of food. As a result, a dense
colony of these bacteria produces for itself an
environment in which it can continue to thrive
indefinitely.
Which one of the following is most strongly
supported by the information above?
(A) A dense colony of the bacteria can indefinitely
continue to produce enough hydrogen sulfide
to kill other organisms in the area and to
prevent oxygen from harming the bacteria.
(B) The hydrogen sulfide produced by the bacteria
kills other organisms in the area by reacting
with and removing oxygen.
(C) Most organisms, if killed by the hydrogen
sulfide produced by the bacteria, can provide
a source of food for the bacteria.
(D) The bacteria can continue to thrive indefinitely
only in an environment in which the
hydrogen sulfide they produce has removed
all oxygen and killed other organisms in the
area.
(E) If any colony of bacteria produces hydrogen
sulfide as a waste product, it thereby ensures
that it is both provided with a source of food
and protected from harm by oxygen.

A

Correct Answer: A

A Inference

Find the answer choice supported by the passage.

A. Yes. From the last sentence, a dense colony of these bacteria produces for itself an environment in which it can continue to thrive indefinitely. From the second and third sentence, the hydrogen sulfide produced by these bacteria reacts with oxygen, preventing it from harming the bacteria. It provides the bacteria with a source of food because it tends to kill other organisms in the area.

B. No. The passage states that the hydrogen sulfide reacts with and removes oxygen and that it tends to kill other organisms in the area. There is no proof that these two facts are related.

C. No. There is no proof in the passage that most organisms, if killed, can provide a source of food for the bacteria. All you know is that, with a dense colony, enough will be killed to sustain the colony. They could be light eaters.

D. No. There is no proof that, if they have this environment, the bacteria can thrive indefinitely or that it’s the only way that they can thrive indefinitely.

E. No. There is no information provided about all colonies of bacteria that might produce hydrogen sulfide as a waste product, just about these particular bacteria.

How well did you know this?
1
Not at all
2
3
4
5
Perfectly
49
Q

LSAT PrepTest 52 Section #3 Question #24 Books that present a utopian future in which the
inequities and sufferings of the present are replaced
by more harmonious and rational social
arrangements will always find enthusiastic buyers.
Since gloomy books predicting that even more
terrifying times await us are clearly not of this genre,
they are unlikely to be very popular.
The questionable pattern of reasoning in which one
of the following arguments is most similar to that in
the argument above?
(A) Art that portrays people as happy and
contented has a tranquilizing effect on the
viewer, an effect that is appealing to those
who are tense or anxious. Thus, people who
dislike such art are neither tense nor anxious.
(B) People who enjoy participating in activities
such as fishing or hiking may nevertheless
enjoy watching such spectator sports as
boxing or football. Thus, one cannot infer
from someone’s participating in vigorous
contact sports that he or she is not also fond
of less violent forms of recreation.
(C) Action movies that involve complicated and
dangerous special-effects scenes are
enormously expensive to produce. Hence,
since traditional dramatic or comedic films
contain no such scenes, it is probable that
they are relatively inexpensive to produce.
(D) Adults usually feel a pleasant nostalgia when
hearing the music they listened to as
adolescents, but since adolescents often like
music specifically because they think it
annoys their parents, adults rarely appreciate
the music that their children will later listen
to with nostalgia.
(E) All self-employed businesspeople have salaries
that fluctuate with the fortunes of the general
economy, but government bureaucrats are not
self-employed. Therefore, not everyone with
an income that fluctuates with the fortunes of
the general economy is a government
bureaucrat.

A

Correct Answer: C

C Parallel

The argument concludes that gloomy books are unlikely to be very popular. This is because they are not a genre that presents a utopian future, and books of the utopian future genre will always find enthusiastic buyers. Utopian future → popular; contrapositive: ~popular → ~Utopian future. Conclusion: ~Utopian future → ~(likely) popular. This argument has a necessary/sufficient problem. Being of the utopian future genre is sufficient for being popular but we don’t know that it is necessary. There might well be a group of angst-filled people that like nothing better than to curl up with a gloomy book.

A. No. This argument does not contain the necessary/sufficient switch.

B. No. This argument shifts from people who participate in less violent forms of recreation and enjoy watching more violent sports to people who participate in more violent sports. This is not the same as a necessary/sufficient flaw.

C. Yes. Complicated and dangerous special effects → enormously expensive; ~enormously expensive → ~complicated and dangerous special effects. Conclusion: ~complicated and dangerous special effects → (probably) ~enormously expensive.

D. No. This argument is not flawed.

E. No. This argument is not flawed.

How well did you know this?
1
Not at all
2
3
4
5
Perfectly
50
Q

LSAT PrepTest 52 Section #3 Question #25 Some people mistakenly believe that since we do not
have direct access to the distant past we cannot learn
much about it. Contemporary historians and
archaeologists find current geography, geology, and
climate to be rich in clues about a given region’s
distant history. However, the more distant the period
we are studying is, the less useful the study of the
present becomes.
Of the following, which one most closely conforms to
the principle that the passage illustrates?
(A) Astronomers often draw inferences about the
earlier years of our solar system on the basis
of recently collected data. Unfortunately, they
have been able to infer comparatively little
about the origin of our solar system.
(B) Much can be learned about the perpetrator of
a crime by applying scientific methods of
investigation to the crime scene. But the more
the crime scene has been studied the less
likely anything will be learned from further
study.
(C) To understand a literary text one needs to
understand the author’s world view. However,
the farther that world view gets from one’s
own the less one will be able to appreciate the
text.
(D) We often extrapolate from ordinary sensory
experience to things beyond such experience
and form a rash judgment, such as the claim
that the earth is the center of the universe
because it appears that way to us.
(E) One crucial clue to the extent of the ancient
Egyptians’ mathematical knowledge came
from studying the pyramids. The more we
studied such structures, the more impressed
we were by how much the Egyptians knew.

A

Correct Answer: A

A Parallel

The passage claims that we can learn about the past, even though we do not have direct access to it. We can do this by looking at current geology, geography, etc., to find clues about a region’s distant history. However, the study of the present becomes less useful the more distant the period we are studying is.

A. Yes. The astronomers are able to use present data about the solar system to find out about the earlier years of the solar system. However, because the origin of the solar system is much more distant, the present data is not very useful in learning about this origin.

B. No. The passage tells us that less will be learned from the present the more distant the period is that we are studying. The oft-studied crime scene is not from a much more distant period.

C. No. This doesn’t talk about time relations.

D. No. This doesn’t talk about time relations.

E. No. This doesn’t relate the pyramids to a much more distant time than the ancient Egyptians.

How well did you know this?
1
Not at all
2
3
4
5
Perfectly
51
Q

LSAT PrepTest 52 Section #4 Question #1 Many critics agree that the primary characteristic
of Senegalese filmmaker Ousmane Sembène’s work is
its sociopolitical commitment. Sembène was trained
in Moscow in the cinematic methods of socialist
realism, and he asserts that his films are not meant to
entertain his compatriots, but rather to raise their
awareness of the past and present realities of their
society. But his originality as a filmmaker lies most
strikingly in his having successfully adapted film,
originally a Western cultural medium, to the needs,
pace, and structures of West African culture. In
particular, Sembène has found within African oral
culture techniques and strategies that enable him to
express his views and to reach both literate and
nonliterate Senegalese viewers.
A number of Sembène’s characters and motifs can
be traced to those found in traditional West African
storytelling. The tree, for instance, which in countless
West African tales symbolizes knowledge, life, death,
and rebirth, is a salient motif in Emitaï. The trickster,
usually a dishonest individual who personifies
antisocial traits, appears in Borom Sarret, Mandabi,
and Xala as a thief, a corrupted civil servant, and a
member of the elite, respectively. In fact, most of
Sembène’s characters, like those of many oral West
African narratives, are types embodying collective
ideas or attitudes. And in the oral tradition, these
types face archetypal predicaments, as is true, for
example, of the protagonist of Borom Sarret, who has
no name and is recognizable instead by his trade—he
is a street merchant—and by the difficulties he
encounters but is unable to overcome.
Moreover, many of Sembène’s films derive their
structure from West African dilemma tales, the
outcomes of which are debated and decided by their
audiences. The open-endedness of most of his plots
reveals that Sembène similarly leaves it to his
viewers to complete his narratives: in such films as
Borom Sarret, Mandabi, and Ceddo, for example, he
provides his spectators with several alternatives as the
films end. The openness of his narratives is also
evidenced by his frequent use of freeze-frames, which
carry the suggestion of continued action.
Finally, like many West African oral tales,
Sembène’s narratives take the form of initiatory
journeys that bring about a basic change in the
worldview of the protagonist and ultimately, Sembène
hopes, in that of the viewer. His films denounce
social and political injustice. and his protagonists’
social consciousness emerges from an acute selfconsciousness brought about by the juxtaposition of
opposites within the films’ social context: good versus
evil, powerlessness versus power, or poverty versus
wealth. Such binary oppositions are used analogously
in West African tales, and it seems likely that these
dialectical elements are related to African oral
storytelling more than, as many critics have supposed,
to the Marxist components of his ideology.

  1. Which one of the following most accurately states the
    main point of the passage?
    (A) Sembène’s originality as a filmmaker lies in his
    adaptation of traditional archetypal
    predicaments and open-ended plots, both of
    which are derived from West African oral
    tales.
    (B) Many of the characters in Sembène’s films are
    variations on character types common to
    traditional West African storytelling.
    (C) Sembène’s films derive their distinctive
    characteristics from oral narrative traditions
    that had not previously been considered
    suitable subject matter for films.
    (D) Sembène’s films give vivid expression to the
    social and political beliefs held by most of the
    Senegalese people.
    (E) Sembène’s films are notable in that they use
    elements derived from traditional West
    African storytelling to comment critically on
    contemporary social and political issues.
A

Correct Answer: E

Passage Analysis

E Big Picture

A. No. This choice starts well but it claims that Sembène’s originality lies in his adaptation of traditional archetypal predicaments and open-ended plots, which were discussed only in the last two paragraphs.

B. No. This choice talks about Sembène’s characters being variations on types common to traditional West African storytelling, which is discussed only in the second paragraph.

C. No. We are never told whether oral narrative traditions were previously considered suitable.

D. No. We don’t know what social and political beliefs are held by most of the Senegalese people.

E. Yes. This choice mentions why Sembène’s films are notable, that they derive elements from traditional West African storytelling, and that these films use these elements to comment critically on contemporary social and political issues.

How well did you know this?
1
Not at all
2
3
4
5
Perfectly
52
Q

LSAT PrepTest 52 Section #4 Question #2 Many critics agree that the primary characteristic
of Senegalese filmmaker Ousmane Sembène’s work is
its sociopolitical commitment. Sembène was trained
in Moscow in the cinematic methods of socialist
realism, and he asserts that his films are not meant to
entertain his compatriots, but rather to raise their
awareness of the past and present realities of their
society. But his originality as a filmmaker lies most
strikingly in his having successfully adapted film,
originally a Western cultural medium, to the needs,
pace, and structures of West African culture. In
particular, Sembène has found within African oral
culture techniques and strategies that enable him to
express his views and to reach both literate and
nonliterate Senegalese viewers.
A number of Sembène’s characters and motifs can
be traced to those found in traditional West African
storytelling. The tree, for instance, which in countless
West African tales symbolizes knowledge, life, death,
and rebirth, is a salient motif in Emitaï. The trickster,
usually a dishonest individual who personifies
antisocial traits, appears in Borom Sarret, Mandabi,
and Xala as a thief, a corrupted civil servant, and a
member of the elite, respectively. In fact, most of
Sembène’s characters, like those of many oral West
African narratives, are types embodying collective
ideas or attitudes. And in the oral tradition, these
types face archetypal predicaments, as is true, for
example, of the protagonist of Borom Sarret, who has
no name and is recognizable instead by his trade—he
is a street merchant—and by the difficulties he
encounters but is unable to overcome.
Moreover, many of Sembène’s films derive their
structure from West African dilemma tales, the
outcomes of which are debated and decided by their
audiences. The open-endedness of most of his plots
reveals that Sembène similarly leaves it to his
viewers to complete his narratives: in such films as
Borom Sarret, Mandabi, and Ceddo, for example, he
provides his spectators with several alternatives as the
films end. The openness of his narratives is also
evidenced by his frequent use of freeze-frames, which
carry the suggestion of continued action.
Finally, like many West African oral tales,
Sembène’s narratives take the form of initiatory
journeys that bring about a basic change in the
worldview of the protagonist and ultimately, Sembène
hopes, in that of the viewer. His films denounce
social and political injustice. and his protagonists’
social consciousness emerges from an acute selfconsciousness brought about by the juxtaposition of
opposites within the films’ social context: good versus
evil, powerlessness versus power, or poverty versus
wealth. Such binary oppositions are used analogously
in West African tales, and it seems likely that these
dialectical elements are related to African oral
storytelling more than, as many critics have supposed,
to the Marxist components of his ideology.

  1. The author says that Sembène does which one of the
    following in at least some of his films?
    (A) uses animals as symbols
    (B) uses slow motion for artistic effect
    (C) provides oral narration of the film’s story
    (D) juxtaposes West African images and Marxist
    symbols
    (E) leaves part of the story to be filled in by
    audiences
A

Correct Answer: E

Passage Analysis

E Extract Fact

A. No. The author does not say that Sembène uses animals as symbols in any of his films.

B. No. This is close but the author claims that Sembène uses freeze-frames, which suggest continued action. The author does not claim that Sembène uses slow motion.

C. No. The author does not say that Sembène provides oral narration in any of his films.

D. No. The author does not claim that Sembène places West African images and Marxist symbols side by side in any of his films.

E. Yes. This answer choice is supported by the third paragraph.

How well did you know this?
1
Not at all
2
3
4
5
Perfectly
53
Q

LSAT PrepTest 52 Section #4 Question #3 Many critics agree that the primary characteristic
of Senegalese filmmaker Ousmane Sembène’s work is
its sociopolitical commitment. Sembène was trained
in Moscow in the cinematic methods of socialist
realism, and he asserts that his films are not meant to
entertain his compatriots, but rather to raise their
awareness of the past and present realities of their
society. But his originality as a filmmaker lies most
strikingly in his having successfully adapted film,
originally a Western cultural medium, to the needs,
pace, and structures of West African culture. In
particular, Sembène has found within African oral
culture techniques and strategies that enable him to
express his views and to reach both literate and
nonliterate Senegalese viewers.
A number of Sembène’s characters and motifs can
be traced to those found in traditional West African
storytelling. The tree, for instance, which in countless
West African tales symbolizes knowledge, life, death,
and rebirth, is a salient motif in Emitaï. The trickster,
usually a dishonest individual who personifies
antisocial traits, appears in Borom Sarret, Mandabi,
and Xala as a thief, a corrupted civil servant, and a
member of the elite, respectively. In fact, most of
Sembène’s characters, like those of many oral West
African narratives, are types embodying collective
ideas or attitudes. And in the oral tradition, these
types face archetypal predicaments, as is true, for
example, of the protagonist of Borom Sarret, who has
no name and is recognizable instead by his trade—he
is a street merchant—and by the difficulties he
encounters but is unable to overcome.
Moreover, many of Sembène’s films derive their
structure from West African dilemma tales, the
outcomes of which are debated and decided by their
audiences. The open-endedness of most of his plots
reveals that Sembène similarly leaves it to his
viewers to complete his narratives: in such films as
Borom Sarret, Mandabi, and Ceddo, for example, he
provides his spectators with several alternatives as the
films end. The openness of his narratives is also
evidenced by his frequent use of freeze-frames, which
carry the suggestion of continued action.
Finally, like many West African oral tales,
Sembène’s narratives take the form of initiatory
journeys that bring about a basic change in the
worldview of the protagonist and ultimately, Sembène
hopes, in that of the viewer. His films denounce
social and political injustice. and his protagonists’
social consciousness emerges from an acute selfconsciousness brought about by the juxtaposition of
opposites within the films’ social context: good versus
evil, powerlessness versus power, or poverty versus
wealth. Such binary oppositions are used analogously
in West African tales, and it seems likely that these
dialectical elements are related to African oral
storytelling more than, as many critics have supposed,
to the Marxist components of his ideology.

Which one of the following would, if true, most
strengthen the claim made by the author in the last
sentence of the passage (lines 54–58)?
(A) Several African novelists who draw upon the
oral traditions of West Africa use binary
oppositions as fundamental structures in
their narratives, even though they have not
read Marxist theory.
(B) Folklorists who have analyzed oral storytelling
traditions from across the world have found
that the use of binary oppositions to
structure narratives is common to many of
these traditions.
(C) When he trained in Moscow, Sembène read
extensively in Marxist political theory and
worked to devise ways of synthesizing
Marxist theory and the collective ideas
expressed in West African storytelling.
(D) Very few filmmakers in Europe or North
America make use of binary oppositions to
structure their narratives.
(E) Binary oppositions do not play an essential
structuring role in the narratives of some
films produced by other filmmakers who
subscribe to Marxist principles.

A

Correct Answer: A

Passage Analysis

A RC Reasoning

The answer choice should support the author’s reading of the dialectical elements as being related to African oral storytelling rather than to the Marxist components of Sembène’s ideology.

A. Yes. This choice demonstrates that other people who draw upon the oral traditions of West Africa do use these dialectical elements, and these people haven’t read Marxist theory. It lends credence to the author’s claim.

B. No. This is too general. The author claims that the binary oppositions are more likely to have come from African oral storytelling, not just some tradition from around the world.

C. No. This weakens the argument. If this were true, then the binary elements would be likely to come equally from his Marxist ideology.

D. No. This is not strong enough. There are other continents besides Europe, North America, and Africa. Also, he still could have gotten the binary oppositions from his Marxist ideology, even if few North American and European filmmakers use binary opposition.

E. No. This is not strong enough. It just says that some films produced by Marxist-principled filmmakers don’t essentially use binary opposition. They could still produce a lot of films that do.

How well did you know this?
1
Not at all
2
3
4
5
Perfectly
53
Q

LSAT PrepTest 52 Section #4 Question #4 Many critics agree that the primary characteristic
of Senegalese filmmaker Ousmane Sembène’s work is
its sociopolitical commitment. Sembène was trained
in Moscow in the cinematic methods of socialist
realism, and he asserts that his films are not meant to
entertain his compatriots, but rather to raise their
awareness of the past and present realities of their
society. But his originality as a filmmaker lies most
strikingly in his having successfully adapted film,
originally a Western cultural medium, to the needs,
pace, and structures of West African culture. In
particular, Sembène has found within African oral
culture techniques and strategies that enable him to
express his views and to reach both literate and
nonliterate Senegalese viewers.
A number of Sembène’s characters and motifs can
be traced to those found in traditional West African
storytelling. The tree, for instance, which in countless
West African tales symbolizes knowledge, life, death,
and rebirth, is a salient motif in Emitaï. The trickster,
usually a dishonest individual who personifies
antisocial traits, appears in Borom Sarret, Mandabi,
and Xala as a thief, a corrupted civil servant, and a
member of the elite, respectively. In fact, most of
Sembène’s characters, like those of many oral West
African narratives, are types embodying collective
ideas or attitudes. And in the oral tradition, these
types face archetypal predicaments, as is true, for
example, of the protagonist of Borom Sarret, who has
no name and is recognizable instead by his trade—he
is a street merchant—and by the difficulties he
encounters but is unable to overcome.
Moreover, many of Sembène’s films derive their
structure from West African dilemma tales, the
outcomes of which are debated and decided by their
audiences. The open-endedness of most of his plots
reveals that Sembène similarly leaves it to his
viewers to complete his narratives: in such films as
Borom Sarret, Mandabi, and Ceddo, for example, he
provides his spectators with several alternatives as the
films end. The openness of his narratives is also
evidenced by his frequent use of freeze-frames, which
carry the suggestion of continued action.
Finally, like many West African oral tales,
Sembène’s narratives take the form of initiatory
journeys that bring about a basic change in the
worldview of the protagonist and ultimately, Sembène
hopes, in that of the viewer. His films denounce
social and political injustice. and his protagonists’
social consciousness emerges from an acute selfconsciousness brought about by the juxtaposition of
opposites within the films’ social context: good versus
evil, powerlessness versus power, or poverty versus
wealth. Such binary oppositions are used analogously
in West African tales, and it seems likely that these
dialectical elements are related to African oral
storytelling more than, as many critics have supposed,
to the Marxist components of his ideology.

Which one of the following inferences about
Sembène is most strongly supported by the passage?
(A) His films have become popular both in parts of
Africa and elsewhere.
(B) He has not received support from government
agencies for his film production.
(C) His films are widely misunderstood by critics
in Senegal.
(D) His characters are drawn from a broad range
of social strata.
(E) His work has been subjected to government
censorship

A

Correct Answer: D

Passage Analysis

D Extract Infer

A. No. The passage doesn’t mention how popular Sembène’s films are.

B. No. The passage doesn’t mention the support of government agencies.

C. No. The passage doesn’t mention how the critics in Senegal interpret Sembène’s films.

D. Yes. This is discussed in the second paragraph.

E. No. The passage doesn’t mention government censorship.

How well did you know this?
1
Not at all
2
3
4
5
Perfectly
54
Q

LSAT PrepTest 52 Section #4 Question #5 Many critics agree that the primary characteristic
of Senegalese filmmaker Ousmane Sembène’s work is
its sociopolitical commitment. Sembène was trained
in Moscow in the cinematic methods of socialist
realism, and he asserts that his films are not meant to
entertain his compatriots, but rather to raise their
awareness of the past and present realities of their
society. But his originality as a filmmaker lies most
strikingly in his having successfully adapted film,
originally a Western cultural medium, to the needs,
pace, and structures of West African culture. In
particular, Sembène has found within African oral
culture techniques and strategies that enable him to
express his views and to reach both literate and
nonliterate Senegalese viewers.
A number of Sembène’s characters and motifs can
be traced to those found in traditional West African
storytelling. The tree, for instance, which in countless
West African tales symbolizes knowledge, life, death,
and rebirth, is a salient motif in Emitaï. The trickster,
usually a dishonest individual who personifies
antisocial traits, appears in Borom Sarret, Mandabi,
and Xala as a thief, a corrupted civil servant, and a
member of the elite, respectively. In fact, most of
Sembène’s characters, like those of many oral West
African narratives, are types embodying collective
ideas or attitudes. And in the oral tradition, these
types face archetypal predicaments, as is true, for
example, of the protagonist of Borom Sarret, who has
no name and is recognizable instead by his trade—he
is a street merchant—and by the difficulties he
encounters but is unable to overcome.
Moreover, many of Sembène’s films derive their
structure from West African dilemma tales, the
outcomes of which are debated and decided by their
audiences. The open-endedness of most of his plots
reveals that Sembène similarly leaves it to his
viewers to complete his narratives: in such films as
Borom Sarret, Mandabi, and Ceddo, for example, he
provides his spectators with several alternatives as the
films end. The openness of his narratives is also
evidenced by his frequent use of freeze-frames, which
carry the suggestion of continued action.
Finally, like many West African oral tales,
Sembène’s narratives take the form of initiatory
journeys that bring about a basic change in the
worldview of the protagonist and ultimately, Sembène
hopes, in that of the viewer. His films denounce
social and political injustice. and his protagonists’
social consciousness emerges from an acute selfconsciousness brought about by the juxtaposition of
opposites within the films’ social context: good versus
evil, powerlessness versus power, or poverty versus
wealth. Such binary oppositions are used analogously
in West African tales, and it seems likely that these
dialectical elements are related to African oral
storytelling more than, as many critics have supposed,
to the Marxist components of his ideology.

Which one of the following most closely expresses
the author’s intended meaning in using the word
“initiatory” (line 45)?
(A) beginning a series
(B) experimental
(C) transformative
(D) unprecedented
(E) prefatory

A

Correct Answer: C

Passage Analysis

C Structure

The author uses the phrase “initiatory journeys” to mean journeys that start or bring a basic change, which supports (C).

How well did you know this?
1
Not at all
2
3
4
5
Perfectly
55
Q

LSAT PrepTest 52 Section #4 Question #6 Many critics agree that the primary characteristic
of Senegalese filmmaker Ousmane Sembène’s work is
its sociopolitical commitment. Sembène was trained
in Moscow in the cinematic methods of socialist
realism, and he asserts that his films are not meant to
entertain his compatriots, but rather to raise their
awareness of the past and present realities of their
society. But his originality as a filmmaker lies most
strikingly in his having successfully adapted film,
originally a Western cultural medium, to the needs,
pace, and structures of West African culture. In
particular, Sembène has found within African oral
culture techniques and strategies that enable him to
express his views and to reach both literate and
nonliterate Senegalese viewers.
A number of Sembène’s characters and motifs can
be traced to those found in traditional West African
storytelling. The tree, for instance, which in countless
West African tales symbolizes knowledge, life, death,
and rebirth, is a salient motif in Emitaï. The trickster,
usually a dishonest individual who personifies
antisocial traits, appears in Borom Sarret, Mandabi,
and Xala as a thief, a corrupted civil servant, and a
member of the elite, respectively. In fact, most of
Sembène’s characters, like those of many oral West
African narratives, are types embodying collective
ideas or attitudes. And in the oral tradition, these
types face archetypal predicaments, as is true, for
example, of the protagonist of Borom Sarret, who has
no name and is recognizable instead by his trade—he
is a street merchant—and by the difficulties he
encounters but is unable to overcome.
Moreover, many of Sembène’s films derive their
structure from West African dilemma tales, the
outcomes of which are debated and decided by their
audiences. The open-endedness of most of his plots
reveals that Sembène similarly leaves it to his
viewers to complete his narratives: in such films as
Borom Sarret, Mandabi, and Ceddo, for example, he
provides his spectators with several alternatives as the
films end. The openness of his narratives is also
evidenced by his frequent use of freeze-frames, which
carry the suggestion of continued action.
Finally, like many West African oral tales,
Sembène’s narratives take the form of initiatory
journeys that bring about a basic change in the
worldview of the protagonist and ultimately, Sembène
hopes, in that of the viewer. His films denounce
social and political injustice. and his protagonists’
social consciousness emerges from an acute selfconsciousness brought about by the juxtaposition of
opposites within the films’ social context: good versus
evil, powerlessness versus power, or poverty versus
wealth. Such binary oppositions are used analogously
in West African tales, and it seems likely that these
dialectical elements are related to African oral
storytelling more than, as many critics have supposed,
to the Marxist components of his ideology.

The passage does NOT provide evidence that
Sembène exhibits which one of the following
attitudes in one or more of his films?
(A) disenchantment with attempts to reform
Senegalese government
(B) confidence in the aptness of using traditional
motifs to comment on contemporary issues
(C) concern with social justice
(D) interest in the vicissitudes of ordinary people’s
lives
(E) desire to educate his audience

A

Correct Answer: A

Passage Analysis

A Extract Fact

A. Yes. The passage never says that one of his films exhibits disenchantment with attempts to reform Senegalese government.

B. No. Paragraph two says that he does this a lot, which provides evidence that Sembène exhibits confidence in the aptness of it.

C. No. Paragraph four discusses this.

D. No. The example of the street merchant in the second paragraph shows this.

E. No. The first paragraph talks about his desire to raise awareness.

How well did you know this?
1
Not at all
2
3
4
5
Perfectly
56
Q

LSAT PrepTest 52 Section #4 Question #7 Passage A
Readers, like writers, need to search for answers.
Part of the joy of reading is in being surprised, but
academic historians leave little to the imagination. The
perniciousness of the historiographic approach became
fully evident to me when I started teaching. Historians
require undergraduates to read scholarly monographs
that sap the vitality of history; they visit on students
what was visited on them in graduate school. They
assign books with formulaic arguments that transform
history into an abstract debate that would have been
unfathomable to those who lived in the past. Aimed so
squarely at the head, such books cannot stimulate
students who yearn to connect to history emotionally as
well as intellectually.
In an effort to address this problem, some historians
have begun to rediscover stories. It has even become
something of a fad within the profession. This year, the
American Historical Association chose as the theme
for its annual conference some putative connection to
storytelling: “Practices of Historical Narrative.”
Predictably, historians responded by adding the word
“narrative” to their titles and presenting papers at
sessions on “Oral History and the Narrative of Class
Identity,” and “Meaning and Time: The Problem of
Historical Narrative.” But it was still historiography.
intended only for other academics. At meetings of
historians, we still encounter very few historians telling
stories or moving audiences to smiles, chills, or tears.
Passage B
Writing is at the heart of the lawyer’s craft, and so,
like it or not, we who teach the law inevitably teach
aspiring lawyers how lawyers write. We do this in a few
stand-alone courses and, to a greater extent, through the
constraints that we impose on their writing throughout
the curriculum. Legal writing, because of the purposes
it serves, is necessarily ruled by linear logic, creating a
path without diversions, surprises, or reversals.
Conformity is a virtue, creativity suspect, humor
forbidden, and voice mute.
Lawyers write as they see other lawyers write, and,
influenced by education, profession, economic
constraints, and perceived self-interest, they too often
write badly. Perhaps the currently fashionable call for
attention to narrative in legal education could have an
effect on this. It is not yet exactly clear what role
narrative should play in the law, but it is nonetheless
true that every case has at its heart a story—of real
events and people, of concerns, misfortunes, conflicts,
feelings. But because legal analysis strips the human
narrative content from the abstract, canonical legal
form of the case, law students learn to act as if there is
no such story.
It may well turn out that some of the terminology
and public rhetoric of this potentially subversive
movement toward attention to narrative will find its
way into the law curriculum, but without producing
corresponding changes in how legal writing is actually
taught or in how our future colleagues will write. Still,
even mere awareness of the value of narrative could
perhaps serve as an important corrective.

  1. Which one of the following does each of the passages
    display?
    (A) a concern with the question of what teaching
    methods are most effective in developing
    writing skills
    (B) a concern with how a particular discipline
    tends to represent points of view it does not
    typically deal with
    (C) a conviction that writing in specialized
    professional disciplines cannot be creatively
    crafted
    (D) a belief that the writing in a particular
    profession could benefit from more attention
    to storytelling
    (E) a desire to see writing in a particular field
    purged of elements from other disciplines
A

Correct Answer: D

Passage Analysis

D Extract Fact

A. No. Passage A does not discuss teaching methods to develop writing skills.

B. No. Passage A does not concern itself with points of view; rather, it is concerned with changing the way history is presented.

C. No. Passage A claims that historic writing can and should be creatively crafted; passage B at least raises the possibility that legal writing might be creatively crafted.

D. Yes. Both passages think that more attention to storytelling would benefit the writing in the respective professions that they discuss.

E. No. Both are entertaining the idea that elements from other disciplines would benefit writing in historical and legal writing, respectively.

How well did you know this?
1
Not at all
2
3
4
5
Perfectly
57
Q

LSAT PrepTest 52 Section #4 Question #8 Passage A
Readers, like writers, need to search for answers.
Part of the joy of reading is in being surprised, but
academic historians leave little to the imagination. The
perniciousness of the historiographic approach became
fully evident to me when I started teaching. Historians
require undergraduates to read scholarly monographs
that sap the vitality of history; they visit on students
what was visited on them in graduate school. They
assign books with formulaic arguments that transform
history into an abstract debate that would have been
unfathomable to those who lived in the past. Aimed so
squarely at the head, such books cannot stimulate
students who yearn to connect to history emotionally as
well as intellectually.
In an effort to address this problem, some historians
have begun to rediscover stories. It has even become
something of a fad within the profession. This year, the
American Historical Association chose as the theme
for its annual conference some putative connection to
storytelling: “Practices of Historical Narrative.”
Predictably, historians responded by adding the word
“narrative” to their titles and presenting papers at
sessions on “Oral History and the Narrative of Class
Identity,” and “Meaning and Time: The Problem of
Historical Narrative.” But it was still historiography.
intended only for other academics. At meetings of
historians, we still encounter very few historians telling
stories or moving audiences to smiles, chills, or tears.
Passage B
Writing is at the heart of the lawyer’s craft, and so,
like it or not, we who teach the law inevitably teach
aspiring lawyers how lawyers write. We do this in a few
stand-alone courses and, to a greater extent, through the
constraints that we impose on their writing throughout
the curriculum. Legal writing, because of the purposes
it serves, is necessarily ruled by linear logic, creating a
path without diversions, surprises, or reversals.
Conformity is a virtue, creativity suspect, humor
forbidden, and voice mute.
Lawyers write as they see other lawyers write, and,
influenced by education, profession, economic
constraints, and perceived self-interest, they too often
write badly. Perhaps the currently fashionable call for
attention to narrative in legal education could have an
effect on this. It is not yet exactly clear what role
narrative should play in the law, but it is nonetheless
true that every case has at its heart a story—of real
events and people, of concerns, misfortunes, conflicts,
feelings. But because legal analysis strips the human
narrative content from the abstract, canonical legal
form of the case, law students learn to act as if there is
no such story.
It may well turn out that some of the terminology
and public rhetoric of this potentially subversive
movement toward attention to narrative will find its
way into the law curriculum, but without producing
corresponding changes in how legal writing is actually
taught or in how our future colleagues will write. Still,
even mere awareness of the value of narrative could
perhaps serve as an important corrective.

  1. The passages most strongly support which one of the
    following inferences regarding the authors’
    relationships to the professions they discuss?
    (A) Neither author is an active member of the
    profession that he or she discusses.
    (B) Each author is an active member of the
    profession he or she discusses.
    (C) The author of passage A is a member of the
    profession discussed in that passage, but the
    author of passage B is not a member of either
    of the professions discussed in the passages.
    (D) Both authors are active members of the
    profession discussed in passage B.
    (E) The author of passage B, but not the author of
    passage A, is an active member of both of the
    professions discussed in the passages.
A

Correct Answer: B

Passage Analysis

B Extract Infer

A. No. The author of passage A is an active member of the history profession (line 5). The author of passage B is an active member of the legal profession (line 30).

B. Yes. See the explanation for (A).

C. No. The author of passage B is a member of the legal profession.

D. No. We don’t know whether or not the author of passage A is also a lawyer.

E. No. We don’t know whether or not the author of passage B is also a historian.

How well did you know this?
1
Not at all
2
3
4
5
Perfectly
58
Q

LSAT PrepTest 52 Section #4 Question #9
Passage A
Readers, like writers, need to search for answers.
Part of the joy of reading is in being surprised, but
academic historians leave little to the imagination. The
perniciousness of the historiographic approach became
fully evident to me when I started teaching. Historians
require undergraduates to read scholarly monographs
that sap the vitality of history; they visit on students
what was visited on them in graduate school. They
assign books with formulaic arguments that transform
history into an abstract debate that would have been
unfathomable to those who lived in the past. Aimed so
squarely at the head, such books cannot stimulate
students who yearn to connect to history emotionally as
well as intellectually.
In an effort to address this problem, some historians
have begun to rediscover stories. It has even become
something of a fad within the profession. This year, the
American Historical Association chose as the theme
for its annual conference some putative connection to
storytelling: “Practices of Historical Narrative.”
Predictably, historians responded by adding the word
“narrative” to their titles and presenting papers at
sessions on “Oral History and the Narrative of Class
Identity,” and “Meaning and Time: The Problem of
Historical Narrative.” But it was still historiography.
intended only for other academics. At meetings of
historians, we still encounter very few historians telling
stories or moving audiences to smiles, chills, or tears.
Passage B
Writing is at the heart of the lawyer’s craft, and so,
like it or not, we who teach the law inevitably teach
aspiring lawyers how lawyers write. We do this in a few
stand-alone courses and, to a greater extent, through the
constraints that we impose on their writing throughout
the curriculum. Legal writing, because of the purposes
it serves, is necessarily ruled by linear logic, creating a
path without diversions, surprises, or reversals.
Conformity is a virtue, creativity suspect, humor
forbidden, and voice mute.
Lawyers write as they see other lawyers write, and,
influenced by education, profession, economic
constraints, and perceived self-interest, they too often
write badly. Perhaps the currently fashionable call for
attention to narrative in legal education could have an
effect on this. It is not yet exactly clear what role
narrative should play in the law, but it is nonetheless
true that every case has at its heart a story—of real
events and people, of concerns, misfortunes, conflicts,
feelings. But because legal analysis strips the human
narrative content from the abstract, canonical legal
form of the case, law students learn to act as if there is
no such story.
It may well turn out that some of the terminology
and public rhetoric of this potentially subversive
movement toward attention to narrative will find its
way into the law curriculum, but without producing
corresponding changes in how legal writing is actually
taught or in how our future colleagues will write. Still,
even mere awareness of the value of narrative could
perhaps serve as an important corrective.

  1. Which one of the following does each passage
    indicate is typical of writing in the respective
    professions discussed in the passages?
    (A) abstraction
    (B) hyperbole
    (C) subversion
    (D) narrative
    (E) imagination
A

Correct Answer: A

Passage Analysis

A Extract Fact

A. Yes. Passage A mentions this in line 10 and passage B mentions this in line 49.

B. No. Both passages seem to indicate that writing in the respective professions is the opposite of hyperbolic.

C. No. Neither passage mentions that the writing in the respective professions is subversive.

D. No. This is the opposite of what both passages claim is typical for the writing in their respective professions.

E. No. This is the opposite of what both passages claim is typical for the writing in their respective professions.

How well did you know this?
1
Not at all
2
3
4
5
Perfectly
59
Q

LSAT PrepTest 52 Section #4 Question #10 Passage A
Readers, like writers, need to search for answers.
Part of the joy of reading is in being surprised, but
academic historians leave little to the imagination. The
perniciousness of the historiographic approach became
fully evident to me when I started teaching. Historians
require undergraduates to read scholarly monographs
that sap the vitality of history; they visit on students
what was visited on them in graduate school. They
assign books with formulaic arguments that transform
history into an abstract debate that would have been
unfathomable to those who lived in the past. Aimed so
squarely at the head, such books cannot stimulate
students who yearn to connect to history emotionally as
well as intellectually.
In an effort to address this problem, some historians
have begun to rediscover stories. It has even become
something of a fad within the profession. This year, the
American Historical Association chose as the theme
for its annual conference some putative connection to
storytelling: “Practices of Historical Narrative.”
Predictably, historians responded by adding the word
“narrative” to their titles and presenting papers at
sessions on “Oral History and the Narrative of Class
Identity,” and “Meaning and Time: The Problem of
Historical Narrative.” But it was still historiography.
intended only for other academics. At meetings of
historians, we still encounter very few historians telling
stories or moving audiences to smiles, chills, or tears.
Passage B
Writing is at the heart of the lawyer’s craft, and so,
like it or not, we who teach the law inevitably teach
aspiring lawyers how lawyers write. We do this in a few
stand-alone courses and, to a greater extent, through the
constraints that we impose on their writing throughout
the curriculum. Legal writing, because of the purposes
it serves, is necessarily ruled by linear logic, creating a
path without diversions, surprises, or reversals.
Conformity is a virtue, creativity suspect, humor
forbidden, and voice mute.
Lawyers write as they see other lawyers write, and,
influenced by education, profession, economic
constraints, and perceived self-interest, they too often
write badly. Perhaps the currently fashionable call for
attention to narrative in legal education could have an
effect on this. It is not yet exactly clear what role
narrative should play in the law, but it is nonetheless
true that every case has at its heart a story—of real
events and people, of concerns, misfortunes, conflicts,
feelings. But because legal analysis strips the human
narrative content from the abstract, canonical legal
form of the case, law students learn to act as if there is
no such story.
It may well turn out that some of the terminology
and public rhetoric of this potentially subversive
movement toward attention to narrative will find its
way into the law curriculum, but without producing
corresponding changes in how legal writing is actually
taught or in how our future colleagues will write. Still,
even mere awareness of the value of narrative could
perhaps serve as an important corrective.

In which one of the following ways are the passages
NOT parallel?
(A) Passage A presents and rejects arguments for
an opposing position, whereas passage B does
not.
(B) Passage A makes evaluative claims, whereas
passage B does not.
(C) Passage A describes specific examples of a
phenomenon it criticizes, whereas passage B
does not.
(D) Passage B offers criticism, whereas passage A
does not.
(E) Passage B outlines a theory, whereas passage A
does not.

A

Correct Answer: C

Passage Analysis

C Structure

A. No. Passage A does not present arguments for an opposing position.

B. No. This is close. However, passage B does make some evaluative claims, albeit weaker than the ones in passage A. It claims, for example, that mere awareness of the value of narrative could perhaps serve as an important corrective.

C. Yes. Passage A notes the titles of papers from the American Historical Association, which are all historiographs. Passage B does not give specific arguments.

D. No. Both offer criticism.

E. No. Passage B does not outline a theory.

How well did you know this?
1
Not at all
2
3
4
5
Perfectly
60
Q

LSAT PrepTest 52 Section #4 Question #11 Passage A
Readers, like writers, need to search for answers.
Part of the joy of reading is in being surprised, but
academic historians leave little to the imagination. The
perniciousness of the historiographic approach became
fully evident to me when I started teaching. Historians
require undergraduates to read scholarly monographs
that sap the vitality of history; they visit on students
what was visited on them in graduate school. They
assign books with formulaic arguments that transform
history into an abstract debate that would have been
unfathomable to those who lived in the past. Aimed so
squarely at the head, such books cannot stimulate
students who yearn to connect to history emotionally as
well as intellectually.
In an effort to address this problem, some historians
have begun to rediscover stories. It has even become
something of a fad within the profession. This year, the
American Historical Association chose as the theme
for its annual conference some putative connection to
storytelling: “Practices of Historical Narrative.”
Predictably, historians responded by adding the word
“narrative” to their titles and presenting papers at
sessions on “Oral History and the Narrative of Class
Identity,” and “Meaning and Time: The Problem of
Historical Narrative.” But it was still historiography.
intended only for other academics. At meetings of
historians, we still encounter very few historians telling
stories or moving audiences to smiles, chills, or tears.
Passage B
Writing is at the heart of the lawyer’s craft, and so,
like it or not, we who teach the law inevitably teach
aspiring lawyers how lawyers write. We do this in a few
stand-alone courses and, to a greater extent, through the
constraints that we impose on their writing throughout
the curriculum. Legal writing, because of the purposes
it serves, is necessarily ruled by linear logic, creating a
path without diversions, surprises, or reversals.
Conformity is a virtue, creativity suspect, humor
forbidden, and voice mute.
Lawyers write as they see other lawyers write, and,
influenced by education, profession, economic
constraints, and perceived self-interest, they too often
write badly. Perhaps the currently fashionable call for
attention to narrative in legal education could have an
effect on this. It is not yet exactly clear what role
narrative should play in the law, but it is nonetheless
true that every case has at its heart a story—of real
events and people, of concerns, misfortunes, conflicts,
feelings. But because legal analysis strips the human
narrative content from the abstract, canonical legal
form of the case, law students learn to act as if there is
no such story.
It may well turn out that some of the terminology
and public rhetoric of this potentially subversive
movement toward attention to narrative will find its
way into the law curriculum, but without producing
corresponding changes in how legal writing is actually
taught or in how our future colleagues will write. Still,
even mere awareness of the value of narrative could
perhaps serve as an important corrective.

The phrase “scholarly monographs that sap the
vitality of history” in passage A (lines 6–7) plays a
role in that passage’s overall argument that is most
analogous to the role played in passage B by which
one of the following phrases?
(A) “Writing is at the heart of the lawyer’s craft”
(line 29)
(B) “Conformity is a virtue, creativity suspect,
humor forbidden, and voice mute” (lines
37–38)
(C) “Lawyers write as they see other lawyers write”
(line 39)
(D) “every case has at its heart a story” (line 46)
(E) “Still, even mere awareness of the value of
narrative could perhaps serve as an important
corrective” (lines 57–59)

A

Correct Answer: B

Passage Analysis

B Structure

The phrase “scholarly monographs that sap the vitality of history” is being used to describe the typical writing that the author sees as making history books not stimulating for the students who read them, discouraging the students from connecting emotionally. In passage B, the phrases “conformity is a virtue, creativity suspect, humor forbidden, and voice mute” plays the most similar role, as it shows that the writing style of lawyers makes legal writing abstract and disconnected from the human narrative content. This supports (B).

How well did you know this?
1
Not at all
2
3
4
5
Perfectly
61
Q

LSAT PrepTest 52 Section #4 Question #12 Passage A
Readers, like writers, need to search for answers.
Part of the joy of reading is in being surprised, but
academic historians leave little to the imagination. The
perniciousness of the historiographic approach became
fully evident to me when I started teaching. Historians
require undergraduates to read scholarly monographs
that sap the vitality of history; they visit on students
what was visited on them in graduate school. They
assign books with formulaic arguments that transform
history into an abstract debate that would have been
unfathomable to those who lived in the past. Aimed so
squarely at the head, such books cannot stimulate
students who yearn to connect to history emotionally as
well as intellectually.
In an effort to address this problem, some historians
have begun to rediscover stories. It has even become
something of a fad within the profession. This year, the
American Historical Association chose as the theme
for its annual conference some putative connection to
storytelling: “Practices of Historical Narrative.”
Predictably, historians responded by adding the word
“narrative” to their titles and presenting papers at
sessions on “Oral History and the Narrative of Class
Identity,” and “Meaning and Time: The Problem of
Historical Narrative.” But it was still historiography.
intended only for other academics. At meetings of
historians, we still encounter very few historians telling
stories or moving audiences to smiles, chills, or tears.
Passage B
Writing is at the heart of the lawyer’s craft, and so,
like it or not, we who teach the law inevitably teach
aspiring lawyers how lawyers write. We do this in a few
stand-alone courses and, to a greater extent, through the
constraints that we impose on their writing throughout
the curriculum. Legal writing, because of the purposes
it serves, is necessarily ruled by linear logic, creating a
path without diversions, surprises, or reversals.
Conformity is a virtue, creativity suspect, humor
forbidden, and voice mute.
Lawyers write as they see other lawyers write, and,
influenced by education, profession, economic
constraints, and perceived self-interest, they too often
write badly. Perhaps the currently fashionable call for
attention to narrative in legal education could have an
effect on this. It is not yet exactly clear what role
narrative should play in the law, but it is nonetheless
true that every case has at its heart a story—of real
events and people, of concerns, misfortunes, conflicts,
feelings. But because legal analysis strips the human
narrative content from the abstract, canonical legal
form of the case, law students learn to act as if there is
no such story.
It may well turn out that some of the terminology
and public rhetoric of this potentially subversive
movement toward attention to narrative will find its
way into the law curriculum, but without producing
corresponding changes in how legal writing is actually
taught or in how our future colleagues will write. Still,
even mere awareness of the value of narrative could
perhaps serve as an important corrective.

Suppose that a lawyer is writing a legal document
describing the facts that are at issue in a case. The
author of passage B would be most likely to expect
which one of the following to be true of the
document?
(A) It will be poorly written because the lawyer
who is writing it was not given explicit advice
by law professors on how lawyers should
write.
(B) It will be crafted to function like a piece of
fiction in its description of the characters and
motivations of the people involved in the
case.
(C) It will be a concise, well-crafted piece of
writing that summarizes most, if not all, of
the facts that are important in the case.
(D) It will not genuinely convey the human
dimension of the case, regardless of how
accurate the document may be in its details.
(E) It will neglect to make appropriate connections
between the details of the case and relevant
legal doctrines.

A

Correct Answer: D

Passage Analysis

D Extract Infer

A. No. This is too strong. The author says that lawyers too often write badly (lines 41–42). This doesn’t mean that legal documents are always written poorly.

B. No. The author claims that legal analysis strips the human narrative content from the abstract, canonical legal form of the case (lines 48–50).

C. No. The author claims that lawyers too often write poorly (lines 41–42).

D. Yes. The author claims that legal analysis strips the human narrative content from the abstract, canonical legal form of the case (lines 48–50).

E. No. The author doesn’t discuss whether legal writing makes the right connections between the details and relevant legal doctrines.

How well did you know this?
1
Not at all
2
3
4
5
Perfectly
62
Q

LSAT PrepTest 52 Section #4 Question #13 Traditional theories of animal behavior assert that
animal conflict within a species is highly ritualized
and does not vary from contest to contest. This
species-specific model assumes that repetitive use of
the same visual and vocal displays and an absence of
escalated fighting evolved to prevent injury. The
contestant that exhibits the “best” display wins the
contested resource. Galápagos tortoises, for instance,
settle contests on the basis of height: the ritualized
display consists of two tortoises facing one another
and stretching their necks skyward; the tortoise
perceived as being “taller” wins.
In populations of the spider Agelenopsis aperta,
however, fighting behavior varies greatly from contest
to contest. In addition, fighting is not limited to
displays: biting and shoving are common. Susan
Riechert argues that a recently developed model,
evolutionary game theory, provides a closer fit to
A. aperta territorial disputes than does the speciesspecific model, because it explains variations in
conflict behavior that may result from varying
conditions, such as differences in size, age, and
experience of combatants. Evolutionary game theory
was adapted from the classical game theory that was
developed by von Neumann and Morganstern to
explain human behavior in conflict situations. In both
classical and evolutionary game theory, strategies are
weighed in terms of maximizing the average payoff
against contestants employing both the same and
different strategies. For example, a spider may engage
in escalated fighting during a dispute only if the
disputed resource is valuable enough to warrant the
risk of physical injury. There are, however, two major
differences between the classical and evolutionary
theories. First, whereas in classical game theory it is
assumed that rational thought is used to determine
which action to take, evolutionary game theory
assumes that instinct and long-term species advantage
ultimately determine the strategies that are exhibited.
The other difference is in the payoffs: in classical
game theory, the payoffs are determined by an
individual’s personal judgment of what constitutes
winning; in evolutionary game theory, the payoffs are
defined in terms of reproductive success.
In studying populations of A. aperta in a
grassland habitat and a riparian habitat, Riechert
predicts that such factors as the size of the opponents,
the potential rate of predation in a habitat, and the
probability of winning a subsequent site if the dispute
is lost will all affect the behavior of spiders in
territorial disputes. In addition, she predicts that the
markedly different levels of competition for web sites
in the two habitats will affect the spiders’ willingness
to engage in escalated fighting. In the grassland,
where 12 percent of the habitat is available for
occupation by A. aperta, Riechert predicts that
spiders will be more willing to engage in escalated
fighting than in the riparian habitat, where 90 percent
of the habitat is suitable for occupation.

Which one of the following best states the main idea
of the passage?
(A) Evolutionary game theory and classical game
theory can be used to analyze the process of
decision-making used by humans and
animals in settling disputes.
(B) A. aperta in grassland habitats and riparian
habitats exhibit an unusually wide variety of
fighting behaviors in territorial disputes.
(C) Evolutionary game theory may be useful in
explaining the behavior of certain spiders
during territorial disputes.
(D) The traditional theory of animal behavior in
conflict situations cannot be used to explain
the fighting behavior of most species.
(E) Evolutionary game theory, adapted from
classical game theory, is currently used by
scientists to predict the behavior of spiders in
site selection.

A

Correct Answer: C

Passage Analysis

C Big Picture

A. No. This is too general. The passage discusses only evolutionary game theory in terms of its success in analyzing the process of decision making, and only in regards to Agelenopsis apertas.

B. No. This was discussed only in the third paragraph. Also, we don’t know that they exhibit an unusually wide variety of behaviors.

C. Yes. This mentions evolutionary game theory and how it might be used to explain the behavior of Agelenopsis apertas in intra-species disputes.

D. No. This is too strong. The author suggests that the traditional theory might not be as good as evolutionary game theory to explain the fighting behavior of Agelenopsis apertas, but doesn’t address most species.

E. No. This is too general. We don’t know that scientists in general use evolutionary game theory to predict the behavior of spiders in site selection.

How well did you know this?
1
Not at all
2
3
4
5
Perfectly
63
Q

LSAT PrepTest 52 Section #4 Question #14 Traditional theories of animal behavior assert that
animal conflict within a species is highly ritualized
and does not vary from contest to contest. This
species-specific model assumes that repetitive use of
the same visual and vocal displays and an absence of
escalated fighting evolved to prevent injury. The
contestant that exhibits the “best” display wins the
contested resource. Galápagos tortoises, for instance,
settle contests on the basis of height: the ritualized
display consists of two tortoises facing one another
and stretching their necks skyward; the tortoise
perceived as being “taller” wins.
In populations of the spider Agelenopsis aperta,
however, fighting behavior varies greatly from contest
to contest. In addition, fighting is not limited to
displays: biting and shoving are common. Susan
Riechert argues that a recently developed model,
evolutionary game theory, provides a closer fit to
A. aperta territorial disputes than does the speciesspecific model, because it explains variations in
conflict behavior that may result from varying
conditions, such as differences in size, age, and
experience of combatants. Evolutionary game theory
was adapted from the classical game theory that was
developed by von Neumann and Morganstern to
explain human behavior in conflict situations. In both
classical and evolutionary game theory, strategies are
weighed in terms of maximizing the average payoff
against contestants employing both the same and
different strategies. For example, a spider may engage
in escalated fighting during a dispute only if the
disputed resource is valuable enough to warrant the
risk of physical injury. There are, however, two major
differences between the classical and evolutionary
theories. First, whereas in classical game theory it is
assumed that rational thought is used to determine
which action to take, evolutionary game theory
assumes that instinct and long-term species advantage
ultimately determine the strategies that are exhibited.
The other difference is in the payoffs: in classical
game theory, the payoffs are determined by an
individual’s personal judgment of what constitutes
winning; in evolutionary game theory, the payoffs are
defined in terms of reproductive success.
In studying populations of A. aperta in a
grassland habitat and a riparian habitat, Riechert
predicts that such factors as the size of the opponents,
the potential rate of predation in a habitat, and the
probability of winning a subsequent site if the dispute
is lost will all affect the behavior of spiders in
territorial disputes. In addition, she predicts that the
markedly different levels of competition for web sites
in the two habitats will affect the spiders’ willingness
to engage in escalated fighting. In the grassland,
where 12 percent of the habitat is available for
occupation by A. aperta, Riechert predicts that
spiders will be more willing to engage in escalated
fighting than in the riparian habitat, where 90 percent
of the habitat is suitable for occupation.

The author of the passage mentions Galápagos
tortoises in the first paragraph most likely in order to
(A) describe a kind of fighting behavior that is
used by only a few species
(B) suggest that repetitive use of the same visual
and vocal displays is a kind of fighting
behavior used by some but not all species
(C) provide evidence to support the claim that
fighting behavior does not vary greatly from
contest to contest for most species
(D) provide an example of a fighting behavior that
is unique to a particular species
(E) provide an example of a ritualized fighting
behavior of the kind that traditional theorists
assume is the norm for most species

A

Correct Answer: E

Passage Analysis

E Structure

The author mentions Galapagos tortoises as an example of the traditional theory, which claims that intra-species conflict is highly ritualized. This supports (E).

How well did you know this?
1
Not at all
2
3
4
5
Perfectly
64
Q

LSAT PrepTest 52 Section #4 Question #15 Traditional theories of animal behavior assert that
animal conflict within a species is highly ritualized
and does not vary from contest to contest. This
species-specific model assumes that repetitive use of
the same visual and vocal displays and an absence of
escalated fighting evolved to prevent injury. The
contestant that exhibits the “best” display wins the
contested resource. Galápagos tortoises, for instance,
settle contests on the basis of height: the ritualized
display consists of two tortoises facing one another
and stretching their necks skyward; the tortoise
perceived as being “taller” wins.
In populations of the spider Agelenopsis aperta,
however, fighting behavior varies greatly from contest
to contest. In addition, fighting is not limited to
displays: biting and shoving are common. Susan
Riechert argues that a recently developed model,
evolutionary game theory, provides a closer fit to
A. aperta territorial disputes than does the speciesspecific model, because it explains variations in
conflict behavior that may result from varying
conditions, such as differences in size, age, and
experience of combatants. Evolutionary game theory
was adapted from the classical game theory that was
developed by von Neumann and Morganstern to
explain human behavior in conflict situations. In both
classical and evolutionary game theory, strategies are
weighed in terms of maximizing the average payoff
against contestants employing both the same and
different strategies. For example, a spider may engage
in escalated fighting during a dispute only if the
disputed resource is valuable enough to warrant the
risk of physical injury. There are, however, two major
differences between the classical and evolutionary
theories. First, whereas in classical game theory it is
assumed that rational thought is used to determine
which action to take, evolutionary game theory
assumes that instinct and long-term species advantage
ultimately determine the strategies that are exhibited.
The other difference is in the payoffs: in classical
game theory, the payoffs are determined by an
individual’s personal judgment of what constitutes
winning; in evolutionary game theory, the payoffs are
defined in terms of reproductive success.
In studying populations of A. aperta in a
grassland habitat and a riparian habitat, Riechert
predicts that such factors as the size of the opponents,
the potential rate of predation in a habitat, and the
probability of winning a subsequent site if the dispute
is lost will all affect the behavior of spiders in
territorial disputes. In addition, she predicts that the
markedly different levels of competition for web sites
in the two habitats will affect the spiders’ willingness
to engage in escalated fighting. In the grassland,
where 12 percent of the habitat is available for
occupation by A. aperta, Riechert predicts that
spiders will be more willing to engage in escalated
fighting than in the riparian habitat, where 90 percent
of the habitat is suitable for occupation.

A

N/A

How well did you know this?
1
Not at all
2
3
4
5
Perfectly
65
Q

LSAT PrepTest 52 Section #4 Question #16 Traditional theories of animal behavior assert that
animal conflict within a species is highly ritualized
and does not vary from contest to contest. This
species-specific model assumes that repetitive use of
the same visual and vocal displays and an absence of
escalated fighting evolved to prevent injury. The
contestant that exhibits the “best” display wins the
contested resource. Galápagos tortoises, for instance,
settle contests on the basis of height: the ritualized
display consists of two tortoises facing one another
and stretching their necks skyward; the tortoise
perceived as being “taller” wins.
In populations of the spider Agelenopsis aperta,
however, fighting behavior varies greatly from contest
to contest. In addition, fighting is not limited to
displays: biting and shoving are common. Susan
Riechert argues that a recently developed model,
evolutionary game theory, provides a closer fit to
A. aperta territorial disputes than does the speciesspecific model, because it explains variations in
conflict behavior that may result from varying
conditions, such as differences in size, age, and
experience of combatants. Evolutionary game theory
was adapted from the classical game theory that was
developed by von Neumann and Morganstern to
explain human behavior in conflict situations. In both
classical and evolutionary game theory, strategies are
weighed in terms of maximizing the average payoff
against contestants employing both the same and
different strategies. For example, a spider may engage
in escalated fighting during a dispute only if the
disputed resource is valuable enough to warrant the
risk of physical injury. There are, however, two major
differences between the classical and evolutionary
theories. First, whereas in classical game theory it is
assumed that rational thought is used to determine
which action to take, evolutionary game theory
assumes that instinct and long-term species advantage
ultimately determine the strategies that are exhibited.
The other difference is in the payoffs: in classical
game theory, the payoffs are determined by an
individual’s personal judgment of what constitutes
winning; in evolutionary game theory, the payoffs are
defined in terms of reproductive success.
In studying populations of A. aperta in a
grassland habitat and a riparian habitat, Riechert
predicts that such factors as the size of the opponents,
the potential rate of predation in a habitat, and the
probability of winning a subsequent site if the dispute
is lost will all affect the behavior of spiders in
territorial disputes. In addition, she predicts that the
markedly different levels of competition for web sites
in the two habitats will affect the spiders’ willingness
to engage in escalated fighting. In the grassland,
where 12 percent of the habitat is available for
occupation by A. aperta, Riechert predicts that
spiders will be more willing to engage in escalated
fighting than in the riparian habitat, where 90 percent
of the habitat is suitable for occupation.

Which one of the following, if true, is LEAST
consistent with Riechert’s theory about fighting
behavior in spiders?
(A) Spiders in the grassland habitat engage in
escalated fighting when a disputed site is
highly desirable.
(B) Spiders in the riparian habitat are not willing
to engage in escalated fighting for less-thansuitable sites.
(C) Spiders in the riparian habitat confine their
fighting to displays more regularly than do
spiders in the grassland habitat.
(D) Spiders in the riparian habitat are as willing to
engage in escalated fighting as are spiders in
the grassland habitat.
(E) Spiders in the riparian habitat are more likely
to withdraw when faced with a larger
opponent in territorial disputes than are
spiders in the grassland habitat.

A

Correct Answer: D

Passage Analysis

D Extract Infer

A. No. This is consistent with the predictions concerning the grassland in paragraph three.

B. No. This is consistent with the predictions concerning the riparian habitat in paragraph three, as the riparian habitat is 90 percent habitable.

C. No. The third paragraph predicts that spiders in the riparian habitat, which is 90 percent habitable, will be less likely to escalate fighting than spiders in the grassland habitat, which is 12 percent habitable.

D. Yes. The third paragraph predicts that spiders in the riparian habitat will be less willing to engage in escalated fighting, not as willing.

E. No. This is consistent with the predictions in the third paragraph.

How well did you know this?
1
Not at all
2
3
4
5
Perfectly
66
Q

LSAT PrepTest 52 Section #4 Question #17 Traditional theories of animal behavior assert that
animal conflict within a species is highly ritualized
and does not vary from contest to contest. This
species-specific model assumes that repetitive use of
the same visual and vocal displays and an absence of
escalated fighting evolved to prevent injury. The
contestant that exhibits the “best” display wins the
contested resource. Galápagos tortoises, for instance,
settle contests on the basis of height: the ritualized
display consists of two tortoises facing one another
and stretching their necks skyward; the tortoise
perceived as being “taller” wins.
In populations of the spider Agelenopsis aperta,
however, fighting behavior varies greatly from contest
to contest. In addition, fighting is not limited to
displays: biting and shoving are common. Susan
Riechert argues that a recently developed model,
evolutionary game theory, provides a closer fit to
A. aperta territorial disputes than does the speciesspecific model, because it explains variations in
conflict behavior that may result from varying
conditions, such as differences in size, age, and
experience of combatants. Evolutionary game theory
was adapted from the classical game theory that was
developed by von Neumann and Morganstern to
explain human behavior in conflict situations. In both
classical and evolutionary game theory, strategies are
weighed in terms of maximizing the average payoff
against contestants employing both the same and
different strategies. For example, a spider may engage
in escalated fighting during a dispute only if the
disputed resource is valuable enough to warrant the
risk of physical injury. There are, however, two major
differences between the classical and evolutionary
theories. First, whereas in classical game theory it is
assumed that rational thought is used to determine
which action to take, evolutionary game theory
assumes that instinct and long-term species advantage
ultimately determine the strategies that are exhibited.
The other difference is in the payoffs: in classical
game theory, the payoffs are determined by an
individual’s personal judgment of what constitutes
winning; in evolutionary game theory, the payoffs are
defined in terms of reproductive success.
In studying populations of A. aperta in a
grassland habitat and a riparian habitat, Riechert
predicts that such factors as the size of the opponents,
the potential rate of predation in a habitat, and the
probability of winning a subsequent site if the dispute
is lost will all affect the behavior of spiders in
territorial disputes. In addition, she predicts that the
markedly different levels of competition for web sites
in the two habitats will affect the spiders’ willingness
to engage in escalated fighting. In the grassland,
where 12 percent of the habitat is available for
occupation by A. aperta, Riechert predicts that
spiders will be more willing to engage in escalated
fighting than in the riparian habitat, where 90 percent
of the habitat is suitable for occupation.

  1. Which one of the following best states the function
    of the third paragraph of the passage?
    (A) It develops a comparison of the two theories
    that were introduced in the preceding
    paragraph.
    (B) It continues a discussion of a controversial
    theory described in the first two paragraphs
    of the passage.
    (C) It describes an experiment that provides
    support for the theory described in the
    preceding paragraph.
    (D) It describes a rare phenomenon that cannot be
    accounted for by the theory described in the
    first paragraph.
    (E) It describes predictions that can be used to test
    the validity of a theory described in a
    preceding paragraph.
A

Correct Answer: E

Passage Analysis

E Structure

The third paragraph gives predictions concerning the behavior of spiders in different habitats, based on Riechert’s claim that Agelenopsis apertas’ territorial disputes are better described using evolutionary game theory.

A. No. The passage does not compare evolutionary game theory and classical game theory.

B. No. We don’t know that evolutionary game theory is controversial, and it isn’t discussed in the first paragraph.

C. No. This is close. However, it makes predictions. The experiment hasn’t been done yet.

D. No. It makes predictions. It does not describe rare phenomena.

E. Yes. It describes predictions that are based on evolutionary game theory.

How well did you know this?
1
Not at all
2
3
4
5
Perfectly
67
Q

LSAT PrepTest 52 Section #4 Question #18 Traditional theories of animal behavior assert that
animal conflict within a species is highly ritualized
and does not vary from contest to contest. This
species-specific model assumes that repetitive use of
the same visual and vocal displays and an absence of
escalated fighting evolved to prevent injury. The
contestant that exhibits the “best” display wins the
contested resource. Galápagos tortoises, for instance,
settle contests on the basis of height: the ritualized
display consists of two tortoises facing one another
and stretching their necks skyward; the tortoise
perceived as being “taller” wins.
In populations of the spider Agelenopsis aperta,
however, fighting behavior varies greatly from contest
to contest. In addition, fighting is not limited to
displays: biting and shoving are common. Susan
Riechert argues that a recently developed model,
evolutionary game theory, provides a closer fit to
A. aperta territorial disputes than does the speciesspecific model, because it explains variations in
conflict behavior that may result from varying
conditions, such as differences in size, age, and
experience of combatants. Evolutionary game theory
was adapted from the classical game theory that was
developed by von Neumann and Morganstern to
explain human behavior in conflict situations. In both
classical and evolutionary game theory, strategies are
weighed in terms of maximizing the average payoff
against contestants employing both the same and
different strategies. For example, a spider may engage
in escalated fighting during a dispute only if the
disputed resource is valuable enough to warrant the
risk of physical injury. There are, however, two major
differences between the classical and evolutionary
theories. First, whereas in classical game theory it is
assumed that rational thought is used to determine
which action to take, evolutionary game theory
assumes that instinct and long-term species advantage
ultimately determine the strategies that are exhibited.
The other difference is in the payoffs: in classical
game theory, the payoffs are determined by an
individual’s personal judgment of what constitutes
winning; in evolutionary game theory, the payoffs are
defined in terms of reproductive success.
In studying populations of A. aperta in a
grassland habitat and a riparian habitat, Riechert
predicts that such factors as the size of the opponents,
the potential rate of predation in a habitat, and the
probability of winning a subsequent site if the dispute
is lost will all affect the behavior of spiders in
territorial disputes. In addition, she predicts that the
markedly different levels of competition for web sites
in the two habitats will affect the spiders’ willingness
to engage in escalated fighting. In the grassland,
where 12 percent of the habitat is available for
occupation by A. aperta, Riechert predicts that
spiders will be more willing to engage in escalated
fighting than in the riparian habitat, where 90 percent
of the habitat is suitable for occupation.

The passage suggests which one of the following
about the behavior of A. aperta in conflict situations?
(A) They exhibit variations in fighting behavior
from contest to contest primarily because of
the different levels of competition for suitable
sites in different habitats.
(B) They may confine their fighting behavior to
displays if the value of a disputed resource is
too low and the risk of physical injury is too
great.
(C) They exhibit variations in fighting behavior
that are similar to those exhibited by
members of most other species of animals.
(D) They are more likely to engage in escalated
fighting during disputes than to limit their
fighting behavior to visual and vocal displays.
(E) They are more willing to engage in escalated
fighting during conflict situations than are
members of most other species of animals

A

Correct Answer: B

Passage Analysis

B Extract Infer

A. No. The second paragraph says that variations in conflict behavior may result from a variety of things, not primarily because of the different levels of competition in different habitats.

B. Yes. The passage states that a spider may engage in escalated fighting during a dispute only if the disputed resource is valuable enough to warrant the risk of physical injury (lines 30–33).

C. No. Agelenopsis apertas’ variations in fighting behavior are not compared to those of most other species.

D. No. The passage never says that Agelenopsis apertas are more likely to engage in escalated fighting, just that they may engage in escalated fighting.

E. No. Agelenopsis apertas’ proclivities concerning escalated fighting are not compared to those of most other species

How well did you know this?
1
Not at all
2
3
4
5
Perfectly
68
Q

LSAT PrepTest 52 Section #4 Question #19 Traditional theories of animal behavior assert that
animal conflict within a species is highly ritualized
and does not vary from contest to contest. This
species-specific model assumes that repetitive use of
the same visual and vocal displays and an absence of
escalated fighting evolved to prevent injury. The
contestant that exhibits the “best” display wins the
contested resource. Galápagos tortoises, for instance,
settle contests on the basis of height: the ritualized
display consists of two tortoises facing one another
and stretching their necks skyward; the tortoise
perceived as being “taller” wins.
In populations of the spider Agelenopsis aperta,
however, fighting behavior varies greatly from contest
to contest. In addition, fighting is not limited to
displays: biting and shoving are common. Susan
Riechert argues that a recently developed model,
evolutionary game theory, provides a closer fit to
A. aperta territorial disputes than does the speciesspecific model, because it explains variations in
conflict behavior that may result from varying
conditions, such as differences in size, age, and
experience of combatants. Evolutionary game theory
was adapted from the classical game theory that was
developed by von Neumann and Morganstern to
explain human behavior in conflict situations. In both
classical and evolutionary game theory, strategies are
weighed in terms of maximizing the average payoff
against contestants employing both the same and
different strategies. For example, a spider may engage
in escalated fighting during a dispute only if the
disputed resource is valuable enough to warrant the
risk of physical injury. There are, however, two major
differences between the classical and evolutionary
theories. First, whereas in classical game theory it is
assumed that rational thought is used to determine
which action to take, evolutionary game theory
assumes that instinct and long-term species advantage
ultimately determine the strategies that are exhibited.
The other difference is in the payoffs: in classical
game theory, the payoffs are determined by an
individual’s personal judgment of what constitutes
winning; in evolutionary game theory, the payoffs are
defined in terms of reproductive success.
In studying populations of A. aperta in a
grassland habitat and a riparian habitat, Riechert
predicts that such factors as the size of the opponents,
the potential rate of predation in a habitat, and the
probability of winning a subsequent site if the dispute
is lost will all affect the behavior of spiders in
territorial disputes. In addition, she predicts that the
markedly different levels of competition for web sites
in the two habitats will affect the spiders’ willingness
to engage in escalated fighting. In the grassland,
where 12 percent of the habitat is available for
occupation by A. aperta, Riechert predicts that
spiders will be more willing to engage in escalated
fighting than in the riparian habitat, where 90 percent
of the habitat is suitable for occupation.

The primary purpose of the passage is to
(A) present an alternative to a traditional
approach
(B) describe a phenomenon and provide specific
examples
(C) evaluate evidence used to support an
argument
(D) present data that refutes a controversial theory
(E) suggest that a new theory may be based on
inadequate research

A

Correct Answer: A

Passage Analysis

A Big Picture

A. Yes. The author presents evolutionary game theory as an alternative to the traditional theories of animal behavior.

B. No. This doesn’t address the first paragraph and its discussion of traditional theories.

C. No. The passage doesn’t evaluate the evidence either for the traditional theories or for evolutionary game theory.

D. No. This is too strong. The data on Agelenopsis apertas doesn’t refute the traditional theory. And, the only theory that might be considered controversial would be evolutionary game theory.

E. No. The passage seems to support the new theory.

How well did you know this?
1
Not at all
2
3
4
5
Perfectly
69
Q

LSAT PrepTest 52 Section #4 Question #20 Most people acknowledge that not all
governments have a moral right to govern and that
there are sometimes morally legitimate reasons for
disobeying the law, as when a particular law
prescribes behavior that is clearly immoral. It is also
commonly supposed that such cases are special
exceptions and that, in general, the fact that
something is against the law counts as a moral, as
well as legal, ground for not doing it; i.e., we
generally have a moral duty to obey a law simply
because it is the law. But the theory known as
philosophical anarchism denies this view, arguing
instead that people who live under the jurisdiction of
governments have no moral duty to those
governments to obey their laws. Some commentators
have rejected this position because of what they take
to be its highly counterintuitive implications: (1) that
no existing government is morally better than any
other (since all are, in a sense, equally illegitimate),
and (2) that, lacking any moral obligation to obey any
laws, people may do as they please without scruple.
In fact, however, philosophical anarchism does not
entail these claims.
First, the conclusion that no government is
morally better than any other does not follow from
the claim that nobody owes moral obedience to any
government. Even if one denies that there is a moral
obligation to follow the laws of any government, one
can still evaluate the morality of the policies and
actions of various governments. Some governments
do more good than harm, and others more harm than
good, to their subjects. Some violate the moral rights
of individuals more regularly, systematically, and
seriously than others. In short, it is perfectly
consistent with philosophical anarchism to hold that
governments vary widely in their moral stature.
Second, philosophical anarchists maintain that all
individuals have basic, nonlegal moral duties to one
another—duties not to harm others in their lives,
liberty, health, or goods. Even if governmental laws
have no moral force, individuals still have duties to
refrain from those actions that constitute crimes in the
majority of legal systems (such as murder, assault,
theft, and fraud). Moreover, philosophical anarchists
hold that people have a positive moral obligation to
care for one another, a moral obligation that they
might even choose to discharge by supporting
cooperative efforts by governments to help those in
need. And where others are abiding by established
laws, even those laws derived from mere conventions,
individuals are morally bound not to violate those
laws when doing so would endanger others. Thus, if
others obey the law and drive their vehicles on the
right, one must not endanger them by driving on the
left, for, even though driving on the left is not
inherently immoral, it is morally wrong to deliberately
harm the innocent.

Which one of the following most accurately expresses
the main point of the passage?
(A) Some views that certain commentators
consider to be implications of philosophical
anarchism are highly counterintuitive.
(B) Contrary to what philosophical anarchists
claim, some governments are morally
superior to others, and citizens under
legitimate governments have moral
obligations to one another.
(C) It does not follow logically from philosophical
anarchism that no government is morally
better than any other or that people have no
moral duties toward one another.
(D) Even if, as certain philosophical anarchists
claim, governmental laws lack moral force,
people still have a moral obligation to refrain
from harming one another.
(E) Contrary to what some of its opponents have
claimed, philosophical anarchism does not
conflict with the ordinary view that one
should obey the law because it is the law.

A

Correct Answer: C

Passage Analysis

C Big Picture

A. No. This is discussed only in the first paragraph.

B. No. The author argues in the second paragraph that it is consistent with philosophical anarchism to hold that some governments may be morally better than others.

C. Yes. The author presents some supposed implications of philosophical anarchism and then shows that these claims do not logically follow from philosophical anarchism.

D. No. The author speaks about philosophical anarchism in general, not about certain philosophical anarchists.

E. No. Philosophical anarchism does not hold that one should obey the law because it is the law.

How well did you know this?
1
Not at all
2
3
4
5
Perfectly
70
Q

LSAT PrepTest 52 Section #4 Question #21 Most people acknowledge that not all
governments have a moral right to govern and that
there are sometimes morally legitimate reasons for
disobeying the law, as when a particular law
prescribes behavior that is clearly immoral. It is also
commonly supposed that such cases are special
exceptions and that, in general, the fact that
something is against the law counts as a moral, as
well as legal, ground for not doing it; i.e., we
generally have a moral duty to obey a law simply
because it is the law. But the theory known as
philosophical anarchism denies this view, arguing
instead that people who live under the jurisdiction of
governments have no moral duty to those
governments to obey their laws. Some commentators
have rejected this position because of what they take
to be its highly counterintuitive implications: (1) that
no existing government is morally better than any
other (since all are, in a sense, equally illegitimate),
and (2) that, lacking any moral obligation to obey any
laws, people may do as they please without scruple.
In fact, however, philosophical anarchism does not
entail these claims.
First, the conclusion that no government is
morally better than any other does not follow from
the claim that nobody owes moral obedience to any
government. Even if one denies that there is a moral
obligation to follow the laws of any government, one
can still evaluate the morality of the policies and
actions of various governments. Some governments
do more good than harm, and others more harm than
good, to their subjects. Some violate the moral rights
of individuals more regularly, systematically, and
seriously than others. In short, it is perfectly
consistent with philosophical anarchism to hold that
governments vary widely in their moral stature.
Second, philosophical anarchists maintain that all
individuals have basic, nonlegal moral duties to one
another—duties not to harm others in their lives,
liberty, health, or goods. Even if governmental laws
have no moral force, individuals still have duties to
refrain from those actions that constitute crimes in the
majority of legal systems (such as murder, assault,
theft, and fraud). Moreover, philosophical anarchists
hold that people have a positive moral obligation to
care for one another, a moral obligation that they
might even choose to discharge by supporting
cooperative efforts by governments to help those in
need. And where others are abiding by established
laws, even those laws derived from mere conventions,
individuals are morally bound not to violate those
laws when doing so would endanger others. Thus, if
others obey the law and drive their vehicles on the
right, one must not endanger them by driving on the
left, for, even though driving on the left is not
inherently immoral, it is morally wrong to deliberately
harm the innocent.

The author identifies which one of the following as a
commonly held belief?
(A) In most cases we are morally obligated to obey
the law simply because it is the law.
(B) All governments are in essence morally equal.
(C) We are morally bound to obey only those laws
we participate in establishing.
(D) Most crimes are morally neutral, even though
they are illegal.
(E) The majority of existing laws are intended to
protect others from harm.

A

Correct Answer: A

Passage Analysis

A Extract Fact

A. Yes. The author states this in the first paragraph.

B. No. The first paragraph says that most people acknowledge that not all governments have a moral right to govern.

C. No. The first paragraph doesn’t discuss the connection between being morally bound to obey the law and participating in establishing the law.

D. No. The first paragraph says that most people believe that we generally have a moral duty to obey the law. This would make most crimes morally bad.

E. No. This was discussed in the third paragraph in reference to what philosophical anarchists believe. Also, there is no discussion about whether the majority of existing laws are to protect others.

How well did you know this?
1
Not at all
2
3
4
5
Perfectly
71
Q

LSAT PrepTest 52 Section #4 Question #22 Most people acknowledge that not all
governments have a moral right to govern and that
there are sometimes morally legitimate reasons for
disobeying the law, as when a particular law
prescribes behavior that is clearly immoral. It is also
commonly supposed that such cases are special
exceptions and that, in general, the fact that
something is against the law counts as a moral, as
well as legal, ground for not doing it; i.e., we
generally have a moral duty to obey a law simply
because it is the law. But the theory known as
philosophical anarchism denies this view, arguing
instead that people who live under the jurisdiction of
governments have no moral duty to those
governments to obey their laws. Some commentators
have rejected this position because of what they take
to be its highly counterintuitive implications: (1) that
no existing government is morally better than any
other (since all are, in a sense, equally illegitimate),
and (2) that, lacking any moral obligation to obey any
laws, people may do as they please without scruple.
In fact, however, philosophical anarchism does not
entail these claims.
First, the conclusion that no government is
morally better than any other does not follow from
the claim that nobody owes moral obedience to any
government. Even if one denies that there is a moral
obligation to follow the laws of any government, one
can still evaluate the morality of the policies and
actions of various governments. Some governments
do more good than harm, and others more harm than
good, to their subjects. Some violate the moral rights
of individuals more regularly, systematically, and
seriously than others. In short, it is perfectly
consistent with philosophical anarchism to hold that
governments vary widely in their moral stature.
Second, philosophical anarchists maintain that all
individuals have basic, nonlegal moral duties to one
another—duties not to harm others in their lives,
liberty, health, or goods. Even if governmental laws
have no moral force, individuals still have duties to
refrain from those actions that constitute crimes in the
majority of legal systems (such as murder, assault,
theft, and fraud). Moreover, philosophical anarchists
hold that people have a positive moral obligation to
care for one another, a moral obligation that they
might even choose to discharge by supporting
cooperative efforts by governments to help those in
need. And where others are abiding by established
laws, even those laws derived from mere conventions,
individuals are morally bound not to violate those
laws when doing so would endanger others. Thus, if
others obey the law and drive their vehicles on the
right, one must not endanger them by driving on the
left, for, even though driving on the left is not
inherently immoral, it is morally wrong to deliberately
harm the innocent.

  1. The author’s stance regarding the theory of
    philosophical anarchism can most accurately be
    described as one of
    (A) ardent approval of most aspects of the theory
    (B) apparent acceptance of some of the basic
    positions of the theory
    (C) concerned pessimism about the theory’s ability
    to avoid certain extreme views
    (D) hesitant rejection of some of the central
    features of the theory
    (E) resolute antipathy toward both the theory and
    certain of its logical consequences
A

Correct Answer: B

Passage Analysis

B Extract Infer

The author defends philosophical anarchists against their critics but does not explicitly endorse this position. This supports (B). Choice (A) is the next best answer but is too strong.

How well did you know this?
1
Not at all
2
3
4
5
Perfectly
72
Q

LSAT PrepTest 52 Section #4 Question #24 Most people acknowledge that not all
governments have a moral right to govern and that
there are sometimes morally legitimate reasons for
disobeying the law, as when a particular law
prescribes behavior that is clearly immoral. It is also
commonly supposed that such cases are special
exceptions and that, in general, the fact that
something is against the law counts as a moral, as
well as legal, ground for not doing it; i.e., we
generally have a moral duty to obey a law simply
because it is the law. But the theory known as
philosophical anarchism denies this view, arguing
instead that people who live under the jurisdiction of
governments have no moral duty to those
governments to obey their laws. Some commentators
have rejected this position because of what they take
to be its highly counterintuitive implications: (1) that
no existing government is morally better than any
other (since all are, in a sense, equally illegitimate),
and (2) that, lacking any moral obligation to obey any
laws, people may do as they please without scruple.
In fact, however, philosophical anarchism does not
entail these claims.
First, the conclusion that no government is
morally better than any other does not follow from
the claim that nobody owes moral obedience to any
government. Even if one denies that there is a moral
obligation to follow the laws of any government, one
can still evaluate the morality of the policies and
actions of various governments. Some governments
do more good than harm, and others more harm than
good, to their subjects. Some violate the moral rights
of individuals more regularly, systematically, and
seriously than others. In short, it is perfectly
consistent with philosophical anarchism to hold that
governments vary widely in their moral stature.
Second, philosophical anarchists maintain that all
individuals have basic, nonlegal moral duties to one
another—duties not to harm others in their lives,
liberty, health, or goods. Even if governmental laws
have no moral force, individuals still have duties to
refrain from those actions that constitute crimes in the
majority of legal systems (such as murder, assault,
theft, and fraud). Moreover, philosophical anarchists
hold that people have a positive moral obligation to
care for one another, a moral obligation that they
might even choose to discharge by supporting
cooperative efforts by governments to help those in
need. And where others are abiding by established
laws, even those laws derived from mere conventions,
individuals are morally bound not to violate those
laws when doing so would endanger others. Thus, if
others obey the law and drive their vehicles on the
right, one must not endanger them by driving on the
left, for, even though driving on the left is not
inherently immoral, it is morally wrong to deliberately
harm the innocent.

  1. Which one of the following scenarios most
    completely conforms to the views attributed to
    philosophical anarchists in lines 37–44?
    (A) A member of a political party that is illegal in
    a particular country divulges the names of
    other members because he fears legal
    penalties.
    (B) A corporate executive chooses to discontinue
    her company’s practice of dumping chemicals
    illegally when she learns that the chemicals
    are contaminating the water supply.
    (C) A person who knows that a coworker has
    stolen funds from their employer decides to
    do nothing because the coworker is widely
    admired.
    (D) A person neglects to pay her taxes, even
    though it is likely that she will suffer severe
    legal penalties as a consequence, because she
    wants to use the money to finance a new
    business.
    (E) A driver determines that it is safe to exceed the
    posted speed limit, in spite of poor visibility,
    because there are apparently no other vehicles
    on the road
A

Correct Answer: B

Passage Analysis

B Extract Infer

A. No. The last part of the analogy discusses legal penalties, which the anarchists don’t recognize as being pertinent to morality.

B. Yes. The corporate executive refrains from a practice because she finds out it might be hurting others.

C. No. The coworker has hurt someone else but the person does nothing.

D. No. This doesn’t discuss the possibility of hurting others.

E. No. This action does not discuss the reasons for refraining from an action.

How well did you know this?
1
Not at all
2
3
4
5
Perfectly
73
Q

LSAT PrepTest 52 Section #4 Question #23 Most people acknowledge that not all
governments have a moral right to govern and that
there are sometimes morally legitimate reasons for
disobeying the law, as when a particular law
prescribes behavior that is clearly immoral. It is also
commonly supposed that such cases are special
exceptions and that, in general, the fact that
something is against the law counts as a moral, as
well as legal, ground for not doing it; i.e., we
generally have a moral duty to obey a law simply
because it is the law. But the theory known as
philosophical anarchism denies this view, arguing
instead that people who live under the jurisdiction of
governments have no moral duty to those
governments to obey their laws. Some commentators
have rejected this position because of what they take
to be its highly counterintuitive implications: (1) that
no existing government is morally better than any
other (since all are, in a sense, equally illegitimate),
and (2) that, lacking any moral obligation to obey any
laws, people may do as they please without scruple.
In fact, however, philosophical anarchism does not
entail these claims.
First, the conclusion that no government is
morally better than any other does not follow from
the claim that nobody owes moral obedience to any
government. Even if one denies that there is a moral
obligation to follow the laws of any government, one
can still evaluate the morality of the policies and
actions of various governments. Some governments
do more good than harm, and others more harm than
good, to their subjects. Some violate the moral rights
of individuals more regularly, systematically, and
seriously than others. In short, it is perfectly
consistent with philosophical anarchism to hold that
governments vary widely in their moral stature.
Second, philosophical anarchists maintain that all
individuals have basic, nonlegal moral duties to one
another—duties not to harm others in their lives,
liberty, health, or goods. Even if governmental laws
have no moral force, individuals still have duties to
refrain from those actions that constitute crimes in the
majority of legal systems (such as murder, assault,
theft, and fraud). Moreover, philosophical anarchists
hold that people have a positive moral obligation to
care for one another, a moral obligation that they
might even choose to discharge by supporting
cooperative efforts by governments to help those in
need. And where others are abiding by established
laws, even those laws derived from mere conventions,
individuals are morally bound not to violate those
laws when doing so would endanger others. Thus, if
others obey the law and drive their vehicles on the
right, one must not endanger them by driving on the
left, for, even though driving on the left is not
inherently immoral, it is morally wrong to deliberately
harm the innocent.

  1. By attributing to commentators the view that
    philosophical anarchism has implications that are
    “counterintuitive” (line 17), the author most likely
    means that the commentators believe that
    (A) the implications conflict with some commonly
    held beliefs
    (B) there is little empirical evidence that the
    implications are actually true
    (C) common sense indicates that philosophical
    anarchism does not have such implications
    (D) the implications appear to be incompatible
    with each other
    (E) each of the implications contains an internal
    logical inconsistency
A

Correct Answer: A

Passage Analysis

A Structure

The author uses the word “counterintuitive” to point out that some commentators think that the implications of philosophical anarchism are “against our intuitions,” which supports (A).

How well did you know this?
1
Not at all
2
3
4
5
Perfectly
74
Q

LSAT PrepTest 52 Section #4 Question #25 Most people acknowledge that not all
governments have a moral right to govern and that
there are sometimes morally legitimate reasons for
disobeying the law, as when a particular law
prescribes behavior that is clearly immoral. It is also
commonly supposed that such cases are special
exceptions and that, in general, the fact that
something is against the law counts as a moral, as
well as legal, ground for not doing it; i.e., we
generally have a moral duty to obey a law simply
because it is the law. But the theory known as
philosophical anarchism denies this view, arguing
instead that people who live under the jurisdiction of
governments have no moral duty to those
governments to obey their laws. Some commentators
have rejected this position because of what they take
to be its highly counterintuitive implications: (1) that
no existing government is morally better than any
other (since all are, in a sense, equally illegitimate),
and (2) that, lacking any moral obligation to obey any
laws, people may do as they please without scruple.
In fact, however, philosophical anarchism does not
entail these claims.
First, the conclusion that no government is
morally better than any other does not follow from
the claim that nobody owes moral obedience to any
government. Even if one denies that there is a moral
obligation to follow the laws of any government, one
can still evaluate the morality of the policies and
actions of various governments. Some governments
do more good than harm, and others more harm than
good, to their subjects. Some violate the moral rights
of individuals more regularly, systematically, and
seriously than others. In short, it is perfectly
consistent with philosophical anarchism to hold that
governments vary widely in their moral stature.
Second, philosophical anarchists maintain that all
individuals have basic, nonlegal moral duties to one
another—duties not to harm others in their lives,
liberty, health, or goods. Even if governmental laws
have no moral force, individuals still have duties to
refrain from those actions that constitute crimes in the
majority of legal systems (such as murder, assault,
theft, and fraud). Moreover, philosophical anarchists
hold that people have a positive moral obligation to
care for one another, a moral obligation that they
might even choose to discharge by supporting
cooperative efforts by governments to help those in
need. And where others are abiding by established
laws, even those laws derived from mere conventions,
individuals are morally bound not to violate those
laws when doing so would endanger others. Thus, if
others obey the law and drive their vehicles on the
right, one must not endanger them by driving on the
left, for, even though driving on the left is not
inherently immoral, it is morally wrong to deliberately
harm the innocent.

  1. It can be inferred that the author would be most
    likely to agree that
    (A) people are subject to more moral obligations
    than is generally held to be the case
    (B) governments that are morally superior
    recognize that their citizens are not morally
    bound to obey their laws
    (C) one may have good reason to support the
    efforts of one’s government even if one has no
    moral duty to obey its laws
    (D) there are some sound arguments for claiming
    that most governments have a moral right to
    require obedience to their laws
    (E) the theory of philosophical anarchism entails
    certain fundamental principles regarding how
    laws should be enacted and enforced
A

Correct Answer: C

Passage Analysis

C Extract Infer

A. No. The author never claims that philosophical anarchism attributes more moral obligations to people than is commonly held.

B. No. The author doesn’t discuss what morally superior governments recognize.

C. Yes. The author discusses this in the third paragraph.

D. No. The author defends philosophical anarchism, which states that there is no moral obligation to obey laws, simply because they are the laws.

E. No. Philosophical anarchism doesn’t recognize laws as such.

How well did you know this?
1
Not at all
2
3
4
5
Perfectly
75
Q

LSAT PrepTest 52 Section #4 Question #26 Most people acknowledge that not all
governments have a moral right to govern and that
there are sometimes morally legitimate reasons for
disobeying the law, as when a particular law
prescribes behavior that is clearly immoral. It is also
commonly supposed that such cases are special
exceptions and that, in general, the fact that
something is against the law counts as a moral, as
well as legal, ground for not doing it; i.e., we
generally have a moral duty to obey a law simply
because it is the law. But the theory known as
philosophical anarchism denies this view, arguing
instead that people who live under the jurisdiction of
governments have no moral duty to those
governments to obey their laws. Some commentators
have rejected this position because of what they take
to be its highly counterintuitive implications: (1) that
no existing government is morally better than any
other (since all are, in a sense, equally illegitimate),
and (2) that, lacking any moral obligation to obey any
laws, people may do as they please without scruple.
In fact, however, philosophical anarchism does not
entail these claims.
First, the conclusion that no government is
morally better than any other does not follow from
the claim that nobody owes moral obedience to any
government. Even if one denies that there is a moral
obligation to follow the laws of any government, one
can still evaluate the morality of the policies and
actions of various governments. Some governments
do more good than harm, and others more harm than
good, to their subjects. Some violate the moral rights
of individuals more regularly, systematically, and
seriously than others. In short, it is perfectly
consistent with philosophical anarchism to hold that
governments vary widely in their moral stature.
Second, philosophical anarchists maintain that all
individuals have basic, nonlegal moral duties to one
another—duties not to harm others in their lives,
liberty, health, or goods. Even if governmental laws
have no moral force, individuals still have duties to
refrain from those actions that constitute crimes in the
majority of legal systems (such as murder, assault,
theft, and fraud). Moreover, philosophical anarchists
hold that people have a positive moral obligation to
care for one another, a moral obligation that they
might even choose to discharge by supporting
cooperative efforts by governments to help those in
need. And where others are abiding by established
laws, even those laws derived from mere conventions,
individuals are morally bound not to violate those
laws when doing so would endanger others. Thus, if
others obey the law and drive their vehicles on the
right, one must not endanger them by driving on the
left, for, even though driving on the left is not
inherently immoral, it is morally wrong to deliberately
harm the innocent.

  1. The author’s discussion of people’s positive moral
    duty to care for one another (lines 44–49) functions
    primarily to
    (A) demonstrate that governmental efforts to help
    those in need are superfluous
    (B) suggest that philosophical anarchists maintain
    that laws that foster the common good are
    extremely rare
    (C) imply that the theoretical underpinnings of
    philosophical anarchism are inconsistent with
    certain widely held moral truths
    (D) indicate that philosophical anarchists
    recognize that people are subject to
    substantial moral obligations
    (E) illustrate that people are morally obligated to
    refrain from those actions that arc crimes in
    most legal systems
A

Correct Answer: D

Passage Analysis

D Structure

The author discusses people’s positive moral duty to care for one another to demonstrate that philosophical anarchists think that people have a lot of moral obligations, which supports (D).

How well did you know this?
1
Not at all
2
3
4
5
Perfectly
76
Q

LSAT PrepTest 52 Section #4 Question #27 Most people acknowledge that not all
governments have a moral right to govern and that
there are sometimes morally legitimate reasons for
disobeying the law, as when a particular law
prescribes behavior that is clearly immoral. It is also
commonly supposed that such cases are special
exceptions and that, in general, the fact that
something is against the law counts as a moral, as
well as legal, ground for not doing it; i.e., we
generally have a moral duty to obey a law simply
because it is the law. But the theory known as
philosophical anarchism denies this view, arguing
instead that people who live under the jurisdiction of
governments have no moral duty to those
governments to obey their laws. Some commentators
have rejected this position because of what they take
to be its highly counterintuitive implications: (1) that
no existing government is morally better than any
other (since all are, in a sense, equally illegitimate),
and (2) that, lacking any moral obligation to obey any
laws, people may do as they please without scruple.
In fact, however, philosophical anarchism does not
entail these claims.
First, the conclusion that no government is
morally better than any other does not follow from
the claim that nobody owes moral obedience to any
government. Even if one denies that there is a moral
obligation to follow the laws of any government, one
can still evaluate the morality of the policies and
actions of various governments. Some governments
do more good than harm, and others more harm than
good, to their subjects. Some violate the moral rights
of individuals more regularly, systematically, and
seriously than others. In short, it is perfectly
consistent with philosophical anarchism to hold that
governments vary widely in their moral stature.
Second, philosophical anarchists maintain that all
individuals have basic, nonlegal moral duties to one
another—duties not to harm others in their lives,
liberty, health, or goods. Even if governmental laws
have no moral force, individuals still have duties to
refrain from those actions that constitute crimes in the
majority of legal systems (such as murder, assault,
theft, and fraud). Moreover, philosophical anarchists
hold that people have a positive moral obligation to
care for one another, a moral obligation that they
might even choose to discharge by supporting
cooperative efforts by governments to help those in
need. And where others are abiding by established
laws, even those laws derived from mere conventions,
individuals are morally bound not to violate those
laws when doing so would endanger others. Thus, if
others obey the law and drive their vehicles on the
right, one must not endanger them by driving on the
left, for, even though driving on the left is not
inherently immoral, it is morally wrong to deliberately
harm the innocent.

  1. In the passage, the author seeks primarily to
    (A) describe the development and theoretical
    underpinnings of a particular theory
    (B) establish that a particular theory conforms to
    the dictates of common sense
    (C) argue that two necessary implications of a
    particular theory are morally acceptable
    (D) defend a particular theory against its critics by
    showing that their arguments are mistaken
    (E) demonstrate that proponents of a particular
    theory are aware of the theory’s defects
A

Correct Answer: D

Passage Analysis

D Big Picture

A. No. The author doesn’t describe the development of philosophical anarchism.

B. No. This is close. However, this doesn’t talk about the claims of the critics of philosophical anarchism.

C. No. The author tries to show that these supposed implications are not necessary implications of philosophical anarchism.

D. Yes. The author tries to show that the critics are wrong in thinking that philosophical anarchism has, as its implications, two counterintuitive claims.

E. No. The author argues that the supposedly counterintuitive implications are not actual implications of philosophical anarchism and, therefore, are not defects of philosophical anarchism.

How well did you know this?
1
Not at all
2
3
4
5
Perfectly
77
Q
  1. Consumer advocate: Businesses are typically
    motivated primarily by the desire to make as
    great a profit as possible, and advertising helps
    businesses to achieve this goal. But it is clear
    that the motive of maximizing profits does not
    impel businesses to present accurate
    information in their advertisements. It follows
    that consumers should be skeptical of the
    claims made in advertisements.
    Each of the following, if true, would strengthen the
    consumer advocate’s argument EXCEPT:
    (A) Businesses know that they can usually
    maximize their profits by using inaccurate
    information in their advertisements.
    (B) Businesses have often included inaccurate
    information in their advertisements.
    (C) Many consumers have a cynical attitude
    toward advertising.
    (D) Those who create advertisements are less
    concerned with the accuracy than with the
    creativity of advertisements.
    (E) The laws regulating truth in advertising are
    not applicable to many of the most common
    forms of inaccurate advertising
A

Correct Answer: C

C Strengthen

The consumer advocate concludes that consumers ought to be skeptical of the claims made in advertisements. The evidence is that, typically, businesses are chiefly motivated by profits, and this motive does not make businesses think that they should present accurate information in their advertisements. The consumer advocate is neglecting to consider other reasons that businesses might not make misleading claims in advertisements, despite their interests in making a profit.

A. No. This strengthens the connection between maximizing profits and using inaccurate information.

B. No. This indicates a past history of businesses making inaccurate statements, thus strengthening the conclusion.

C. Yes. The conclusion claims that consumers OUGHT to be skeptical. It doesn’t address whether they actually ARE skeptical or not.

D. No. This strengthens the claim that advertisements are likely to contain inaccurate information.

E. No. This eliminates another reason as to why businesses might give accurate information, even though they want to increase profits.

How well did you know this?
1
Not at all
2
3
4
5
Perfectly
78
Q

Elaine: The purpose of art museums is to preserve
artworks and make them available to the
public. Museums, therefore, should seek to
acquire and display the best examples of
artworks from each artistic period and genre,
even if some of these works are not recognized
by experts as masterpieces.
Frederick: Art museums ought to devote their limited
resources to acquiring the works of recognized
masters in order to ensure the preservation of
the greatest artworks.
Elaine’s and Frederick’s statements provide the most
support for the claim that they would disagree about
whether
(A) many artistic masterpieces are not recognized
as such by art experts
(B) museums should seek to represent all genres of
art in their collections
(C) art museums should seek to preserve works of art
(D) an art museum ought to acquire an unusual
example of a period or genre if more
characteristic examples are prohibitively
expensive
(E) all of the artworks that experts identify as
masterpieces are actually masterpieces

A

Correct Answer: B

B Point at Issue

Elaine concludes that museums ought to seek to acquire the best examples of artworks from each period and genre, even if some of the works are not recognized as masterpieces. Her premise is that the purpose of museums is to preserve artworks and make them available to all. Frederick disagrees, claiming that art museums ought to acquire the works of recognized masters, as museums have limited resources and a museum’s purpose is to ensure the preservation of the greatest artworks.

A. No. Frederick does not make any connection between greatest artworks and who deems them as such.

B. Yes. Elaine claims that they should, while Frederick thinks that museums have a different purpose.

C. No. Both Elaine and Frederick agree on this.

D. No. Neither is concerned with the expense of a single piece of art.

E. No. Neither questions the status of artwork identified as masterpieces.

How well did you know this?
1
Not at all
2
3
4
5
Perfectly
79
Q
  1. Science columnist: It is clear why humans have so
    many diseases in common with cats. Many
    human diseases are genetically based, and cats
    are genetically closer to humans than are any
    other mammals except nonhuman primates.
    Each of the genes identified so far in cats has
    an exact counterpart in humans.
    Which one of the following, if true, most weakens the
    science columnist’s explanation for the claim that
    humans have so many diseases in common with cats?
    (A) Cats have built up resistance to many of the
    diseases they have in common with humans.
    (B) Most diseases that humans have in common
    with cats have no genetic basis.
    (C) Cats have more diseases in common with
    nonhuman primates than with humans.
    (D) Many of the diseases humans have in common
    with cats are mild and are rarely diagnosed.
    (E) Humans have more genes in common with
    nonhuman primates than with cats.
A

Correct Answer: B

B Weaken

The science columnist presents the following evidence: Many human diseases are genetically based, and cats are genetically closer to humans than are any other mammals save nonhuman primates. The columnist claims that this evidence makes is clear why humans and cats have so many diseases in common.

A. No. The argument doesn’t make claims about how many cats (as opposed to humans) actually get the diseases.

B. Yes. The argument claims only that many human diseases are genetically based, not all. And it doesn’t tell us whether the diseases we share with cats are among those. If they are not, the fact that cats are genetically close to humans still doesn’t explain why cats and humans share a lot of diseases.

C. No. The argument doesn’t address the diseases cats and nonhuman primates have in common.

D. No. The argument doesn’t address the severity of the diseases shared by humans and cats.

E. No. The argument already states this.

How well did you know this?
1
Not at all
2
3
4
5
Perfectly
80
Q

This region must find new ways to help business
grow. After all, shoe manufacturing used to be a
major local industry, but recently has experienced
severe setbacks due to overseas competition, so there
is a need for expansion into new manufacturing
areas. Moreover, our outdated public policy generally
inhibits business growth.
Which one of the following most accurately expresses
the main conclusion drawn in the argument?
(A) The region needs to find new ways to enhance
business growth.
(B) Shoe manufacturing is no longer a major
source of income in the region.
(C) Shoe manufacturing in the region has
dramatically declined due to overseas
competition.
(D) Business in the region must expand into new
areas of manufacturing.
(E) Outdated public policy inhibits business
growth in the region.

A

Correct Answer: A

A Main Point

Because shoe manufacturing, which used to be a major local industry, has experienced severe setbacks and because outdated public policy generally prevents business growth, the argument concludes that this region must find new ways to help business grow.

A. Yes. This is a restatement of the conclusion.

B. No. We don’t know whether it is still a major source of income or not.

C. No. This is a premise.

D. No. This is a premise.

E. No. This is a premise.

How well did you know this?
1
Not at all
2
3
4
5
Perfectly
81
Q

As a result of modern medicine, more people have
been able to enjoy long and pain-free lives. But the
resulting increase in life expectancy has contributed
to a steady increase in the proportion of the
population that is of advanced age. This population
shift is creating potentially devastating financial
problems for some social welfare programs.
Which one of the following propositions is most
precisely exemplified by the situation presented above?
(A) Technical or scientific innovation cannot be
the solution to all problems.
(B) Implementing technological innovations
should be delayed until the resulting social
changes can be managed.
(C) Every enhancement of the quality of life has
unavoidable negative consequences.
(D) All social institutions are affected by a
preoccupation with prolonging life.
(E) Solving one set of problems can create a
different set of problems.

A

Correct Answer: E

E Principle Match

Modern medicine has enabled more people to live longer and pain-free lives. However, the benefits created by modern medicine have resulted in more and more of the population being older, which gives rise to financial problems for some social welfare programs.

A. No. This is too strong. The passage doesn’t discuss all problems.

B. No. The passage does not make a recommendation as to what should be done. It describes a situation.

C. No. This is too strong. The passage doesn’t discuss every enhancement of the quality of life.

D. No. This is too strong. The passage doesn’t discuss all social institutions, just some.

E. Yes. Modern medicine solved the short and painful life-span problem, but in the process it created a financial problem.

How well did you know this?
1
Not at all
2
3
4
5
Perfectly
82
Q

Since Jackie is such a big fan of Moral Vacuum’s
music, she will probably like The Cruel Herd’s new
album. Like Moral Vacuum, The Cruel Herd on this
album plays complex rock music that employs the
acoustic instrumentation and harmonic
sophistication of early sixties jazz. The Cruel Herd
also has very witty lyrics, full of puns and sardonic
humor, like some of Moral Vacuum’s best lyrics.
Which one of the following, if true, most strengthens
the argument?
(A) Jackie has not previously cared for The Cruel
Herd, but on the new album The Cruel Herd’s
previous musical arranger has been replaced
by Moral Vacuum’s musical arranger.
(B) Though The Cruel Herd’s previous albums’
production quality was not great, the new
album is produced by one of the most widely
employed producers in the music industry.
(C) Like Moral Vacuum, The Cruel Herd regularly
performs in clubs popular with many
students at the university that Jackie attends.
(D) All of the music that Jackie prefers to listen to
on a regular basis is rock music.
(E) Jackie’s favorite Moral Vacuum songs have
lyrics that are somber and marked by a strong
political awareness.

A

Correct Answer: A

A Strengthen

The argument concludes that Jackie will probably like The Cruel Herd’s new album. Jackie is a fan of Moral Vacuum’s music and, on this album, The Cruel Herd plays a type of music similar to Moral Vacuum’s. Also, the witty lyrics are similar to those of some of Moral Vacuum’s best. This is an analogy and the answer choice should make the two bands’ music even more similar.

A. Yes. If The Cruel Herd’s new musical arranger is Moral Vacuum’s musical arranger, the two bands’ music should be even more similar.

B. No. This doesn’t make The Cruel Herd sound more like Moral Vacuum.

C. No. This is too general. We don’t know that these clubs are popular with Jackie.

D. No. This doesn’t make The Cruel Herd sound more like Moral Vacuum.

E. No. This weakens the argument. If Jackie likes the somber and political Moral Vacuum lyrics, then she might not like the witty lyrics of The Cruel Herd

How well did you know this?
1
Not at all
2
3
4
5
Perfectly
83
Q

Doctor: In three separate studies, researchers
compared children who had slept with nightlights in their rooms as infants to children who
had not. In the first study, the children who
had slept with night-lights proved more likely
to be nearsighted, but the later studies found
no correlation between night-lights and
nearsightedness. However, the children in the
first study were younger than those in the later
studies. This suggests that if night-lights cause
nearsightedness, the effect disappears with age.
Which one of the following, if true, would most
weaken the doctor’s argument?
(A) A fourth study comparing infants who were
currently sleeping with night-lights to infants
who were not did not find any correlation
between night-lights and nearsightedness.
(B) On average, young children who are already
very nearsighted are no more likely to sleep
with night-lights than young children who are
not already nearsighted.
(C) In a study involving children who had not
slept with night-lights as infants but had slept
with night-lights when they were older, most
of the children studied were not nearsighted.
(D) The two studies in which no correlation was
found did not examine enough children to
provide significant support for any
conclusion regarding a causal relationship
between night-lights and nearsightedness.
(E) In a fourth study involving 100 children who
were older than those in any of the first three
studies, several of the children who had slept
with night-lights as infants were nearsighted.

A

Correct Answer: D

D Weaken

The doctor concludes that the evidence suggests that, if it’s true that night-lights cause nearsightedness, the effect disappears with age. He cites as evidence the results of three separate studies concerning children who had or had not slept with night-lights as infants. The first study involved children who were younger than those in the other studies, and it was only in the first study that a correlation between sleeping with a night-light and nearsightedness was observed. The doctor’s language is very tentative, even in his conclusion, so the weakness of the argument must lie in the studies themselves.

A. No. The doctor doesn’t claim that sleeping with a night-light definitely causes nearsightedness.

B. No. The doctor doesn’t claim that sleeping with a night-light definitely causes nearsightedness, so this is irrelevant.

C. No. The evidence and conclusion concern the possible correlation between infants sleeping with night-lights and nearsightedness, not older children.

D. Yes. This answer choice attacks the studies themselves. If the sample size is not large enough, the studies can’t give enough support to the doctor’s claim that, if there is a causal relationship, the effect disappears with time.

E. No. There needs to be more than a few children who are still nearsighted to show anything definitively. These few might be nearsighted for other reasons.

How well did you know this?
1
Not at all
2
3
4
5
Perfectly
84
Q
  1. Superconductors are substances that conduct electricity
    without resistance at low temperatures.Their use,
    however,will never be economically feasible, unless there
    is a substance that superconducts at a temperature above
    minus 148 degrees Celsius.If there is such a substance,
    that substance must be an alloy of niobium and
    germanium. Unfortunately,such alloys superconduct at
    temperatures no higher than minus 160 degrees Celsius.
    If the statements above are true, which one of the
    following must also be true?
    (A) The use of superconductors will never be
    economically feasible.
    (B) If the alloys of niobium and germanium do
    not superconduct at temperatures above
    minus 148 degrees Celsius, then there are
    other substances that will do so.
    (C) The use of superconductors could be
    economically feasible if there is a substance
    that superconducts at temperatures below
    minus 148 degrees Celsius.
    (D) Alloys of niobium and germanium do not
    superconduct at temperatures below minus
    160 degrees Celsius.
    (E) No use of alloys of niobium and germanium
    will ever be economically feasible.
A

Correct Answer: A

A Inference

Connect the facts. Economically feasible → superconducts above –148°C; ~superconduct above –148°C → ~economically feasible. Superconducts above –148°C → alloy of niobium and germanium; ~ alloy of niobium and germanium → ~superconduct above –148°C. Alloys of niobium and germanium superconduct at –160°C or lower. Therefore, ~superconduct above –148°C → ~economically feasible.

A. Yes. Given the information above, this must be true.

B. No. According to the passage, if anything will superconduct above –148°C, it would have to be an alloy of niobium and germanium.

C. No. We know that, if they are going to be economically feasible at all, they must superconduct above –148°C.

D. No. We don’t know this.

E. No. This is too strong. Their use might be economically feasible in other areas besides superconducting.

How well did you know this?
1
Not at all
2
3
4
5
Perfectly
85
Q

Public health experts have waged a long-standing
educational campaign to get people to eat more
vegetables, which are known to help prevent cancer.
Unfortunately, the campaign has had little impact on
people’s diets. The reason is probably that many
people simply dislike the taste of most vegetables.
Thus, the campaign would probably be more
effective if it included information on ways to make
vegetables more appetizing.
Which one of the following, if true, most strengthens
the argument?
(A) The campaign to get people to eat more
vegetables has had little impact on the diets of
most people who love the taste of vegetables.
(B) Some ways of making vegetables more
appetizing diminish vegetables’ ability to help
prevent cancer.
(C) People who find a few vegetables appetizing
typically do not eat substantially more
vegetables than do people who dislike the
taste of most vegetables.
(D) People who dislike the taste of most vegetables
would eat many more vegetables if they knew
how to make them more appetizing.
(E) The only way to make the campaign to get
people to eat more vegetables more effective
would be to ensure that anyone who at
present dislikes the taste of certain vegetables
learns to find those vegetables appetizing

A

Correct Answer: D

D Strengthen

There has been a long-standing campaign to get people to eat more vegetables. However, this campaign has had little impact on what people eat. The offered solution to make the campaign more effective is to include information on ways to make vegetables more appetizing, as the argument claims that the probable reason for the campaign’s ineffectiveness is that many people dislike the taste of most vegetables. The answer will strengthen the connection between the campaign’s ineffectiveness and people’s dislike of most vegetables as they now prepare them.

A. No. This weakens the argument. If it doesn’t make people who love vegetables eat more of them, then giving people information about how to make veggies tastier won’t make people eat more of them.

B. No. This weakens the argument. The campaign urges people to eat more vegetables in order to help prevent cancer. If eating the vegetables once they have been made more appetizing is less likely to prevent cancer, then the campaign will fail in its ultimate goal.

C. No. This weakens the argument.

D. Yes. If knowing how to make the vegetables more appetizing will cause people to eat more vegetables, then the campaign would likely be more effective if it gave out that information.

E. No. The campaign wants people to eat vegetables in general, not specific kinds of vegetables. If the people who disliked broccoli started to eat more Brussels sprouts and green beans, the campaign would still be more effective than it is now, even if these people never learned to like broccoli.

How well did you know this?
1
Not at all
2
3
4
5
Perfectly
86
Q

. Global surveys estimate the earth’s population of
nesting female leatherback turtles has fallen by more
than two-thirds in the past 15 years. Any species
whose population declines by more than two-thirds
in 15 years is in grave danger of extinction, so the
leatherback turtle is clearly in danger of extinction.
Which one of the following is an assumption that the
argument requires?
(A) The decline in the population of nesting
female leatherback turtles is proportional to
the decline in the leatherback turtle
population as a whole.
(B) If the global population of leatherback turtles
falls by more than two-thirds over the next 15
years, the species will eventually become
extinct.
(C) The global population of leatherback turtles
consists in roughly equal numbers of females
and males.
(D) Very few leatherback turtles exist in captivity.
(E) The only way to ensure the continued survival
of leatherback turtles in the wild is to breed
them in captivity

A

Correct Answer: A

A Necessary Assumption

The argument concludes that the leatherback turtle is in danger of extinction, based on evidence about nesting female leatherback turtles. This population of turtles has fallen by more than two-thirds in the past 15 years. Any species whose population declines by that amount in that amount of time is in grave danger of extinction. There needs to be a connection between the statistics for nesting female leatherback turtles and the leatherback turtle population as a whole.

A. Yes. This connects the decline of nesting female leatherback turtles to a similar decline in the leatherback turtle population as a whole.

B. No. This is too strong. The argument never claims that the turtles will actually become extinct.

C. No. This is too general. The argument needs a connection between the numbers of nesting female leatherbacks and the leatherback population as a whole. This compares the numbers of females in general with the number of males.

D. No. The argument doesn’t address turtles in captivity.

E. No. The argument doesn’t attempt to solve the problem; it just points out the problem.

How well did you know this?
1
Not at all
2
3
4
5
Perfectly
87
Q

Pure science—research with no immediate
commercial or technological application—is a public
good. Such research requires a great amount of
financial support and does not yield profits in the
short term. Since private corporations will not
undertake to support activities that do not yield
short-term profits, a society that wants to reap the
benefits of pure science ought to use public funds to
support such research.
The claim about private corporations serves which
one of the following functions in the argument?
(A) It expresses the conclusion of the argument.
(B) It explains what is meant by the expression
“pure research” in the context of the
argument.
(C) It distracts attention from the point at issue by
introducing a different but related goal.
(D) It supports the conclusion by ruling out an
alternative way of achieving the benefits
mentioned.
(E) It illustrates a case where unfortunate
consequences result from a failure to accept
the recommendation offered.

A

Correct Answer: D

D Reasoning

The argument concludes that a society that wants to reap the benefits of pure science ought to use public funds to support such research. The argument defines pure science as research with no immediate commercial or technological application and claims that it is a public good. Because of its nature, pure science needs a lot of monetary support and doesn’t make profits in the short term. The argument then eliminates another possible funding avenue for pure science by claiming that private corporations will not fund activities that do not yield short-term profits.

A. No. The claim about private corporations is not the conclusion.

B. No. The claim about private corporations does not help define “pure research.”

C. No. The claim about private corporations does not address a different goal.

D. Yes. This claim eliminates another possible funding source for pure science, which benefits the public.

E. No. This claim is not an example, so it doesn’t illustrate a case.

How well did you know this?
1
Not at all
2
3
4
5
Perfectly
87
Q
  1. Some doctors believe that a certain drug reduces the
    duration of episodes of vertigo, claiming that the
    average duration of vertigo for people who suffer
    from it has decreased since the drug was introduced.
    However, during a recent three-month shortage of
    the drug, there was no significant change in the
    average duration of vertigo. Thus, we can conclude
    that the drug has no effect on the duration of vertigo.
    Which one of the following is an assumption
    required by the argument?
    (A) If a drug made a difference in the duration of
    vertigo, a three-month shortage of that drug
    would have caused a significant change in the
    average duration of vertigo.
    (B) If there were any change in the average
    duration of vertigo since the introduction of
    the drug, it would have demonstrated that the
    drug has an effect on the duration of vertigo.
    (C) A period of time greater than three months
    would not have been better to use in judging
    whether the drug has an effect on the
    duration of vertigo.
    (D) Changes in diet and smoking habits are not
    responsible for any change in the average
    duration of vertigo since the introduction of
    the drug.
    (E) There are various significant factors other than
    drugs that decrease the duration of vertigo
    for many people who suffer from it.
A

Correct Answer: A

A Necessary Assumption

The author disagrees with the doctors who believe that a specific drug reduces the duration of episodes of vertigo, concluding that the drug has no effect on the duration of vertigo. The author cites the three-month shortage of the drug, during which there was no significant change in the average duration of vertigo, as evidence. The answer choice will indicate the drug’s effects would have worn off within three months.

A. Yes. This states that the reduction of duration of vertigo supposedly brought about by the drug would have been at least somewhat reversed in three month’s time.

B. No. Some doctors claim that there has been a reduction in the duration of vertigo. This weakens the argument.

C. No. This is close but it doesn’t go far enough. If no amount of time would have been good to use in judging whether the drug has an effect on the duration of vertigo, then three months would have been just as ineffective as five years to use in judging this.

D. No. This doesn’t address the time period wherein some people had to stop taking the drug.

E. No. This weakens the argument, if anything. If there were other significant factors that decrease the duration of vertigo, then stopping the drug for three months wouldn’t necessarily cause an increase in the duration of one’s vertigo.

How well did you know this?
1
Not at all
2
3
4
5
Perfectly
88
Q

Melinda: Hazard insurance decreases an individual’s
risk by judiciously spreading the risk among
many policyholders.
Jack: I disagree. It makes sense for me to buy fire
insurance for my house, but I don’t see how
doing so lessens the chances that my house will
burn down.
Jack’s response most clearly trades on an ambiguity
in which one of the following expressions used by
Melinda?
(A) judiciously spreading
(B) many policyholders
(C) risk
(D) decreases
(E) hazard insurance

A

Correct Answer: C

C Flaw

Melinda concludes that hazard insurance decreases an individual’s risk. Her evidence is that the risk is judiciously spread among many policyholders. Jack disagrees with Melinda’s conclusion. Jack concludes that hazard insurance makes sense but that having, say, fire insurance doesn’t seem to decrease the risk of his house burning down. Melinda and Jack are talking about different kinds of risk. Melinda is referring to monetary risk and Jack is referring to the risk of some specific event occurring.

A. No. Jack doesn’t address this part of Melinda’s argument.

B. No. Jack doesn’t address other policyholders. He’s just interested in one policyholder: himself.

C. Yes. Jack claims that the risk of his own house burning down won’t decrease if he has insurance, while Melinda speaks of the monetary risk that such events tend to bring about.

D. No. Both use this term in the same way.

E. No. Both use this expression in the same way.

How well did you know this?
1
Not at all
2
3
4
5
Perfectly
89
Q
  1. It has been suggested that a television set should be
    thought of as nothing more than “a toaster with
    pictures” and that since we let market forces
    determine the design of kitchen appliances we can let
    them determine what is seen on television. But that
    approach is too simple. Some governmental control
    is needed, since television is so important politically
    and culturally. It is a major source of commercial
    entertainment. It plays an important political role
    because it is the primary medium through which
    many voters obtain information about current
    affairs. It is a significant cultural force in that in the
    average home it is on for more than five hours a day.
    Which one of the following most accurately expresses
    the role played in the argument by the claim that
    television is so important politically and culturally?
    (A) It states a view that the argument as a whole is
    designed to discredit.
    (B) It is an intermediate conclusion that is offered
    in support of the claim that a television set
    should be thought of as nothing more than “a
    toaster with pictures” and for which the claim
    that we can let market forces determine what
    is seen on television is offered as support.
    (C) It is a premise that is offered in support of the
    claim that we let market forces determine the
    design of kitchen appliances.
    (D) It is an intermediate conclusion that is offered
    in support of the claim that some
    governmental control of television is needed
    and for which the claim that the television is
    on for more than five hours a day in the
    average home is offered as partial support.
    (E) It is a premise that is offered in support of the
    claim that television is the primary medium
    through which many voters obtain
    information about current affairs.
A

Correct Answer: D

D Reasoning

Some people make an analogy between a television and other kitchen appliances. These people suggest that, since we let market forces determine the design of kitchen appliances, we can let market forces determine what appears on television. The argument disagrees with this conclusion, claiming that this view is too simple, and that some government control is needed. It is too simple because television is a major source of political information and it is also a significant cultural force, as it is on for more than five hours a day in most households.

A. No. This claim supports the argument’s conclusion.

B. No. This claim is meant to discredit the claim that a television should be thought of as a “toaster with pictures.”

C. No. It does not support the claim about kitchen appliances.

D. Yes. This claim helps support the argument’s conclusion against those who think there shouldn’t be any governmental control. The claim that the television is on for more than five hours a day in the average home is meant to show that the television is so culturally important.

E. No. The claim that television is the primary medium through which many voters obtain information about current affairs partially supports the claim that television is so politically and culturally important, not the other way around.

How well did you know this?
1
Not at all
2
3
4
5
Perfectly
90
Q

. Earthworms, vital to the health of soil, prefer soil that
is approximately neutral on the acid-to-alkaline scale.
Since decomposition of dead plants makes the top layer
of soil highly acidic, application of crushed limestone,
which is highly alkaline, to the soil’s surface should
make the soil more attractive to earthworms.
Which one of the following is an assumption on
which the argument depends?
(A) As far as soil health is concerned, aiding the
decomposition of dead plants is the most
important function performed by earthworms.
(B) After its application to the soil’s surface,
crushed limestone stays in the soil’s top layer
long enough to neutralize some of the top
layer’s acidity.
(C) Crushed limestone contains available calcium
and magnesium, both of which are just as
vital as earthworms to healthy soil.
(D) By itself, acidity of soil does nothing to hasten
decomposition of dead plants.
(E) Alkaline soil is significantly more likely to
benefit from an increased earthworm
population than is highly acidic soil.

A

Correct Answer: B

B Necessary Assumption

The argument concludes that application of highly alkaline crushed limestone to the soil’s surface should make the soil more attractive to earthworms. Decomposition of dead plants makes the top layer of soil highly acidic and earthworms, which are vital to soil’s heath, prefer soil that is approximately neutral on the acid-to-alkaline scale.

A. No. This is too strong. Aiding the decomposition of dead plants doesn’t have to be the most important function performed by earthworms in order for the conclusion to follow.

B. Yes. If the limestone immediately washed off, it wouldn’t have a chance to neutralize the top layer’s acidity and so limestone application wouldn’t make the soil’s surface more attractive to earthworms.

C. No. The argument is focused on the interaction between earthworms and the soil.

D. No. The argument is focused on the interaction between earthworms and the soil.

E. No. The argument is focused on a method to make the soil more neutral, in order to make it more attractive to earthworms. It doesn’t discuss which type of non-neutral soil is more likely to benefit from earthworms.

How well did you know this?
1
Not at all
2
3
4
5
Perfectly
91
Q

Salesperson: When a salesperson is successful, it is
certain that that person has been in sales for at
least three years. This is because to succeed as
a salesperson, one must first establish a strong
client base, and studies have shown that
anyone who spends at least three years
developing a client base can eventually make a
comfortable living in sales.
The reasoning in the salesperson’s argument is
vulnerable to criticism on the grounds that it fails to
consider the possibility that
(A) salespeople who have spent three years
developing a client base might not yet be
successful in sales
(B) some salespeople require fewer than three
years in which to develop a strong client base
(C) a salesperson who has not spent three years
developing a client base may not succeed in
sales
(D) it takes longer than three years for a
salesperson to develop a strong client base
(E) few salespeople can afford to spend three years
building a client base

A

Correct Answer: B

B Flaw

This is diagrammable. Conclusion: successful salesperson → been in sales for at least three years. Evidence: successful salesperson → establish a strong client base; at least three years developing a client base → eventually make a comfortable living in sales. The argument is confusing something that is sufficient for being a successful salesperson with something that is necessary for being a successful salesperson.

A. No. The evidence states that they will eventually be successful in sales.

B. Yes. The argument claims that successful salespeople must have spent at least three years in sales, while the evidence states that, if salespeople spend at least three years in sales, they will eventually make a comfortable living in sales. It is confusing something that is sufficient for being successful in sales with something that is necessary for being successful in sales.

C. No. This is the contrapositive of the conclusion.

D. No. The argument claims that salespeople need to spend at least three years in sales, so it allows for the fact that it might take longer than three years to develop a strong client base.

E. No. The argument doesn’t make any claims as to how many salespeople are able to do this.

How well did you know this?
1
Not at all
2
3
4
5
Perfectly
91
Q

Jurist: A nation’s laws must be viewed as expressions
of a moral code that transcends those laws and
serves as a measure of their adequacy.
Otherwise, a society can have no sound basis
for preferring any given set of laws to all
others. Thus, any moral prohibition against
the violation of statutes must leave room for
exceptions.
Which one of the following can be properly inferred
from the jurist’s statements?
(A) Those who formulate statutes are not primarily
concerned with morality when they do so.
(B) Sometimes criteria other than the criteria
derived from a moral code should be used in
choosing one set of laws over another.
(C) Unless it is legally forbidden ever to violate
some moral rules, moral behavior and
compliance with laws are indistinguishable.
(D) There is no statute that a nation’s citizens have
a moral obligation to obey.
(E) A nation’s laws can sometimes come into
conflict with the moral code they express.

A

Correct Answer: E

E Inference

Pick the answer best supported by the passage.

A. No. The passage doesn’t mention the motivations of the statute-makers.

B. No. The passage claims that, in order to have a sound basis for preferring a given set of laws to any others, laws must be viewed as expressions of a transcendental moral code.

C. No. The passage suggests that the moral rules have the preferred status, not the laws. Also, the last sentence suggests that moral behavior and compliance with laws are at least sometimes distinguishable.

D. No. This is too strong. The passage doesn’t say that there is no stature that the citizens have a moral obligation to obey.

E. Yes. If the laws are to be seen as expression of a moral code that has precedence over these laws, and that measures the adequacy of these laws, then there shouldn’t be an absolute moral prohibition against the violation of statutes. What if, for example, there was a statute that wasn’t in accord with the moral code?

How well did you know this?
1
Not at all
2
3
4
5
Perfectly
92
Q
  1. An association between two types of conditions does
    not establish that conditions of one type cause
    conditions of the other type. Even persistent and
    inviolable association is inconclusive; such
    association is often due to conditions of both types
    being effects of the same kind of cause.
    Which one of the following judgments most closely
    conforms to the principle stated above?
    (A) Some people claim that rapid growth of the
    money supply is what causes inflation. But
    this is a naive view. What these people do
    not realize is that growth in the money supply
    and inflation are actually one and the same
    phenomenon.
    (B) People who have high blood pressure tend to
    be overweight. But before we draw any
    inferences, we should consider that an
    unhealthy lifestyle can cause high blood
    pressure, and weight gain can result from
    living unhealthily.
    (C) In some areas, there is a high correlation
    between ice cream consumption and the
    crime rate. Some researchers have proposed
    related third factors, but we cannot rule out
    that the correlation is purely coincidental.
    (D) People’s moods seem to vary with the color of
    the clothes they wear. Dark colors are
    associated with gloomy moods, and bright
    colors are associated with cheerful moods.
    This correlation resolves nothing, however.
    We cannot say whether it is the colors that
    cause the moods or the converse.
    (E) Linguists propose that the similarities between
    Greek and Latin are due to their common
    descent from an earlier language. But how are
    we to know that the similarities are not
    actually due to the two languages having
    borrowed structures from one another, as
    with the languages Marathi and Telegu?
A

Correct Answer: B

B Principle Match

The principle concerns correlation versus causation: that is, that persistent correlation does not conclusively prove a causal relationship because this correlation is often due to a common cause. The answer choice will be an example of this.

A. No. Supply and inflation are the same phenomenon, which is different from them having a common cause.

B. Yes. The unhealthy lifestyle is likely the common cause for both high blood pressure and being overweight, so we shouldn’t necessarily think that being overweight causes high blood pressure. This is an example of the principle in the passage.

C. No. This doesn’t propose a common cause for ice cream consumption and high crime rates.

D. No. This doesn’t propose a common cause for mood and colors worn.

E. No. This questions a proposed common cause, claiming that the two languages borrowed from each other instead.

How well did you know this?
1
Not at all
2
3
4
5
Perfectly
93
Q
  1. People who have habitually slept less than six hours a
    night and then begin sleeping eight or more hours a
    night typically begin to feel much less anxious.
    Therefore, most people who sleep less than six hours
    a night can probably cause their anxiety levels to fall
    by beginning to sleep at least eight hours a night.
    The reasoning in which one of the following arguments
    is most similar to that in the argument above?
    (A) When a small company first begins to
    advertise on the Internet, its financial
    situation generally improves. This shows that
    most small companies that have never
    advertised on the Internet can probably
    improve their financial situation by doing so.
    (B) Certain small companies that had never
    previously advertised on the Internet have
    found that their financial situations began to
    improve after they started to do so. So most
    small companies can probably improve their
    financial situations by starting to advertise on
    the Internet.
    (C) It must be true that any small company that
    increases its Internet advertising will improve
    its financial situation, since most small
    companies that advertise on the Internet
    improved their financial situations soon after
    they first began to do so.
    (D) Usually, the financial situation of a small
    company that has never advertised on the
    Internet will improve only if that company
    starts to advertise on the Internet. Therefore,
    a typical small company that has never
    advertised on the Internet can probably
    improve its financial situation by doing so.
    (E) A small company’s financial situation usually
    improves soon after that company first
    begins to advertise on the Internet. Thus,
    most small companies that have never
    advertised on the Internet could probably
    become financially strong.
A

Correct Answer: A

A Parallel

The argument concludes that most people who sleep less than six hours a night can probably cause their anxiety levels to fall by beginning to sleep at least eight hours a night. The evidence is a correlation between a drop in anxiety levels of people who have habitually slept less than six hours a night but who start sleeping eight or more hours a night.

A. Yes. This concludes that most small companies that have never advertised on the Internet can probably improve their financial situation by advertising on it. The evidence is a correlation between an improvement in finances and small companies who haven’t previously advertised on the Internet starting to do so.

B. No. This is too strong. This concludes that most small companies can probably improve their financial situations by advertising on the Internet, on the basis of evidence that certain small companies improved their financial situations by doing so.

C. No. This is too strong. This concludes that it must be true that any small company that increases Internet advertising will improve its financial situation. The above argument doesn’t claim that all people will decrease their anxiety.

D. No. This claims that it is necessary for a small company to start to advertise on the Internet in order to improve their financial situation. The above argument claims that starting to sleep more than eight hours a night is sufficient for a reduction in anxiety.

E. No. This is too strong. This concludes that most small companies that have never advertised on the Internet but start doing so could probably become financially strong. The above argument claims that people’s anxiety levels might drop, not that they will become anxiety-free.

How well did you know this?
1
Not at all
2
3
4
5
Perfectly
94
Q

Biologist: Lions and tigers are so similar to each
other anatomically that their skeletons are
virtually indistinguishable. But their behaviors
are known to be quite different: tigers hunt
only as solitary individuals, whereas lions hunt
in packs. Thus, paleontologists cannot
reasonably infer solely on the basis of skeletal
anatomy that extinct predatory animals, such
as certain dinosaurs, hunted in packs.
The conclusion is properly drawn if which one of the
following is assumed?
(A) The skeletons of lions and tigers are at least
somewhat similar in structure in certain key
respects to the skeletons of at least some
extinct predatory animals.
(B) There have existed at least two species of
extinct predatory dinosaurs that were so
similar to each other that their skeletal
anatomy is virtually indistinguishable.
(C) If skeletal anatomy alone is ever an inadequate
basis for inferring a particular species’
hunting behavior, then it is never reasonable
to infer, based on skeletal anatomy alone, that
a species of animals hunted in packs.
(D) If any two animal species with virtually
indistinguishable skeletal anatomy exhibit
quite different hunting behaviors, then it is
never reasonable to infer, based solely on the
hunting behavior of those species, that the
two species have the same skeletal anatomy.
(E) If it is unreasonable to infer, solely on the basis
of differences in skeletal anatomy, that extinct
animals of two distinct species differed in
their hunting behavior, then the skeletal
remains of those two species are virtually
indistinguishable.

A

Correct Answer: C

C Assumption Sufficient

The biologist concludes that paleontologists cannot reasonably infer that extinct predatory animals hunted in packs, solely on the basis of skeletal anatomy. The biologist cites the differences between the hunting patterns of lions and tigers, whose skeletons are virtually indistinguishable. Tigers hunt alone, while lions hunt in packs. The credited response will strengthen the analogy between tigers and lions, on the one hand, and the skeletons of extinct predatory animals, on the other.

A. No. The skeletons themselves don’t need to be similar. The important part of the analogy is the similarity in the skeletons of creatures and their difference in hunting habits.

B. No. This doesn’t address the difference in hunting habits.

C. Yes. The biologist claims that skeletal anatomy alone is an inadequate basis for inferring the hunting behavior of tigers and lions. So, given this answer choice, it is not reasonable to infer that extinct predatory animals hunted in packs, based on skeletal anatomy alone.

D. No. According to the argument, lions and tigers have virtually indistinguishable anatomy. Plus, the argument makes a claim about hunting behaviors based on skeletal anatomy, not the other way around.

E. No. Lions and tigers are not extinct. Also, the argument makes a claim about hunting behaviors based on skeletal anatomy, not the other way around.

How well did you know this?
1
Not at all
2
3
4
5
Perfectly
95
Q
  1. Doctor: Being overweight has long been linked with a
    variety of health problems, such as high blood
    pressure and heart disease. But recent research
    conclusively shows that people who are slightly
    overweight are healthier than those who are
    considerably underweight. Therefore, to be
    healthy, it suffices to be slightly overweight.
    The argument’s reasoning is flawed because the
    argument
    (A) ignores medical opinions that tend to lead to a
    conclusion contrary to the one drawn
    (B) never adequately defines what is meant by
    “healthy”
    (C) does not take into account the fact that
    appropriate weight varies greatly from person
    to person
    (D) holds that if a person lacks a property that
    would suffice to make the person unhealthy,
    then that person must be healthy
    (E) mistakes a merely relative property for one
    that is absolute
A

Correct Answer: E

E Flaw

The doctor concludes that being slightly overweight is sufficient to be healthy. The doctor cites recent research that conclusively shows that people who are slightly overweight are healthier than those who are considerably underweight. He bases an absolute—being healthy—on a comparison: which of two groups of people is healthier.

A. No. The doctor acknowledges the previous medical opinions to the contrary but thinks that the new evidence supports his own conclusion.

B. No. This is never the credited response.

C. No. The doctor never discusses absolute numbers in terms of people’s weights.

D. No. The doctor never discusses a property that would be sufficient to make people unhealthy.

E. Yes. Being healthier is a merely relative property, while being healthy is an absolute property.

How well did you know this?
1
Not at all
2
3
4
5
Perfectly
96
Q

The trees always blossom in May if April rainfall
exceeds 5 centimeters. If April rainfall exceeds 5
centimeters, then the reservoirs are always full on
May 1. The reservoirs were not full this May 1 and
thus the trees will not blossom this May.
Which one of the following exhibits a flawed pattern
of reasoning most similar to the flawed pattern of
reasoning in the argument above?
(A) If the garlic is in the pantry, then it is still
fresh. And the potatoes are on the basement
stairs if the garlic is in the pantry. The
potatoes are not on the basement stairs, so
the garlic is not still fresh.
(B) The jar reaches optimal temperature if it is
held over the burner for 2 minutes. The
contents of the jar liquefy immediately if the
jar is at optimal temperature. The jar was held
over the burner for 2 minutes, so the contents
of the jar must have liquefied immediately.
(C) A book is classified “special” if it is more than
200 years old. If a book was set with wooden
type, then it is more than 200 years old. This
book is not classified “special,” so it is not
printed with wooden type.
(D) The mower will operate only if the engine is
not flooded. The engine is flooded if the foot
pedal is depressed. The foot pedal is not
depressed, so the mower will operate.
(E) If the kiln is too hot, then the plates will crack.
If the plates crack, then the artisan must redo
the order. The artisan need not redo the
order. Thus, the kiln was not too hot.

A

Correct Answer: A

A Parallel Flaw

This argument is diagrammable. April rainfall exceeds 5 centimeters → trees blossom in May; ~trees blossom in May → ~April rainfall exceeds 5 centimeters. April rainfall exceeds 5 centimeters → reservoirs full on May 1; ~reservoirs full on May 1 → ~April rainfall exceeds 5 centimeters. Conclusion: ~reservoirs full on May 1 → ~trees blossom in May. The conclusion claims that the reservoirs not being full on May 1 is sufficient to know that the trees will not blossom in May, while the evidence does not support this claim. The argument doesn’t flip the terms in the contrapositive of the first premise.

A. Yes. Garlic in pantry → still fresh; → ~still fresh ~garlic in pantry. Garlic in pantry → potatoes on basement stairs; ~potatoes on stairs → ~garlic in pantry. Conclusion: ~potatoes on stairs → ~garlic still fresh.

B. No. Held over burner for two minutes → optimal temperature; ~optimal temperature → ~held over burner for two minutes. Optimal temperature contents liquefy immediately; ~liquefy immediately → ~optimal temperature. Conclusion: held over burner for two minutes → liquefied immediately. This argument is not flawed.

C. No. More than 200 years old → classified “special”; ~classified “special” → ~more than 200 years old. Set with wooden type → more than 200 years old; ~more than 200 years old → ~set with wooden type. Conclusion: ~classified “special” → ~printed with wooden type. This argument is not flawed.

D. No. Mower operates → ~engine flooded; engine flooded → ~mower operates. Foot pedal depressed → engine flooded; → ~engine flooded ~foot pedal depressed. Conclusion: ~foot pedal depressed mower operates. This argument is flawed, but not in the same manner as the original argument.

E. No. Kiln too hot → plates crack; ~plates crack → ~kiln too hot. Plates crack → redo; → ~redo ~plates crack. Conclusion: ~redo → ~kiln too hot. This argument is not flawed.

97
Q

Robust crops not only withstand insect attacks more
successfully than other crops, they are also less likely
to be attacked in the first place, since insects tend to
feed on weaker plants. Killing insects with pesticides
does not address the underlying problem of inherent
vulnerability to damage caused by insect attacks.
Thus, a better way to reduce the vulnerability of
agricultural crops to insect pest damage is to grow
those crops in good soil—soil with adequate
nutrients, organic matter, and microbial activity.
Which one of the following is an assumption on
which the argument depends?
(A) The application of nutrients and organic
matter to farmland improves the soil’s
microbial activity.
(B) Insects never attack crops grown in soil
containing adequate nutrients, organic
matter, and microbial activity.
(C) The application of pesticides to weak crops
fails to reduce the extent to which they are
damaged by insect pests.
(D) Crops that are grown in good soil tend to be
more robust than other crops.
(E) Growing crops without the use of pesticides
generally produces less robust plants than
when pesticides are used.

A

Correct Answer: D

D Necessary Assumption

The argument solves a problem with killing insects with pesticides. Using pesticides does not address the underlying problem of weaker plants being more vulnerable to damage caused by insect attacks. This is because insects tend to feed on weaker plants, while more robust plants are less likely to be attacked in the first place. More robust plants are also more likely to withstand insects’ attacks more successfully. Instead of using pesticides, the argument concludes that a better way to reduce the vulnerability of crops to insect damage is to grow the crops in good soil. The credited response will close the gap between robust plants and growing plants in good soil.

A. No. This doesn’t address the robustness of the plants.

B. No. This is too strong. The argument claims that growing crops in good soil will reduce the damage, not eliminate it.

C. No. This is too strong. The argument claims that growing crops in good soil is a better way to reduce the damage. The pesticides just need to be less effective than growing crops in good soil.

D. Yes. This connects growing crops in good soil to their increased robustness.

E. No. This would weaken the argument, as the argument advocates growing crops in good soil as opposed to using pesticides.

98
Q

People perceive color by means of certain
photopigments in the retina that are sensitive to
certain wavelengths of light. People who are colorblind are unable to distinguish between red and
green, for example, due to an absence of certain
photopigments. What is difficult to explain, however,
is that in a study of people who easily distinguish red
from green, 10 to 20 percent failed to report
distinctions between many shades of red that the
majority of the subjects were able to distinguish.
Each of the following, if true, helps to explain the
result of the study cited above EXCEPT:
(A) People with abnormally low concentrations of
the photopigments for perceiving red can
perceive fewer shades of red than people with
normal concentrations.
(B) Questions that ask subjects to distinguish
between different shades of the same color are
difficult to phrase with complete clarity.
(C) Some people are uninterested in fine
gradations of color and fail to notice or
report differences they do not care about.
(D) Some people are unable to distinguish red
from green due to an absence in the retina of
the photopigment sensitive to green.
(E) Some people fail to report distinctions
between certain shades of red because they
lack the names for those shades.

A

Correct Answer: D

D Resolve/Explain

Fact 1: People perceive color by means of certain photopigments in the retina that are sensitive to certain wavelengths of light. Fact 2: Of people who easily distinguish between red and green, 10 to 20 percent in a certain study failed to report distinctions between many shades of red that most subjects were able to distinguish.

A. No. This would explain why 10 to 20 percent could distinguish between red and green, but didn’t report distinctions between many shades of red.

B. No. This would explain why 10 to 20 percent failed to report distinctions. They might not have understood the questions.

C. No. This would explain why 10 to 20 percent failed to report distinctions. They could potentially see them but they just failed to notice them because they don’t care.

D. Yes. The people in the study were easily able to distinguish red from green, so this evidence doesn’t help explain the result of the study.

E. No. This would explain why 10 to 20 percent failed to report distinctions. They might not have had the vocabulary to report such distinctions.

99
Q

Occultist: The issue of whether astrology is a science
is easily settled: it is both an art and a science.
The scientific components are the complicated
mathematics and the astronomical knowledge
needed to create an astrological chart. The art
is in the synthesis of a multitude of factors and
symbols into a coherent statement of their
relevance to an individual.
The reasoning in the occultist’s argument is most
vulnerable to criticism on the grounds that the
argument
(A) presumes, without providing justification, that
any science must involve complicated
mathematics
(B) incorrectly infers that a practice is a science
merely from the fact that the practice has
some scientific components
(C) denies the possibility that astrology involves
components that are neither artistic nor
scientific
(D) incorrectly infers that astronomical knowledge
is scientific merely from the fact that such
knowledge is needed to create an astrological
chart
(E) presumes, without providing justification, that
any art must involve the synthesis of a
multitude of factors and symbols

A

Correct Answer: B

B Flaw

The occultist concludes that astrology is both an art and a science. To create an astrological chart, complicated mathematics and astronomical knowledge are needed, which are scientific components. The synthesis of a multitude of factors and symbols into a coherent statement is the art component. The occultist is making a part-whole mistake. That the parts have certain qualities does not mean that the whole has these qualities.

A. No. The occultist doesn’t address all sciences.

B. Yes. The occultist claims that astrology is a science because it has scientific components.

C. No. The occultist doesn’t claim that the components cited are the only components of astrology.

D. No. The occultist doesn’t claim that astronomical knowledge is scientific just because it is used to create an astrological chart. The occultist states from the onset that astronomical knowledge is scientific.

E. No. The occultist doesn’t address all arts.

100
Q

At many electronics retail stores, the consumer has
the option of purchasing product warranties that
extend beyond the manufacturer’s warranty.
However, consumers are generally better off not
buying extended warranties. Most problems with
electronic goods occur within the period covered by
the manufacturer’s warranty.
Which one of the following, if true, most strengthens
the argument?
(A) Problems with electronic goods that occur
after the manufacturer’s warranty expires are
generally inexpensive to fix in comparison
with the cost of an extended warranty.
(B) Because problems are so infrequent after the
manufacturer’s warranty expires, extended
warranties on electronic goods are generally
inexpensive.
(C) Most of those who buy extended warranties on
electronic goods do so because special
circumstances make their item more likely to
break than is usually the case.
(D) Some extended warranties on electronic goods
cover the product for the period covered by
the manufacturer’s warranty as well as
subsequent years.
(E) Retail stores sell extended warranties in part
because consumers who purchase them are
likely to purchase other products from the
same store.

A

Correct Answer: A

A Strengthen

The argument concludes that consumers are generally better off not buying the extended warranties that are offered for sale at many electronics stores. While these warranties extend beyond the manufacturer’s warranty, most problems with electronic goods occur within the span of time covered by the manufacturer’s warranty. The credited response will eliminate any other reasons—such as cost—that would make the warranties worthwhile for customers.

A. Yes. If the extended warranty is more expensive than the cost to fix the problems the electronic goods are likely to have, then there seems to be no good reason to buy the warranty.

B. No. This weakens the argument, if anything. If the warranties are generally inexpensive, they might be worthwhile to buy, just in case your electronics do have a problem after the manufacturer’s warranty expires.

C. No. This is irrelevant.

D. No. This is irrelevant.

E. No. The reason that retail stores sell the warranties doesn’t matter.

101
Q
  1. Since the 1970s, environmentalists have largely
    succeeded in convincing legislators to enact extensive
    environmental regulations. Yet, as environmentalists
    themselves not only admit but insist, the condition of
    the environment is worsening, not improving.
    Clearly, more environmental regulations are not the
    solution to the environment’s problems.
    The argument’s reasoning is flawed because the
    argument
    (A) attacks the environmentalists themselves
    instead of their positions
    (B) presumes, without providing warrant, that
    only an absence of environmental regulations
    could prevent environmental degradation
    (C) fails to consider the possibility that the
    condition of the environment would have
    worsened even more without environmental
    regulations
    (D) fails to justify its presumption that reducing
    excessive regulations is more important than
    preserving the environment
    (E) fails to consider the views of the
    environmentalists’ opponents
A

Correct Answer: C

C Flaw

The argument concludes that more environmental regulations are not the solution to the environment’s problems. This is because, as environmentalists insist, the condition of the environment is worsening, even though environmentalists have been successful in convincing legislators to enact extensive environmental regulations. However, there is no evidence to suggest that these regulations have had no effect; perhaps the condition of the environment is not as bad as it would have been had there been no environmental regulations.

A. No. There is no personal attack.

B. No. This is too strong. The argument maintains that the environmental regulations are not the solution to the problem, but it never claims that the prevention of environmental degradation requires the absence of environmental regulations.

C. Yes. It is certainly possible that the environment would have worsened even more than it did without environmental regulations.

D. No. The importance of reducing regulations versus the importance of the environment is not discussed.

E. No. The argument’s author is an opponent of the environmentalists.

102
Q

Although it is unwise to take a developmental view of
an art like music—as if Beethoven were an advance
over Josquin, or Miles Davis an advance over Louis
Armstrong—there are ways in which it makes sense
to talk about musical knowledge growing over time.
We certainly know more about certain sounds than
was known five centuries ago; that is, we understand
how sounds that earlier composers avoided can be
used effectively in musical compositions. For
example, we now know how the interval of the third,
which is considered dissonant, can be used in
compositions to create consonant musical phrases.
Which one of the following most accurately expresses
the main conclusion of the argument?
(A) Sounds that were never used in past musical
compositions are used today.
(B) Sounds that were once considered dissonant
are more pleasing to modern listeners.
(C) It is inappropriate to take a developmental
view of music.
(D) It is unwise to say that one composer is better
than another.
(E) Our understanding of music can improve over
the course of time.

A

Correct Answer: E

E Main Point

The argument concludes that there are ways in which it makes sense to talk about musical knowledge growing over time. While it is not advocating a developmental view of music, the argument claims that we certainly know more about how to effectively use certain sounds that earlier composers avoided in musical compositions. An example is the interval of the third.

A. No. There were sounds that were avoided, but the argument doesn’t discuss sounds that were never used.

B. No. The argument doesn’t claim anything about what is more pleasing to modern listeners.

C. No. This is a qualifier to the conclusion but not the conclusion itself.

D. No. The argument doesn’t discuss all value judgments in music.

E. Yes. This is a restatement of the main point.

103
Q

A recent test of an electric insect control device
discovered that, of the more than 300 insects killed
during one 24-hour period, only 12 were mosquitoes.
Thus this type of device may kill many insects, but
will not significantly aid in controlling the
potentially dangerous mosquito population.
Which one of the following, if true, most seriously
weakens the argument?
(A) A careful search discovered no live mosquitoes
in the vicinity of the device after the test.
(B) A very large proportion of the insects that
were attracted to the device were not
mosquitoes.
(C) The device is more likely to kill beneficial
insects than it is to kill harmful insects.
(D) Many of the insects that were killed by the
device are mosquito-eating insects.
(E) The device does not succeed in killing all of the
insects that it attracts.

A

Correct Answer: A

A Weaken

The argument interprets the evidence concerning the electric insect control device to indicate that this type of electric insect control device may kill many insects but will not significantly aid in controlling the potentially dangerous mosquito population. The evidence is that, during a 24-hour period, the device killed more than 300 insects but killed only 12 mosquitoes. The argument is overlooking the fact that the actual number of mosquitoes killed might not indicate effectiveness against mosquitoes as well as the percentage of mosquitoes killed. The credited response will widen this gap.

A. Yes. If the device killed all of the mosquitoes present in the area, then it is highly effective at killing mosquitoes. The small number killed just means that there weren’t many mosquitoes in the area during that 24-hour period.

B. No. The proportion of insects attracted to the device is not relevant.

C. No. This strengthens the argument, if anything. If it is less likely to kill harmful insects, then it will be less likely to kill the potentially harmful mosquitoes.

D. No. This strengthens the argument. If the device kills a lot of mosquito-eating insects, then there will be less of them to help control the mosquitoes.

E. No. This is irrelevant. The argument is concerned specifically with the mosquitoes.

104
Q
  1. Brain-scanning technology provides information about
    processes occurring in the brain. For this information
    to help researchers understand how the brain enables
    us to think, however, researchers must be able to rely
    on the accuracy of the verbal reports given by subjects
    while their brains are being scanned. Otherwise brainscan data gathered at a given moment might not
    contain information about what the subject reports
    thinking about at that moment, but instead about
    some different set of thoughts.
    Which one of the following most accurately expresses
    the main conclusion of the argument?
    (A) It is unlikely that brain-scanning technology
    will ever enable researchers to understand
    how the brain enables us to think.
    (B) There is no way that researchers can know for
    certain that subjects whose brains are being
    scanned are accurately reporting what they
    are thinking.
    (C) Because subjects whose brains are being
    scanned may not accurately report what they
    are thinking, the results of brain-scanning
    research should be regarded with great
    skepticism.
    (D) Brain scans can provide information about the
    accuracy of the verbal reports of subjects
    whose brains are being scanned.
    (E) Information from brain scans can help
    researchers understand how the brain enables
    us to think only if the verbal reports of those
    whose brains are being scanned are accurate.
A

Correct Answer: E

E Main Point

Connecting the first two sentences of the argument results in the main point. Brain-scanning technology provides information about brain processes but only if researchers can rely on the accuracy of the verbal reports given by the subjects while the scan is going on. If the reports are inaccurate, the data might not contain information about the thoughts reported.

A. No. The argument does not make claims about the likelihood of the technology enabling researchers to understand how the brain enables us to think.

B. No. This raises a potential problem concerning the accuracy of the reports, but it is not the conclusion of the argument.

C. No. The argument does not make claims about how skeptically we should regard the results of brain-scanning research.

D. No. The argument never states this.

E. Yes. This is a restatement of the first two sentences of the argument, where the point of the argument is located.

104
Q
  1. Ornithologist: This bird species is widely thought to
    subsist primarily on vegetation, but my research
    shows that this belief is erroneous. While
    concealed in a well-camouflaged blind, I have
    observed hundreds of these birds every morning
    over a period of months, and I estimate that
    over half of what they ate consisted of insects
    and other animal food sources.
    The reasoning in the ornithologist’s argument is
    most vulnerable to criticism on the grounds that the
    argument
    (A) assumes, without providing justification, that
    the feeding behavior of the birds observed
    was not affected by the ornithologist’s act of
    observation
    (B) fails to specify the nature of the animal food
    sources, other than insects, that were
    consumed by the birds
    (C) adopts a widespread belief about the birds’
    feeding habits without considering the
    evidence that led to the belief
    (D) neglects the possibility that the birds have
    different patterns of food consumption
    during different parts of the day and night
    (E) fails to consider the possibility that the birds’
    diet has changed since the earlier belief about
    their diet was formed
A

Correct Answer: D

D Flaw

The ornithologist disagrees with those who think that a certain bird species subsists primarily on vegetation. The ornithologist estimates that over half of what these birds eat consists of insects and other animal food sources. The ornithologist bases this claim on the observation of hundreds of these birds every morning while concealed in a well-camouflaged blind. The ornithologist has observed the birds at the same time every day. If someone did that to humans, isn’t it likely that they would conclude that humans primarily subsist on breakfast cereal and orange juice?

A. No. The ornithologist mentions that he/she was concealed in a well-camouflaged blind.

B. No. This doesn’t matter. As long as they were animal food sources, they support the ornithologist’s conclusion.

C. No. The ornithologist does not adopt a widespread belief; he actually looks to counter one with his own research.

D. Yes. The ornithologist observed the birds only in the morning. The birds might eat a lot of animals in the morning and then eat exclusively vegetation throughout the rest of the day.

E. No. The belief cited is about what the birds do eat, not what they have eaten in the past.

105
Q
  1. Educator: Only those students who are genuinely
    curious about a topic can successfully learn
    about that topic. They find the satisfaction of
    their curiosity intrinsically gratifying, and
    appreciate the inherent rewards of the learning
    process itself. However, almost no child enters
    the classroom with sufficient curiosity to learn
    successfully all that the teacher must instill. A
    teacher’s job, therefore, _______.
    Which one of the following most logically completes
    the educator’s argument?
    (A) requires for the fulfillment of its goals the
    stimulation as well as the satisfaction of
    curiosity
    (B) necessitates the creative use of rewards that are
    not inherent in the learning process itself
    (C) is to focus primarily on those topics that do
    not initially interest the students
    (D) is facilitated by students’ taking responsibility
    for their own learning
    (E) becomes easier if students realize that some
    learning is not necessarily enjoyable
A

Correct Answer: A

A Main Point

The first statement is diagrammable. Students can successfully learn a topic genuinely → curious about the topic; ~genuinely curious → ~successfully learn. Almost no child starts out curious enough about all the topics that a teacher must instill. The argument is structured to conclude something about teachers needing to make students genuinely curious about all of the topics that the students need to successfully learn.

A. Yes. In order to ensure that students will successfully learn all of the topics that they need to learn, a teacher must make students genuinely curious about those topics for which they hadn’t previously developed a sufficient level of curiosity.

B. No. The evidence does not discuss how rewards relate to curiosity.

C. No. The argument is making a connection between the teacher’s job and the students’ curiosity. Focusing on these topics won’t ensure that they’ll become curious enough to successfully learn these topics.

D. No. The evidence does not discuss what the students’ responsibilities are.

E. No. The argument connects genuine curiosity and enjoyment of learning.

105
Q
  1. Environmentalist: When bacteria degrade household
    cleaning products, vapors that are toxic to
    humans are produced. Unfortunately,
    household cleaning products are often found
    in landfills. Thus, the common practice of
    converting landfills into public parks is
    damaging human health.
    Which one of the following is an assumption the
    environmentalist’s argument requires?
    (A) In at least some landfills that have been
    converted into public parks there are bacteria
    that degrade household cleaning products.
    (B) Converting a landfill into a public park will
    cause no damage to human health unless
    toxic vapors are produced in that landfill and
    humans are exposed to them.
    (C) If a practice involves the exposure of humans
    to vapors from household cleaning products,
    then it causes at least some damage to human
    health.
    (D) When landfills are converted to public parks,
    measures could be taken that would prevent
    people using the parks from being exposed to
    toxic vapors.
    (E) If vapors toxic to humans are produced by the
    degradation of household cleaning products
    by bacteria in any landfill, then the health of
    at least some humans will suffer.
A

Correct Answer: A

A Necessary Assumption

The environmentalist argues that the common practice of converting landfills into public parks is damaging human health. The environmentalist cites the fact that household cleaning products are often found in landfills; when bacteria degrade these cleaning products, toxic vapors are produced. There is a gap between damaging human health by converting landfills to public parks and whether, in these landfills, bacteria are actually degrading the cleaning products.

A. Yes. This makes a connection between the converted landfills and the bacteria that degrade the products, thereby causing toxic vapors to be emitted.

B. No. This doesn’t state that there are bacteria in these converted landfills that will degrade the cleaning products.

C. No. This doesn’t state that there are bacteria in these converted landfills that will degrade the cleaning products.

D. No. This would weaken the argument. If people weren’t exposed to these vapors, the converted landfills wouldn’t necessarily be damaging to human health.

E. No. This is too general. The environmentalist is arguing specifically about the practice of converting landfills into public parks, not about landfills in general.

106
Q
  1. Tea made from camellia leaves is a popular beverage.
    However, studies show that regular drinkers of
    camellia tea usually suffer withdrawal symptoms if
    they discontinue drinking the tea. Furthermore,
    regular drinkers of camellia tea are more likely than
    people in general to develop kidney damage. Regular
    consumption of this tea, therefore, can result in a
    heightened risk of kidney damage.
    Which one of the following, if true, most seriously
    weakens the argument?
    (A) Several other popular beverages contain the
    same addictive chemical that is found in
    camellia tea.
    (B) Addictive chemicals are unlikely to cause
    kidney damage solely by virtue of their
    addictive qualities.
    (C) Some people claim that regular consumption
    of camellia tea helps alleviate their stress.
    (D) Most people who regularly drink camellia tea
    do not develop kidney damage.
    (E) Many people who regularly consume camellia
    tea also regularly consume other beverages
    suspected of causing kidney damage.
A

Correct Answer: E

E Weaken

The argument concludes that regular consumption of camellia tea can result in a heightened risk of kidney damage. This is because studies show that regular drinkers of camellia tea are more likely than people in general to develop kidney damage. The argument is treating a correlation as if one event causes the other. It is overlooking another possible cause for the increased risk of kidney damage.

A. No. The argument is concerned with camellia tea and the risk of kidney damage associated with drinking it, not other popular beverages.

B. No. This is too general. The argument is concerned with the chemicals in camellia tea, not addictive chemicals in general.

C. No. The argument doesn’t mention stress levels.

D. No. This doesn’t go far enough. As long as more people who regularly drink camellia tea develop kidney damage than those who do not, the conclusion may still hold.

E. Yes. This presents another possible cause for the kidney damage, which makes it less probable that it is the tea that is causing the greater incidence of kidney damage.

107
Q

Artist: Avant-garde artists intend their work to
challenge a society’s mainstream beliefs and
initiate change. And some art collectors claim
that an avant-garde work that becomes
popular in its own time is successful. However,
a society’s mainstream beliefs do not generally
show any significant changes over a short
period of time. Therefore, when an avantgarde work becomes popular it is a sign that
the work is not successful, since it does not
fulfil the intentions of its creator.
The reference to the claim of certain art collectors
plays which one of the following roles in the artist’s
argument?
(A) It serves to bolster the argument’s main
conclusion.
(B) It identifies a view that is ultimately disputed
by the argument.
(C) It identifies a position supported by the initial
premise in the argument.
(D) It provides support for the initial premise in
the argument.
(E) It provides support for a counterargument to
the initial premise.

A

Correct Answer: B

B Reasoning

The artist disagrees with those art collectors who claim that an avant-garde work that becomes popular in its own time is successful. Instead, the artist argues that when an avant-garde work becomes popular, it is a sign that the work is not successful. This is because avant-garde artists intend their work to challenge a society’s mainstream beliefs and initiate change, and a society’s mainstream beliefs do not generally show any significant change over a short period of time.

A. No. The artist argues against these art collectors.

B. Yes. The artist brings up the claims of these art collectors in order to dispute them.

C. No. The premise about the avant-garde artists’ intentions does not support the claims of these art collectors.

D. No. The claims of these art collectors do not provide support for the premise about the avant-garde artists’ intentions.

E. No. The claims of the art collectors are not part of a counterargument.

108
Q

A recent epidemiological study found that
businesspeople who travel internationally on
business are much more likely to suffer from chronic
insomnia than are businesspeople who do not travel
on business. International travelers experience the
stresses of dramatic changes in climate, frequent
disruption of daily routines, and immersion in
cultures other than their own, stresses not commonly
felt by those who do not travel. Thus, it is likely that
these stresses cause the insomnia.
Which one of the following would, if true, most
strengthen the reasoning above?
(A) Most international travel for the sake of
business occurs between countries with
contiguous borders.
(B) Some businesspeople who travel
internationally greatly enjoy the changes in
climate and immersion in another culture.
(C) Businesspeople who already suffer from
chronic insomnia are no more likely than
businesspeople who do not to accept
assignments from their employers that
require international travel.
(D) Experiencing dramatic changes in climate and
disruption of daily routines through
international travel can be beneficial to some
people who suffer from chronic insomnia.
(E) Some businesspeople who once traveled
internationally but no longer do so complain
of various sleep-related ailments.

A

Correct Answer: C

C Strengthen

The argument concludes that it is likely that the stresses felt more commonly while traveling cause the insomnia of businesspeople who travel internationally on business. These businesspeople are much more likely to suffer from chronic insomnia than are the businesspeople who don’t travel for business. The businesspeople who travel internationally on business frequently experience stresses that are not commonly felt by those who do not travel.

A. No. The argument discusses international travel in general. It doesn’t matter whether the borders of the countries visited are contiguous or not.

B. No. This would weaken the argument, if anything. If some businesspeople who travel greatly enjoy the changes in climate and immersion in another culture, then these are not stresses for them.

C. Yes. This eliminates the possibility that there is another reason for the increased insomnia in businesspeople who must travel internationally.

D. No. This would weaken the argument, as it suggests that the changes and disruptions may ameliorate insomnia instead of cause it.

E. No. The argument is concerned with businesspeople who are currently traveling internationally, not those who once did.

109
Q

Each of the smallest particles in the universe has an
elegantly simple structure. Since these particles
compose the universe, we can conclude that the
universe itself has an elegantly simple structure.
Each of the following arguments exhibits flawed
reasoning similar to that in the argument above
EXCEPT:
(A) Each part of this car is nearly perfectly
engineered. Therefore this car is nearly
perfect, from an engineering point of view.
(B) Each part of this desk is made of metal.
Therefore this desk is made of metal.
(C) Each brick in this wall is rectangular. Therefore
this wall is rectangular.
(D) Each piece of wood in this chair is sturdy.
Therefore this chair is sturdy.
(E) Each sentence in this novel is well constructed.
Therefore this is a well-constructed novel.

A

Correct Answer: B

B Parallel Flaw

The argument concludes that the universe has an elegantly simple structure, on the basis of evidence that each of the smallest particles in the universe has an elegantly simple structure and that these particles compose the universe. The argument exhibits a part-whole flaw.

A. No. This argument claims that the car is nearly perfectly engineered, on the basis of a characteristic of its parts. This does exhibit the same flaw.

B. Yes. While this argument makes a claim about the desk as a whole on the basis of its parts, it is warranted to do so. If all the parts of the desk are made of metal, then the desk must be made of metal.

C. No. This argument claims that the wall is rectangular because its parts are. However, bricks can make walls of all shapes. So, this exhibits the same flaw.

D. No. This argument claims that the chair is sturdy because all of its parts are. Yet, the chair could be poorly glued or poorly constructed in some other way. So, this exhibits the same flaw.

E. No. The novel might consist of well-constructed sentences but the sentences might not form good prose. So, this exhibits the same flaw.

109
Q

Many mountain climbers regard climbing Mount
Everest as the ultimate achievement. But climbers
should not attempt this climb since the risk of death
or serious injury in an Everest expedition is very
high. Moreover, the romantic notion of gaining
“spiritual discovery” atop Everest is dispelled by
climbers’ reports that the only profound experiences
they had at the top were of exhaustion and fear.
Which one of the following principles, if valid, most
helps to justify the reasoning above?
(A) Projects undertaken primarily for spiritual
reasons ought to be abandoned if the risks are
great.
(B) Dangerous activities that are unlikely to result
in significant spiritual benefits for those
undertaking them should be avoided.
(C) Activities that are extremely dangerous ought
to be legally prohibited unless they are
necessary to produce spiritual enlightenment.
(D) Profound spiritual experiences can be achieved
without undergoing the serious danger
involved in mountain climbing.
(E) Mountain climbers and other athletes should
carefully examine the underlying reasons they
have for participating in their sports.

A

Correct Answer: B

B Principle Strengthen

The argument advocates against climbers trying to climb Mount Everest because the risk of death or injury is very high, and the climb does not seem to actually enable one to gain “spiritual discovery.”

A. No. The argument doesn’t state that climbing Mount Everest is undertaken primarily for spiritual reasons.

B. Yes. The argument highlights the fact that climbing Mount Everest is dangerous and that climbing Mount Everest is unlikely to result in significant spiritual benefits. This principle would thereby help justify the conclusion that climbers should not attempt the climb.

C. No. The argument doesn’t claim that climbing Mount Everest should be legally prohibited.

D. No. The argument doesn’t claim that there are other ways to achieve profound spiritual experiences.

E. No. This isn’t strong enough. The principle needs to justify the conclusion that mountain climbers should not try to climb Mount Everest.

110
Q

Criminologist: A judicial system that tries and
punishes criminals without delay is an
effective deterrent to violent crime. Long,
drawn-out trials and successful legal
maneuvering may add to criminals’ feelings of
invulnerability. But if potential violent
criminals know that being caught means
prompt punishment, they will hesitate to break
the law.
Which one of the following, if true, would most
seriously weaken the criminologist’s argument?
(A) It is in the nature of violent crime that it is not
premeditated.
(B) About one-fourth of all suspects first arrested
for a crime are actually innocent.
(C) Many violent crimes are committed by firsttime offenders.
(D) Everyone accused of a crime has the right to a
trial.
(E) Countries that promptly punish suspected
lawbreakers have lower crime rates than
countries that allow long trials.

A

Correct Answer: A

A Weaken

The criminologist concludes that a judicial system that tries and punishes criminals without delay is an effective deterrent to violent crime. If potential violent criminals know that being caught means quick punishment, they will hesitate to break the law, whereas long, drawn-out trials may add to criminals’ feelings of invulnerability.

A. Yes. If potential violent criminals don’t think about their crimes, then they won’t be deterred by the prompt punishment.

B. No. This doesn’t claim that innocent people actually get convicted, just that some innocent people get arrested for a crime.

C. No. The number of offenses committed by violent criminals doesn’t matter.

D. No. This is too general. The argument is allowing for trials.

E. No. This strengthens the argument, as it shows a correlation between prompt punishment and a relatively lower crime rate.

111
Q

. Editorial: Contrary to popular belief, teaching
preschoolers is not especially difficult, for they
develop strict systems (e.g., for sorting toys by
shape), which help them to learn, and they are
always intensely curious about something new
in their world.
Which one of the following, if true, most seriously
weakens the editorial’s argument?
(A) Preschoolers have a tendency to imitate adults,
and most adults follow strict routines.
(B) Children intensely curious about new things
have very short attention spans.
(C) Some older children also develop strict systems
that help them learn.
(D) Preschoolers ask as many creative questions as
do older children.
(E) Preschool teachers generally report lower
levels of stress than do other teachers.

A

Correct Answer: B

B Weaken

The editorial concludes that teaching preschoolers is not especially difficult. The evidence is that preschoolers develop strict systems that help them to learn and that preschoolers are always intensely curious about new things. The credited response will give another reason to think that, as a result of their tendencies, preschoolers are difficult to teach.

A. No. This strengthens the argument. If preschoolers follow strict routines, they might well be easier to teach.

B. Yes. Since preschoolers are intensely curious about new things, they will have short attention spans. This is a reason to think that teaching preschoolers is difficult.

C. No. The editorial addresses preschoolers, not older children.

D. No. This isn’t strong enough. If they ask as many creative questions as do older children, and this doesn’t make older children particularly difficult to teach, then the argument’s conclusion would still hold.

E. No. This answer choice strengthens the argument. If preschool teachers are reporting lower stress levels, that gives us a reason to think that teaching preschoolers isn’t especially difficult.

111
Q

Journalist: Many people object to mandatory
retirement at age 65 as being arbitrary, arguing
that people over 65 make useful contributions.
However, if those who reach 65 are permitted to
continue working indefinitely, we will face
unacceptable outcomes. First, young people
entering the job market will not be able to
obtain decent jobs in the professions for which
they were trained, resulting in widespread
dissatisfaction among the young. Second, it is
not fair for those who have worked 40 or more
years to deprive others of opportunities.
Therefore, mandatory retirement should be
retained.
The journalist’s argument depends on assuming
which one of the following?
(A) Anyone who has worked 40 years is at least 65
years old.
(B) All young people entering the job market are
highly trained professionals.
(C) It is unfair for a person not to get a job in the
profession for which that person was trained.
(D) If people are forced to retire at age 65, there
will be much dissatisfaction among at least
some older people.
(E) If retirement ceases to be mandatory at age 65,
at least some people will choose to work past
age 65.

A

Correct Answer: E

E Necessary Assumption

The journalist argues against those people who object to mandatory retirement at age 65. The journalist gives two reasons for this. First, the young will become dissatisfied because they won’t be able to get decent jobs in the professions for which they were trained. Second, the people over 65 will be depriving others of opportunities and this is not fair.

A. No. The second claim concerns people who have worked 40 or more years.

B. No. The journalist never claims that all young people are highly trained.

C. No. The unfairness that the journalist points out is found in the second reason, while the training was referenced in the first reason.

D. No. The journalist isn’t concerned with the feelings of older people.

E. Yes. People might want to retire at 65 anyway. The reasons that the journalist cites for retaining mandatory retirement apply only if at least some people over 65 would still want to work.

112
Q

Lawyer: A body of circumstantial evidence is like a
rope, and each item of evidence is like a strand
of that rope. Just as additional pieces of
circumstantial evidence strengthen the body of
evidence, adding strands to the rope
strengthens the rope. And if one strand breaks,
the rope is not broken nor is its strength much
diminished. Thus, even if a few items of a body
of circumstantial evidence are discredited, the
overall body of evidence retains its basic
strength.
The reasoning in the lawyer’s argument is most
vulnerable to criticism on the grounds that the
argument
(A) takes for granted that no items in a body of
circumstantial evidence are significantly more
critical to the strength of the evidence than
other items in that body
(B) presumes, without providing justification, that
the strength of a body of evidence is less than
the sum of the strengths of the parts of that
body
(C) fails to consider the possibility that if many
items in a body of circumstantial evidence
were discredited, the overall body of evidence
would be discredited
(D) offers an analogy in support of a conclusion
without indicating whether the two types of
things compared share any similarities
(E) draws a conclusion that simply restates a claim
presented in support of that conclusion

A

Correct Answer: A

A Flaw

The lawyer concludes that, even if a few items of a body of circumstantial evidence are discredited, the overall body of evidence retains its basic strength. This conclusion is based on the analogy the lawyer makes between a body of circumstantial evidence and a rope. This is a bad analogy because while every strand of the rope is similar, different pieces of a body of circumstantial evidence might be more or less essential to its strength, depending.

A. Yes. This points out the problem with the lawyer’s analogy.

B. No. This is the wrong common flaw. The problem in the argument is not a part-whole problem.

C. No. The argument never claims that many items could be discredited and the overall body of evidence would remain strong.

D. No. This is close but the lawyer does indicate similarities between a body of circumstantial evidence and a rope.

E. No. The argument isn’t circular.

113
Q

People who object to the proposed hazardous waste
storage site by appealing to extremely implausible
scenarios in which the site fails to contain the waste
safely are overlooking the significant risks associated
with delays in moving the waste from its present
unsafe location. If we wait to remove the waste until
we find a site certain to contain it safely, the waste
will remain in its current location for many years,
since it is currently impossible to guarantee that any
site can meet that criterion. Yet keeping the waste at
the current location for that long clearly poses
unacceptable risks.
The statements above, if true, most strongly support
which one of the following?
(A) The waste should never have been stored in its
current location.
(B) The waste should be placed in the most secure
location that can ever be found.
(C) Moving the waste to the proposed site would
reduce the threat posed by the waste.
(D) Whenever waste must be moved, one should
limit the amount of time allotted to locating
alternative waste storage sites.
(E) Any site to which the waste could be moved
will be safer than its present site.

A

Correct Answer: C

C Inference

Choose the answer best supported by the passage.

A. No. The passage does not make any claims about whether the waste should have been initially stored in its current location.

B. No. The passage claims that the waste should be placed somewhere more secure. It doesn’t claim that it should be placed in the most secure location ever.

C. Yes. The passage claims that keeping the waste at the current location for as long as it takes to find a site certain to contain it safely would pose an unacceptable risk. So, moving the waste would reduce the threat posed.

D. No. This is too general. The passage discusses a specific instance of moving waste, not all waste moving.

E. No. This is too strong. The passage does not claim that any site would be safer.

114
Q

Ethicist: Many environmentalists hold that the
natural environment is morally valuable for its
own sake, regardless of any benefits it provides
us. However, even if nature has no moral
value, nature can be regarded as worth
preserving simply on the grounds that people
find it beautiful. Moreover, because it is
philosophically disputable whether nature is
morally valuable but undeniable that it is
beautiful, an argument for preserving nature
that emphasizes nature’s beauty will be less
vulnerable to logical objections than one that
emphasizes its moral value.
The ethicist’s reasoning most closely conforms to
which one of the following principles?
(A) An argument in favor of preserving nature will
be less open to logical objections if it avoids
the issue of what makes nature worth
preserving.
(B) If an argument for preserving nature
emphasizes a specific characteristic of nature
and is vulnerable to logical objections, then
that characteristic does not provide a
sufficient reason for preserving nature.
(C) If it is philosophically disputable whether
nature has a certain characteristic, then
nature would be more clearly worth
preserving if it did not have that
characteristic.
(D) Anything that has moral value is worth
preserving regardless of whether people
consider it to be beautiful.
(E) An argument for preserving nature will be less
open to logical objections if it appeals to a
characteristic that can be regarded as a basis
for preserving nature and that philosophically
indisputably belongs to nature.

A

Correct Answer: E

E Principle Match

The ethicist concludes that an argument for preserving nature emphasizing nature’s beauty will be less vulnerable to logical objections than one that emphasizes its moral value. This is because it is philosophically disputable whether nature is morally valuable but not disputable that it is beautiful.

A. No. The ethicist wants to change focus as to what makes nature worth preserving. She doesn’t want to avoid the issue of what makes nature worth preserving.

B. No. The ethicist does not judge whether the argument that emphasizes the moral value of nature provides a sufficient reason for preserving it.

C. No. The ethicist makes no judgment as to whether nature would be more clearly worth preserving if it didn’t have the characteristic of moral worth.

D. No. The ethicist’s argument concentrates on the beauty of nature, so this would weaken the thrust of the ethicist’s argument.

E. Yes. The ethicist argues that the argument based on natural beauty will be less open to logical objections because everyone agrees that beauty is a characteristic of nature and nature’s beauty can be regarded as a basis for preserving nature.

115
Q

An editor is compiling a textbook containing essays
by several different authors. The book will contain
essays by Lind, Knight, or Jones, but it will not
contain essays by all three. If the textbook contains
an essay by Knight, then it will also contain an essay
by Jones.
If the statements above are true, which one of the
following must be true?
(A) If the textbook contains an essay by Lind, then
it will not contain an essay by Knight.
(B) The textbook will contain an essay by only one
of Lind, Knight, and Jones.
(C) The textbook will not contain an essay by
Knight.
(D) If the textbook contains an essay by Lind, then
it will also contain an essay by Jones.
(E) The textbook will contain an essay by Lind.

A

Correct Answer: A

A Inference

This is diagrammable. The book will contain essays by Lind, Knight, or Jones but not all three. Contains essay by Knight → contains an essay by Jones. Connecting this with the previous information, contains essay by Knight → contains an essay by Jones and ~contains an essay by Lind. The contrapositive is as follows: contains an essay by Lind or ~contains an essay by Jones → ~contains an essay by Knight.

A. Yes. This is the contrapositive of the information given in the passage.

B. No. We know that if it contains one by Knight, it will contain an essay by Jones.

C. No. We don’t know this.

D. No. We don’t know about essays by Jones if the book contains an essay by Lind. The only thing that we know if the book contains an essay by Lind is that it won’t contain an essay by Knight.

E. No. We don’t know for sure which essays will be in the book.

116
Q

The ability of mammals to control their internal
body temperatures is a factor in the development of
their brains and intelligence. This can be seen from
the following facts: the brain is a chemical machine,
all chemical reactions are temperature dependent,
and any organism that can control its body
temperature can assure that these reactions occur at
the proper temperatures.
Which one of the following is an assumption on
which the argument depends?
(A) Organisms unable to control their body
temperatures do not have the capacity to
generate internal body heat without relying
on external factors.
(B) Mammals are the only animals that have the
ability to control their internal body
temperatures.
(C) The brain cannot support intelligence if the
chemical reactions within it are subject to
uncontrolled temperatures.
(D) The development of intelligence in mammals
is not independent of the chemical reactions
in their brains taking place at the proper
temperatures.
(E) Organisms incapable of controlling their
internal body temperatures are subject to
unpredictable chemical processes.

A

Correct Answer: D

D Necessary Assumption

The argument concludes that the ability of mammals to control their internal body temperatures is a factor in the development of their brains and intelligence. This conclusion is derived from the facts that the brain is a chemical machine, all chemical reactions are temperature dependent, and any organism that can control this can assure that the reactions occur at the proper temperatures. There is a gap between the evidence, which discusses elements and processes of the brain, and the conclusion, which talks about brains and intelligence.

A. No. The argument is about organisms that are able to control their body temperatures.

B. No. This is too strong. Mammals don’t have to be the only animals that have the ability to control their internal body temperatures for the conclusion to follow.

C. No. This is too strong. The argument claims that the ability to control internal body temperatures is a factor; it doesn’t need to be the only factor.

D. Yes. This closes the gap between the proper temperatures of brain processes and intelligence.

E. No. The argument is about organisms that can control their body temperatures.

117
Q

. A recent survey indicates that the average number of
books read annually per capita has declined in each
of the last three years. However, it also found that
most bookstores reported increased profits during
the same period.
Each of the following, if true, helps to resolve the
survey’s apparently paradoxical results EXCEPT:
(A) Recent cutbacks in government spending have
forced public libraries to purchase fewer
popular contemporary novels.
(B) Due to the installation of sophisticated new
antitheft equipment, the recent increase in
shoplifting that has hit most retail businesses
has left bookstores largely unaffected.
(C) Over the past few years many bookstores have
capitalized on the lucrative coffee industry by
installing coffee bars.
(D) Bookstore owners reported a general shift
away from the sale of inexpensive paperback
novels and toward the sale of lucrative
hardback books.
(E) Citing a lack of free time, many survey
respondents indicated that they had canceled
magazine subscriptions in favor of
purchasing individual issues at bookstores
when time permits.

A

Correct Answer: B

B Resolve/Explain

Fact 1: The average number of books read annually per capita has declined in each of the last three years. Fact 2: Most bookstores reported increased profits during this period.

A. No. This would explain why bookstores reported increased profits while the amount read has declined; more people may now buy popular contemporary novels since they can no longer borrow them for free from public libraries.

B. Yes. This does not explain why bookstores reported increased profits during the three-year period. The profits are higher than their own previous profits, not the profits of other stores.

C. No. This would explain where the increase in revenue came from, given that the average amount read has declined.

D. No. If bookstores were making more money per purchase, they could have an increase in profits even though they weren’t selling as many books.

E. No. The additional sales from magazines would help explain the bookstores’ increased profits, even though the average number of books read by each individual has declined.

118
Q

Naturalist: A species can survive a change in
environment, as long as the change is not too
rapid. Therefore, the threats we are creating to
woodland species arise not from the fact that
we are cutting down trees, but rather from the
rate at which we are doing so.
The reasoning in which one of the following is most
similar to that in the naturalist’s argument?
(A) The problem with burning fossil fuels is that
the supply is limited; so, the faster we expend
these resources, the sooner we will be left
without an energy source.
(B) Many people gain more satisfaction from
performing a job well—regardless of whether
they like the job—than from doing merely
adequately a job they like; thus, people who
want to be happy should choose jobs they can
do well.
(C) Some students who study thoroughly do well
in school. Thus, what is most important for
success in school is not how much time a
student puts into studying, but rather how
thoroughly the student studies.
(D) People do not fear change if they know what
the change will bring; so, our employees’ fear
stems not from our company’s undergoing
change, but from our failing to inform them
of what the changes entail.
(E) Until ten years ago, we had good soil and our
agriculture flourished. Therefore, the recent
decline of our agriculture is a result of our
soil rapidly eroding and there being nothing
that can replace the good soil we lost.

A

Correct Answer: D

D Parallel

The naturalist concludes that the threats we are creating to woodland species arise from the rate at which we are cutting down trees. Species can survive a change in environment as long as the change is slow enough. So, the change in environment caused by cutting down trees is not by itself causing the threats to woodland species.

A. No. This does not amend the cause of the problem.

B. No. The threat in the first still involved the cutting down of the trees. This argument discards one possibility for another, unrelated one.

C. No. This argument makes a claim about some students. The argument above makes a claim about all species.

D. Yes. This argument claims that the problem isn’t from the company’s undergoing change, per se, but from the failure to inform employees of what the changes entail.

E. No. This doesn’t present an alternative but related reason for the problem.

119
Q

Professor: A person who can select a beverage from
among 50 varieties of cola is less free than one
who has only these 5 choices: wine, coffee,
apple juice, milk, and water. It is clear, then,
that meaningful freedom cannot be measured
simply by the number of alternatives available;
the extent of the differences among the
alternatives is also a relevant factor.
The professor’s argument proceeds by
(A) supporting a general principle by means of an
example
(B) drawing a conclusion about a particular case
on the basis of a general principle
(C) supporting its conclusion by means of an
analogy
(D) claiming that whatever holds for each member
of a group must hold for the whole group
(E) inferring one general principle from another,
more general, principle

A

Correct Answer: A

A Reasoning

The professor argues that meaningful freedom cannot be measured simply by the number of alternatives available; rather, the extent of the differences among the alternatives is also a relevant factor. He supports his conclusion by the use of an example, in which he compares choosing one of 50 types of cola to choosing from among 5 different types of beverage—wine, coffee, apple juice, milk, and water.

A. Yes. The professor’s conclusion is a general principle and the cola versus milk, etc, example supports this principle.

B. No. The conclusion is the general principle.

C. No. There is no analogy. There is an example.

D. No. The professor’s argument does not use part-whole reasoning.

E. No. There is only one general principle.

120
Q

Principle: Meetings should be kept short, addressing
only those issues relevant to a majority of
those attending. A person should not be
required to attend a meeting if none of the
issues to be addressed at the meeting are
relevant to that person.
Application: Terry should not be required to attend
today’s two o’clock meeting.
Which one of the following, if true, most justifies the
stated application of the principle?
(A) The only issues on which Terry could make a
presentation at the meeting are issues
irrelevant to at least a majority of those who
could attend.
(B) If Terry makes a presentation at the meeting,
the meeting will not be kept short.
(C) No issue relevant to Terry could be relevant to
a majority of those attending the meeting.
(D) If Terry attends the meeting a different set of
issues will be relevant to a majority of those
attending than if Terry does not attend.
(E) The majority of the issues to be addressed at
the meeting are not relevant to Terry.

A

Correct Answer: C

C Strengthen

The principle claims that meetings should be kept short and should address only the issues that are relevant to the majority of the people attending. Moreover, people to whom none of the issues to be addressed are relevant should not have to attend the meeting. The application is that Terry should not be required to attend today’s meeting. The credited response will make it clear that none of the issues to be addressed are relevant to Terry, or that the issues that are relevant to Terry won’t be relevant to the majority of the attendees.

A. No. The principle doesn’t claim that people should come only if they are also presenting.

B. No. This doesn’t go far enough. The meetings should be kept short, in the sense that topics not relevant to the majority should not be discussed. But Terry wouldn’t necessarily have to make a presentation if he/she attended the meeting.

C. Yes. This supports the application of the principle because, if none of the issues relevant to Terry could be relevant to a majority of those attending the meeting, then the issues relevant to Terry will not be addressed at the meeting. So, Terry should not be required to attend.

D. No. This doesn’t make it clear that Terry shouldn’t have to go to the meeting. If any of the issues to be addressed are relevant to Terry, he/she might still have to go to the meeting.

E. No. The principle claims that if none of the issues to be addressed are relevant, then a person should not have to attend. If at least one of the issues to be addressed is relevant to Terry, even if the majority of them are not, Terry might still be required to attend.

121
Q

Asian American poetry from Hawaii, the Pacific
island state of the United States, is generally
characterizable in one of two ways: either as
portraying a model multicultural paradise, or as
exemplifying familiar Asian American literary themes
such as generational conflict. In this light, the recent
work of Wing Tek Lum in Expounding the Doubtful
Points is striking for its demand to be understood on
its own terms. Lum offers no romanticized notions of
multicultural life in Hawaii, and while he does
explore themes of family, identity, history, and
literary tradition, he does not do so at the expense of
attempting to discover and retain a local sensibility.
For Lum such a sensibility is informed by the fact
that Hawaii’s population, unlike that of the
continental U.S., has historically consisted
predominantly of people of Asian and Pacific island
descent, making the experience of its Asian
Americans somewhat different than that of mainland
Asian Americans.
In one poem, Lum meditates on the ways in
which a traditional Chinese lunar celebration he is
attending at a local beach both connects him to and
separates him from the past. In the company of new
Chinese immigrants, the speaker realizes that while
ties to the homeland are comforting and necessary, it
is equally important to have “a sense of new family”
in this new land of Hawaii, and hence a new
identity—one that is sensitive to its new environment.
The role of immigrants in this poem is significant in
that, through their presence, Lum is able to refer both
to the traditional culture of his ancestral homeland as
well as to the flux within Hawaiian society that has
been integral to its heterogeneity. Even in a laudatory
poem to famous Chinese poet Li Po (701–762 A.D.),
which partly serves to place Lum’s work within a
distinguished literary tradition, Lum refuses to offer a
stereotypical nostalgia for the past, instead pointing
out the often elitist tendencies inherent in the work of
some traditionally acclaimed Chinese poets.
Lum closes his volume with a poem that further
points to the complex relationships between heritage
and local culture in determining one’s identity.
Pulling together images and figures as vastly
disparate as a famous Chinese American literary
character and an old woman selling bread, Lum
avoids an excessively romantic vision of U.S. culture,
while simultaneously acknowledging the dream of
this culture held by many newly arrived immigrants. The central image of a communal pot where each
person chooses what she or he wishes to eat but
shares with others the “sweet soup / spooned out at
the end of the meal” is a hopeful one; however, it
also appears to caution that the strong cultural
emphasis in the U.S. on individual drive and success
that makes retaining a sense of homeland tradition
difficult should be identified and responded to in
ways that allow for a healthy new sense of identity to
be formed.

LSAT Preptest 53 Reading Comprehension Passage 1 Anaylysis.

A

The purpose of this passage is to Advocate/Defend. The author critiques the recent work of the Asian American poet Wing Tek Lum, claiming that his book, Expounding the Doubtful Points, demands to be understood on its own terms. This is in contrast to most Asian American poetry from Hawaii, which can usually be characterized either as portraying a model multicultural paradise or as exemplifying familiar Asian American themes like generational conflict. In the first paragraph, the author introduces the contrast between most Asian American poetry from Hawaii and the recent work of Wing Tek Lum. In the second paragraph, an example of Lum’s poetry is introduced in order to illustrate how, through the presence of immigrants, Lum is able to refer both to the traditional culture of his Chinese homeland and to the flux within Hawaiian society. A laudatory poem to a famous Chinese poet is introduced to illustrate Lum’s refusal to offer a stereotypical nostalgia for the past, while still participating in a distinguished literary tradition. In the third paragraph, the author discusses the final poem in Lum’s volume. This poem illustrates the complex relationship between heritage and local culture in determining one’s identity. In this poem, Lum acknowledges the hope that many immigrants have for their lives in the United States, while cautioning that immigrants should come to terms with the strong cultural emphasis in the United States on individual drive and success so as to form a healthy new sense of identity.

122
Q

Asian American poetry from Hawaii, the Pacific
island state of the United States, is generally
characterizable in one of two ways: either as
portraying a model multicultural paradise, or as
exemplifying familiar Asian American literary themes
such as generational conflict. In this light, the recent
work of Wing Tek Lum in Expounding the Doubtful
Points is striking for its demand to be understood on
its own terms. Lum offers no romanticized notions of
multicultural life in Hawaii, and while he does
explore themes of family, identity, history, and
literary tradition, he does not do so at the expense of
attempting to discover and retain a local sensibility.
For Lum such a sensibility is informed by the fact
that Hawaii’s population, unlike that of the
continental U.S., has historically consisted
predominantly of people of Asian and Pacific island
descent, making the experience of its Asian
Americans somewhat different than that of mainland
Asian Americans.
In one poem, Lum meditates on the ways in
which a traditional Chinese lunar celebration he is
attending at a local beach both connects him to and
separates him from the past. In the company of new
Chinese immigrants, the speaker realizes that while
ties to the homeland are comforting and necessary, it
is equally important to have “a sense of new family”
in this new land of Hawaii, and hence a new
identity—one that is sensitive to its new environment.
The role of immigrants in this poem is significant in
that, through their presence, Lum is able to refer both
to the traditional culture of his ancestral homeland as
well as to the flux within Hawaiian society that has
been integral to its heterogeneity. Even in a laudatory
poem to famous Chinese poet Li Po (701–762 A.D.),
which partly serves to place Lum’s work within a
distinguished literary tradition, Lum refuses to offer a
stereotypical nostalgia for the past, instead pointing
out the often elitist tendencies inherent in the work of
some traditionally acclaimed Chinese poets.
Lum closes his volume with a poem that further
points to the complex relationships between heritage
and local culture in determining one’s identity.
Pulling together images and figures as vastly
disparate as a famous Chinese American literary
character and an old woman selling bread, Lum
avoids an excessively romantic vision of U.S. culture,
while simultaneously acknowledging the dream of
this culture held by many newly arrived immigrants.
The central image of a communal pot where each
person chooses what she or he wishes to eat but
shares with others the “sweet soup / spooned out at
the end of the meal” is a hopeful one; however, it
also appears to caution that the strong cultural
emphasis in the U.S. on individual drive and success
that makes retaining a sense of homeland tradition
difficult should be identified and responded to in
ways that allow for a healthy new sense of identity to
be formed.

  1. Given the information in the passage, which one of
    the following is Lum most likely to believe?
    (A) Images in a poem should be explained in that
    poem so that their meaning will be widely
    understood.
    (B) The experience of living away from one’s
    homeland is necessary for developing a
    healthy perspective on one’s cultural
    traditions.
    (C) It is important to reconcile the values of
    individual achievement and enterprise with
    the desire to retain one’s cultural traditions.
    (D) One’s identity is continually in transition and
    poetry is a way of developing a static identity.
    (E) One cannot both seek a new identity and
    remain connected to one’s cultural traditions.
A

Correct Answer: C

Passage Analysis

C Extract Infer

A. No. In the examples cited by the author, Lum does not explain his images in great detail.

B. No. In the poem cited in the beginning of the second paragraph, Lum addresses how connections to the homeland are necessary but that one needs to have a new sense of family. However, this is only in reference to someone who has, in fact, immigrated. Lum never advocates immigration per se.

C. Yes. This is exactly what Lum is suggesting in the poem cited in the third paragraph.

D. No. Lum’s poetry attempts to discover and retain a local sensibility while keeping ties to the homeland. This illustrates a dynamic identity, not a static one.

E. No. Lum says that it’s necessary to keep ties to one’s homeland in the poem cited at the beginning of the second paragraph.

122
Q

Asian American poetry from Hawaii, the Pacific
island state of the United States, is generally
characterizable in one of two ways: either as
portraying a model multicultural paradise, or as
exemplifying familiar Asian American literary themes
such as generational conflict. In this light, the recent
work of Wing Tek Lum in Expounding the Doubtful
Points is striking for its demand to be understood on
its own terms. Lum offers no romanticized notions of
multicultural life in Hawaii, and while he does
explore themes of family, identity, history, and
literary tradition, he does not do so at the expense of
attempting to discover and retain a local sensibility.
For Lum such a sensibility is informed by the fact
that Hawaii’s population, unlike that of the
continental U.S., has historically consisted
predominantly of people of Asian and Pacific island
descent, making the experience of its Asian
Americans somewhat different than that of mainland
Asian Americans.
In one poem, Lum meditates on the ways in
which a traditional Chinese lunar celebration he is
attending at a local beach both connects him to and
separates him from the past. In the company of new
Chinese immigrants, the speaker realizes that while
ties to the homeland are comforting and necessary, it
is equally important to have “a sense of new family”
in this new land of Hawaii, and hence a new
identity—one that is sensitive to its new environment.
The role of immigrants in this poem is significant in
that, through their presence, Lum is able to refer both
to the traditional culture of his ancestral homeland as
well as to the flux within Hawaiian society that has
been integral to its heterogeneity. Even in a laudatory
poem to famous Chinese poet Li Po (701–762 A.D.),
which partly serves to place Lum’s work within a
distinguished literary tradition, Lum refuses to offer a
stereotypical nostalgia for the past, instead pointing
out the often elitist tendencies inherent in the work of
some traditionally acclaimed Chinese poets.
Lum closes his volume with a poem that further
points to the complex relationships between heritage
and local culture in determining one’s identity.
Pulling together images and figures as vastly
disparate as a famous Chinese American literary
character and an old woman selling bread, Lum
avoids an excessively romantic vision of U.S. culture,
while simultaneously acknowledging the dream of
this culture held by many newly arrived immigrants.
The central image of a communal pot where each
person chooses what she or he wishes to eat but
shares with others the “sweet soup / spooned out at
the end of the meal” is a hopeful one; however, it
also appears to caution that the strong cultural
emphasis in the U.S. on individual drive and success
that makes retaining a sense of homeland tradition
difficult should be identified and responded to in
ways that allow for a healthy new sense of identity to
be formed.

Which one of the following most accurately expresses
the main point of the passage?
(A) The poetry of Lum departs from other Asian
American poetry from Hawaii in that it
acknowledges its author’s heritage but also
expresses the poet’s search for a new local
identity.
(B) Lum’s poetry is in part an expression of the
conflict between a desire to participate in a
community with shared traditions and values
and a desire for individual success.
(C) Lum writes poetry that not only rejects
features of the older literary tradition in
which he participates but also rejects the
popular literary traditions of Hawaiian
writers.
(D) The poetry of Lum illustrates the extent to
which Asian American writers living in
Hawaii have a different cultural perspective
than those living in the continental U.S.
(E) Lum’s poetry is an unsuccessful attempt to
manage the psychological burdens of
reconciling a sense of tradition with a healthy
sense of individual identity.

A

Correct Answer: A

Passage Analysis

A Big Picture

A. Yes. The passage highlights how Lum’s poetry is striking in its departure from other Asian American poetry from Hawaii. Lum does address his own heritage but combines that with a search for a new local identity.

B. No. Individual success is addressed only at the end of the last paragraph and only in reference to a caution that Lum raises for immigrants.

C. No. The passage does not discuss Hawaiian writers in general, only Asian American Hawaiian writers.

D. No. The poetry of Lum is different from that of other Asian American writers in Hawaii, so it can’t illustrate something about Asian American writers in Hawaii as a whole.

E. No. The author is admiring of Lum’s poetry, so the author would not say that Lum’s poetry is unsuccessful.

123
Q

Asian American poetry from Hawaii, the Pacific
island state of the United States, is generally
characterizable in one of two ways: either as
portraying a model multicultural paradise, or as
exemplifying familiar Asian American literary themes
such as generational conflict. In this light, the recent
work of Wing Tek Lum in Expounding the Doubtful
Points is striking for its demand to be understood on
its own terms. Lum offers no romanticized notions of
multicultural life in Hawaii, and while he does
explore themes of family, identity, history, and
literary tradition, he does not do so at the expense of
attempting to discover and retain a local sensibility.
For Lum such a sensibility is informed by the fact
that Hawaii’s population, unlike that of the
continental U.S., has historically consisted
predominantly of people of Asian and Pacific island
descent, making the experience of its Asian
Americans somewhat different than that of mainland
Asian Americans.
In one poem, Lum meditates on the ways in
which a traditional Chinese lunar celebration he is
attending at a local beach both connects him to and
separates him from the past. In the company of new
Chinese immigrants, the speaker realizes that while
ties to the homeland are comforting and necessary, it
is equally important to have “a sense of new family”
in this new land of Hawaii, and hence a new
identity—one that is sensitive to its new environment.
The role of immigrants in this poem is significant in
that, through their presence, Lum is able to refer both
to the traditional culture of his ancestral homeland as
well as to the flux within Hawaiian society that has
been integral to its heterogeneity. Even in a laudatory
poem to famous Chinese poet Li Po (701–762 A.D.),
which partly serves to place Lum’s work within a
distinguished literary tradition, Lum refuses to offer a
stereotypical nostalgia for the past, instead pointing
out the often elitist tendencies inherent in the work of
some traditionally acclaimed Chinese poets.
Lum closes his volume with a poem that further
points to the complex relationships between heritage
and local culture in determining one’s identity.
Pulling together images and figures as vastly
disparate as a famous Chinese American literary
character and an old woman selling bread, Lum
avoids an excessively romantic vision of U.S. culture,
while simultaneously acknowledging the dream of
this culture held by many newly arrived immigrants.
The central image of a communal pot where each
person chooses what she or he wishes to eat but
shares with others the “sweet soup / spooned out at
the end of the meal” is a hopeful one; however, it
also appears to caution that the strong cultural
emphasis in the U.S. on individual drive and success
that makes retaining a sense of homeland tradition
difficult should be identified and responded to in
ways that allow for a healthy new sense of identity to
be formed.

According to the passage, some Asian American
literature from Hawaii has been characterized as
which one of the following?
(A) inimical to the process of developing a local
sensibility
(B) centered on the individual’s drive to succeed
(C) concerned with conflicts between different age
groups
(D) focused primarily on retaining ties to one’s
homeland
(E) tied to a search for a new sense of family in a
new land

A

Correct Answer: C

Passage Analysis

C Extract Fact

A. No. This is too strong. It is not totally opposed to or against the process of developing a local sensibility.

B. No. This was a part of the United States’ strong cultural emphasis, not a characteristic of Asian American literature from Hawaii.

C. Yes. This was cited as a characteristic of one of two types of Asian American literature from Hawaii in the first paragraph.

D. No. This is not discussed in reference to Asian American literature from Hawaii.

E. No. This is true of Lum’s poetry, which is strikingly different from Asian American literature from Hawaii.

124
Q

Asian American poetry from Hawaii, the Pacific
island state of the United States, is generally
characterizable in one of two ways: either as
portraying a model multicultural paradise, or as
exemplifying familiar Asian American literary themes
such as generational conflict. In this light, the recent
work of Wing Tek Lum in Expounding the Doubtful
Points is striking for its demand to be understood on
its own terms. Lum offers no romanticized notions of
multicultural life in Hawaii, and while he does
explore themes of family, identity, history, and
literary tradition, he does not do so at the expense of
attempting to discover and retain a local sensibility.
For Lum such a sensibility is informed by the fact
that Hawaii’s population, unlike that of the
continental U.S., has historically consisted
predominantly of people of Asian and Pacific island
descent, making the experience of its Asian
Americans somewhat different than that of mainland
Asian Americans.
In one poem, Lum meditates on the ways in
which a traditional Chinese lunar celebration he is
attending at a local beach both connects him to and
separates him from the past. In the company of new
Chinese immigrants, the speaker realizes that while
ties to the homeland are comforting and necessary, it
is equally important to have “a sense of new family”
in this new land of Hawaii, and hence a new
identity—one that is sensitive to its new environment.
The role of immigrants in this poem is significant in
that, through their presence, Lum is able to refer both
to the traditional culture of his ancestral homeland as
well as to the flux within Hawaiian society that has
been integral to its heterogeneity. Even in a laudatory
poem to famous Chinese poet Li Po (701–762 A.D.),
which partly serves to place Lum’s work within a
distinguished literary tradition, Lum refuses to offer a
stereotypical nostalgia for the past, instead pointing
out the often elitist tendencies inherent in the work of
some traditionally acclaimed Chinese poets.
Lum closes his volume with a poem that further
points to the complex relationships between heritage
and local culture in determining one’s identity.
Pulling together images and figures as vastly
disparate as a famous Chinese American literary
character and an old woman selling bread, Lum
avoids an excessively romantic vision of U.S. culture,
while simultaneously acknowledging the dream of
this culture held by many newly arrived immigrants.
The central image of a communal pot where each
person chooses what she or he wishes to eat but
shares with others the “sweet soup / spooned out at
the end of the meal” is a hopeful one; however, it
also appears to caution that the strong cultural
emphasis in the U.S. on individual drive and success
that makes retaining a sense of homeland tradition
difficult should be identified and responded to in
ways that allow for a healthy new sense of identity to
be formed.

The author of the passage uses the phrase “the flux
within Hawaiian society” (line 33) primarily in order to
(A) describe the social tension created by the mix
of attitudes exhibited by citizens of Hawaii
(B) deny that Hawaiian society is culturally
distinct from that of the continental U.S.
(C) identify the process by which immigrants learn
to adapt to their new communities
(D) refer to the constant change to which the
culture in Hawaii is subject due to its diverse
population
(E) emphasize the changing attitudes of many
immigrants to Hawaii toward their traditional
cultural norms

A

Correct Answer: D

Passage Analysis

D Structure

The author uses the phrase “the flux within Hawaiian society” in order to highlight the continuous changes in that society brought about by the influx of immigrants, which supports (D).

125
Q

Asian American poetry from Hawaii, the Pacific
island state of the United States, is generally
characterizable in one of two ways: either as
portraying a model multicultural paradise, or as
exemplifying familiar Asian American literary themes
such as generational conflict. In this light, the recent
work of Wing Tek Lum in Expounding the Doubtful
Points is striking for its demand to be understood on
its own terms. Lum offers no romanticized notions of
multicultural life in Hawaii, and while he does
explore themes of family, identity, history, and
literary tradition, he does not do so at the expense of
attempting to discover and retain a local sensibility.
For Lum such a sensibility is informed by the fact
that Hawaii’s population, unlike that of the
continental U.S., has historically consisted
predominantly of people of Asian and Pacific island
descent, making the experience of its Asian
Americans somewhat different than that of mainland
Asian Americans.
In one poem, Lum meditates on the ways in
which a traditional Chinese lunar celebration he is
attending at a local beach both connects him to and
separates him from the past. In the company of new
Chinese immigrants, the speaker realizes that while
ties to the homeland are comforting and necessary, it
is equally important to have “a sense of new family”
in this new land of Hawaii, and hence a new
identity—one that is sensitive to its new environment.
The role of immigrants in this poem is significant in
that, through their presence, Lum is able to refer both
to the traditional culture of his ancestral homeland as
well as to the flux within Hawaiian society that has
been integral to its heterogeneity. Even in a laudatory
poem to famous Chinese poet Li Po (701–762 A.D.),
which partly serves to place Lum’s work within a
distinguished literary tradition, Lum refuses to offer a
stereotypical nostalgia for the past, instead pointing
out the often elitist tendencies inherent in the work of
some traditionally acclaimed Chinese poets.
Lum closes his volume with a poem that further
points to the complex relationships between heritage
and local culture in determining one’s identity.
Pulling together images and figures as vastly
disparate as a famous Chinese American literary
character and an old woman selling bread, Lum
avoids an excessively romantic vision of U.S. culture,
while simultaneously acknowledging the dream of
this culture held by many newly arrived immigrants.
The central image of a communal pot where each
person chooses what she or he wishes to eat but
shares with others the “sweet soup / spooned out at
the end of the meal” is a hopeful one; however, it
also appears to caution that the strong cultural
emphasis in the U.S. on individual drive and success
that makes retaining a sense of homeland tradition
difficult should be identified and responded to in
ways that allow for a healthy new sense of identity to
be formed.

The author of the passage describes Expounding the
Doubtful Points as “striking” (lines 7–8) primarily in
order to
(A) underscore the forceful and contentious tone
of the work
(B) indicate that the work has not been properly
analyzed by literary critics
(C) stress the radical difference between this work
and Lum’s earlier work
(D) emphasize the differences between this work
and that of other Asian American poets from
Hawaii
(E) highlight the innovative nature of Lum’s
experiments with poetic form

A

Correct Answer: D

Passage Analysis

D Structure

The author describes Expounding the Doubtful Points as striking in order to emphasize how different Lum’s recent work is from the usual Asian American poetry from Hawaii, which supports (D).

126
Q

Asian American poetry from Hawaii, the Pacific
island state of the United States, is generally
characterizable in one of two ways: either as
portraying a model multicultural paradise, or as
exemplifying familiar Asian American literary themes
such as generational conflict. In this light, the recent
work of Wing Tek Lum in Expounding the Doubtful
Points is striking for its demand to be understood on
its own terms. Lum offers no romanticized notions of
multicultural life in Hawaii, and while he does
explore themes of family, identity, history, and
literary tradition, he does not do so at the expense of
attempting to discover and retain a local sensibility.
For Lum such a sensibility is informed by the fact
that Hawaii’s population, unlike that of the
continental U.S., has historically consisted
predominantly of people of Asian and Pacific island
descent, making the experience of its Asian
Americans somewhat different than that of mainland
Asian Americans.
In one poem, Lum meditates on the ways in
which a traditional Chinese lunar celebration he is
attending at a local beach both connects him to and
separates him from the past. In the company of new
Chinese immigrants, the speaker realizes that while
ties to the homeland are comforting and necessary, it
is equally important to have “a sense of new family”
in this new land of Hawaii, and hence a new
identity—one that is sensitive to its new environment.
The role of immigrants in this poem is significant in
that, through their presence, Lum is able to refer both
to the traditional culture of his ancestral homeland as
well as to the flux within Hawaiian society that has
been integral to its heterogeneity. Even in a laudatory
poem to famous Chinese poet Li Po (701–762 A.D.),
which partly serves to place Lum’s work within a
distinguished literary tradition, Lum refuses to offer a
stereotypical nostalgia for the past, instead pointing
out the often elitist tendencies inherent in the work of
some traditionally acclaimed Chinese poets.
Lum closes his volume with a poem that further
points to the complex relationships between heritage
and local culture in determining one’s identity.
Pulling together images and figures as vastly
disparate as a famous Chinese American literary
character and an old woman selling bread, Lum
avoids an excessively romantic vision of U.S. culture,
while simultaneously acknowledging the dream of
this culture held by many newly arrived immigrants.
The central image of a communal pot where each
person chooses what she or he wishes to eat but
shares with others the “sweet soup / spooned out at
the end of the meal” is a hopeful one; however, it
also appears to caution that the strong cultural
emphasis in the U.S. on individual drive and success
that makes retaining a sense of homeland tradition
difficult should be identified and responded to in
ways that allow for a healthy new sense of identity to
be formed.

With which one of the following statements
regarding Lum’s poetry would the author of the
passage be most likely to agree?
(A) It cannot be used to support any specific
political ideology.
(B) It is an elegant demonstration of the poet’s
appreciation of the stylistic contributions of
his literary forebears.
(C) It is most fruitfully understood as a meditation
on the choice between new and old that
confronts any human being in any culture.
(D) It conveys thoughtful assessments of both his
ancestral homeland tradition and the culture
in which he is attempting to build a new
identity.
(E) It conveys Lum’s antipathy toward tradition by
juxtaposing traditional and nontraditional
images.

A

Correct Answer: D

Passage Analysis

D Extract Infer

A. No. The author does not discuss political ideology at all, so we don’t know whether the author would agree with this claim.

B. No. The author does not discuss whether Lum’s poetry exhibits appreciation of stylistic contributions, so we don’t know whether the author would agree with this claim.

C. No. The culture discussed is exclusively that of Asian Americans in Hawaii, so we don’t know whether the author would agree with this claim about people in all cultures.

D. Yes. The author cites this in the discussion of the example in the first part of the second paragraph.

E. No. The author discusses the fact that Lum recognizes the value of tradition, so the author would not claim that Lum’s poetry conveys antipathy toward tradition.

127
Q

In England the burden of history weighs heavily
on common law, that unwritten code of time-honored
laws derived largely from English judicial custom and
precedent. Students of contemporary British law are
frequently required to study medieval cases, to
interpret archaic Latin maxims, or to confront
doctrinal principles whose validity is based solely on
their being part of the “timeless reason” of the
English legal tradition. Centuries-old custom serves as
the basis both for the divisions of law school subject
matter and for much of the terminology of legal
redress. Connected not only with legal history but
also with the cultural history of the English people,
common law cannot properly be understood without
taking a long historical view.
Yet the academic study of jurisprudence has
seldom treated common law as a constantly evolving
phenomenon rooted in history; those interpretive
theories that do acknowledge the antiquity of
common law ignore the practical contemporary
significance of its historical forms. The reasons for
this omission are partly theoretical and partly
political. In theoretical terms, modern jurisprudence
has consistently treated law as a unified system of
rules that can be studied at any given moment in time
as a logical whole. The notion of jurisprudence as a
system of norms or principles deemphasizes history
in favor of the coherence of a system. In this view,
the past of the system is conceived as no more than
the continuous succession of its states of presence. In
political terms, believing in the logic of law is a
necessary part of believing in its fairness; even if
history shows the legal tradition to be far from
unitary and seldom logical, the prestige of the legal
institution requires that jurisprudence treat the
tradition as if it were, in essence, the application of
known rules to objectively determined facts. To
suggest otherwise would be dispiriting for the student
and demoralizing for the public.
Legal historian Peter Goodrich has argued,
however, that common law is most fruitfully studied
as a continually developing tradition rather than as a
set of rules. Taking his cue from the study of
literature, Goodrich sees common law as a sort of
literary text, with history and tradition serving as the
text’s narrative development. To study the common
law historically, says Goodrich, is to study a text in
which fiction is as influential as analysis, perception
as significant as rule, and the play of memory as
strong as the logic of argument. The concept of
tradition, for Goodrich, implies not only the
preservation and transmission of existing forms, but
also the continuous rewriting of those forms to adapt
them to contemporary legal circumstances.

LSAT Preptest 53 Reading Comprehension Passage 2 Analysis.

A

The purpose of this passage is to Criticize. It concerns the problems with traditional academic study of jurisprudence in England and one legal historian’s argument for a change in how jurisprudence is studied. In the first paragraph, the author introduces the topic of common law, which is the basis of the English legal system, and suggests that common law must be understood in a historical context in order to be understood properly. In the second paragraph, the author introduces how academic study of jurisprudence has traditionally regarded common law as static, for the most part, and criticizes this view. The author then gives the reasons for this view of common law, which are partly political and partly theoretical, noting that the political reasons for this view of common law are to prevent the demoralization of the public and the dispiriting of students of the law. In the third paragraph, the author introduces another way to regard common law, attributed to legal historian Peter Goodrich. He argues that common law is most fruitfully studied as a continually developing tradition instead of as a set of rules.

128
Q

In England the burden of history weighs heavily
on common law, that unwritten code of time-honored
laws derived largely from English judicial custom and
precedent. Students of contemporary British law are
frequently required to study medieval cases, to
interpret archaic Latin maxims, or to confront
doctrinal principles whose validity is based solely on
their being part of the “timeless reason” of the
English legal tradition. Centuries-old custom serves as
the basis both for the divisions of law school subject
matter and for much of the terminology of legal
redress. Connected not only with legal history but
also with the cultural history of the English people,
common law cannot properly be understood without
taking a long historical view.
Yet the academic study of jurisprudence has
seldom treated common law as a constantly evolving
phenomenon rooted in history; those interpretive
theories that do acknowledge the antiquity of
common law ignore the practical contemporary
significance of its historical forms. The reasons for
this omission are partly theoretical and partly
political. In theoretical terms, modern jurisprudence
has consistently treated law as a unified system of
rules that can be studied at any given moment in time
as a logical whole. The notion of jurisprudence as a
system of norms or principles deemphasizes history
in favor of the coherence of a system. In this view,
the past of the system is conceived as no more than
the continuous succession of its states of presence. In
political terms, believing in the logic of law is a
necessary part of believing in its fairness; even if
history shows the legal tradition to be far from
unitary and seldom logical, the prestige of the legal
institution requires that jurisprudence treat the
tradition as if it were, in essence, the application of
known rules to objectively determined facts. To
suggest otherwise would be dispiriting for the student
and demoralizing for the public.
Legal historian Peter Goodrich has argued,
however, that common law is most fruitfully studied
as a continually developing tradition rather than as a
set of rules. Taking his cue from the study of
literature, Goodrich sees common law as a sort of
literary text, with history and tradition serving as the
text’s narrative development. To study the common
law historically, says Goodrich, is to study a text in
which fiction is as influential as analysis, perception
as significant as rule, and the play of memory as
strong as the logic of argument. The concept of
tradition, for Goodrich, implies not only the
preservation and transmission of existing forms, but
also the continuous rewriting of those forms to adapt
them to contemporary legal circumstances.

  1. Which one of the following statements best expresses
    the main idea of the passage?
    (A) The residual influences of common law explain
    not only the divisions of subject matter but
    also the terminology associated with many
    legal procedures.
    (B) In the academic study of jurisprudence,
    theoretical interpretations of common law
    have traditionally been at odds with political
    interpretations of common law.
    (C) Common law, while often treated as an oral
    history of the English people, would,
    according to one scholar, be more fruitfully
    studied as a universally adaptable and
    constantly changing system of rules.
    (D) Although obviously steeped in history and
    tradition, common law has seldom been
    studied in relation to its development, as one
    theorist proposes that it be understood.
    (E) Although usually studied as a unitary and
    logical system of rules and norms, the history
    of common law shows that body of law to be
    anything but consistent and fair.
A

Correct Answer: D

Passage Analysis

D Big Picture

A. No. This is too narrow. This is discussed only in the first paragraph.

B. No. The passage claims that both theoretical and political interpretations of common law are, if anything, somewhat similar.

C. No. The academic study of jurisprudence does not treat common law as an oral history of the English people.

D. Yes. This identifies the problem that the author sees with the study of common law, discussed in the first two paragraphs, and mentions the alternative way to regard common law, discussed in the third paragraph.

E. No. This is too narrow and too strong. This topic is discussed only around lines 30–35 and the author never claims that the body of law is actually inconsistent and unfair.

129
Q

In England the burden of history weighs heavily
on common law, that unwritten code of time-honored
laws derived largely from English judicial custom and
precedent. Students of contemporary British law are
frequently required to study medieval cases, to
interpret archaic Latin maxims, or to confront
doctrinal principles whose validity is based solely on
their being part of the “timeless reason” of the
English legal tradition. Centuries-old custom serves as
the basis both for the divisions of law school subject
matter and for much of the terminology of legal
redress. Connected not only with legal history but
also with the cultural history of the English people,
common law cannot properly be understood without
taking a long historical view.
Yet the academic study of jurisprudence has
seldom treated common law as a constantly evolving
phenomenon rooted in history; those interpretive
theories that do acknowledge the antiquity of
common law ignore the practical contemporary
significance of its historical forms. The reasons for
this omission are partly theoretical and partly
political. In theoretical terms, modern jurisprudence
has consistently treated law as a unified system of
rules that can be studied at any given moment in time
as a logical whole. The notion of jurisprudence as a
system of norms or principles deemphasizes history
in favor of the coherence of a system. In this view,
the past of the system is conceived as no more than
the continuous succession of its states of presence. In
political terms, believing in the logic of law is a
necessary part of believing in its fairness; even if
history shows the legal tradition to be far from
unitary and seldom logical, the prestige of the legal
institution requires that jurisprudence treat the
tradition as if it were, in essence, the application of
known rules to objectively determined facts. To
suggest otherwise would be dispiriting for the student
and demoralizing for the public.
Legal historian Peter Goodrich has argued,
however, that common law is most fruitfully studied
as a continually developing tradition rather than as a
set of rules. Taking his cue from the study of
literature, Goodrich sees common law as a sort of
literary text, with history and tradition serving as the
text’s narrative development. To study the common
law historically, says Goodrich, is to study a text in
which fiction is as influential as analysis, perception
as significant as rule, and the play of memory as
strong as the logic of argument. The concept of
tradition, for Goodrich, implies not only the
preservation and transmission of existing forms, but
also the continuous rewriting of those forms to adapt
them to contemporary legal circumstances.

It can be inferred that the author of the passage
believes which one of the following about the history
of law in relation to modern jurisprudence?
(A) Modern jurisprudence misinterprets the
nature of the legal tradition.
(B) The history of law proves the original forms of
common law to be antiquated and irrelevant
to modern jurisprudence.
(C) The history of law, if it is to be made
applicable to modern jurisprudence, is best
studied as a system of rules rather than as a
literary text.
(D) Mainstream theories of modern jurisprudence
overlook the order and coherence inherent in
legal history.
(E) Mainstream theories of modern jurisprudence,
by and large devoid of a sense of legal history,
are unnecessarily dispiriting to students and
the public alike

A

Correct Answer: A

Passage Analysis

A Extract Infer

The author discusses this topic in the second paragraph.

A. Yes. The author criticizes modern jurisprudence’s treatment of common law, as it does not acknowledge its history and how that might affect contemporary law.

B. No. The author does not agree that the original forms of common law are irrelevant to modern jurisprudence.

C. No. This is how the academic study of modern jurisprudence views common law, a view with which the author disagrees.

D. No. It’s not that they overlook the order and coherence inherent in legal history; it’s that they overlook legal history too much.

E. No. This is the opposite of what the author claims at the end of the second paragraph.

130
Q

In England the burden of history weighs heavily
on common law, that unwritten code of time-honored
laws derived largely from English judicial custom and
precedent. Students of contemporary British law are
frequently required to study medieval cases, to
interpret archaic Latin maxims, or to confront
doctrinal principles whose validity is based solely on
their being part of the “timeless reason” of the
English legal tradition. Centuries-old custom serves as
the basis both for the divisions of law school subject
matter and for much of the terminology of legal
redress. Connected not only with legal history but
also with the cultural history of the English people,
common law cannot properly be understood without
taking a long historical view.
Yet the academic study of jurisprudence has
seldom treated common law as a constantly evolving
phenomenon rooted in history; those interpretive
theories that do acknowledge the antiquity of
common law ignore the practical contemporary
significance of its historical forms. The reasons for
this omission are partly theoretical and partly
political. In theoretical terms, modern jurisprudence
has consistently treated law as a unified system of
rules that can be studied at any given moment in time
as a logical whole. The notion of jurisprudence as a
system of norms or principles deemphasizes history
in favor of the coherence of a system. In this view,
the past of the system is conceived as no more than
the continuous succession of its states of presence. In
political terms, believing in the logic of law is a
necessary part of believing in its fairness; even if
history shows the legal tradition to be far from
unitary and seldom logical, the prestige of the legal
institution requires that jurisprudence treat the
tradition as if it were, in essence, the application of
known rules to objectively determined facts. To
suggest otherwise would be dispiriting for the student
and demoralizing for the public.
Legal historian Peter Goodrich has argued,
however, that common law is most fruitfully studied
as a continually developing tradition rather than as a
set of rules. Taking his cue from the study of
literature, Goodrich sees common law as a sort of
literary text, with history and tradition serving as the
text’s narrative development. To study the common
law historically, says Goodrich, is to study a text in
which fiction is as influential as analysis, perception
as significant as rule, and the play of memory as
strong as the logic of argument. The concept of
tradition, for Goodrich, implies not only the
preservation and transmission of existing forms, but
also the continuous rewriting of those forms to adapt
them to contemporary legal circumstances.

  1. Which one of the following would best exemplify the
    kind of interpretive theory referred to in the first
    sentence of the second paragraph of the passage?
    (A) a theory that traced modern customs involving
    property ownership to their origins in
    medieval practice
    (B) a theory that relied on a comparison between
    modern courtroom procedures and medieval
    theatrical conventions
    (C) a theory that analyzed medieval marriage laws
    without examining their relationship to
    modern laws
    (D) a theory that compared the development of
    English common law in the twentieth century
    with simultaneous developments in German
    common law without examining the social
    repercussions of either legal system
    (E) a theory that compared rules of evidence in
    civil courts with those in criminal courts
A

Correct Answer: C

Passage Analysis

C RC Reasoning

This kind of interpretive theory is one in which common law is either not seen as evolving or rooted in history or, if it is seen as rooted in history, is not seen as making a significant impact on contemporary law.

A. No. This connects modern law with the past, against the interpretive theory discussed at the beginning of the second paragraph.

B. No. This connects modern law with the past, against this interpretive theory.

C. Yes. This acknowledges the history of common law without addressing how it might impact contemporary law.

D. No. This discusses developments that occurred simultaneously, so it isn’t relevant to the interpretive theory as stated.

E. No. The theory doesn’t discuss civil versus criminal courts.

131
Q

In England the burden of history weighs heavily
on common law, that unwritten code of time-honored
laws derived largely from English judicial custom and
precedent. Students of contemporary British law are
frequently required to study medieval cases, to
interpret archaic Latin maxims, or to confront
doctrinal principles whose validity is based solely on
their being part of the “timeless reason” of the
English legal tradition. Centuries-old custom serves as
the basis both for the divisions of law school subject
matter and for much of the terminology of legal
redress. Connected not only with legal history but
also with the cultural history of the English people,
common law cannot properly be understood without
taking a long historical view.
Yet the academic study of jurisprudence has
seldom treated common law as a constantly evolving
phenomenon rooted in history; those interpretive
theories that do acknowledge the antiquity of
common law ignore the practical contemporary
significance of its historical forms. The reasons for
this omission are partly theoretical and partly
political. In theoretical terms, modern jurisprudence
has consistently treated law as a unified system of
rules that can be studied at any given moment in time
as a logical whole. The notion of jurisprudence as a
system of norms or principles deemphasizes history
in favor of the coherence of a system. In this view,
the past of the system is conceived as no more than
the continuous succession of its states of presence. In
political terms, believing in the logic of law is a
necessary part of believing in its fairness; even if
history shows the legal tradition to be far from
unitary and seldom logical, the prestige of the legal
institution requires that jurisprudence treat the
tradition as if it were, in essence, the application of
known rules to objectively determined facts. To
suggest otherwise would be dispiriting for the student
and demoralizing for the public.
Legal historian Peter Goodrich has argued,
however, that common law is most fruitfully studied
as a continually developing tradition rather than as a
set of rules. Taking his cue from the study of
literature, Goodrich sees common law as a sort of
literary text, with history and tradition serving as the
text’s narrative development. To study the common
law historically, says Goodrich, is to study a text in
which fiction is as influential as analysis, perception
as significant as rule, and the play of memory as
strong as the logic of argument. The concept of
tradition, for Goodrich, implies not only the
preservation and transmission of existing forms, but
also the continuous rewriting of those forms to adapt
them to contemporary legal circumstances.

  1. Which one of the following best defines the word
    “political” as it is used in the second paragraph of the
    passage?
    (A) concerned with the ways by which people seek
    to advance themselves in a profession
    (B) concerned with the covert and possibly
    unethical methods by which governments
    achieve their goals
    (C) having to do with the maintenance of ethical
    standards between professions and the
    citizenry
    (D) having to do with the maintenance of an
    institution’s effectiveness
    (E) having to do with the manner in which
    institutions are perceived by radical theorists
A

Correct Answer: D

Passage Analysis

D Structure

The word “political” is used in the second paragraph to discuss the ways in which modern jurisprudence treats common law in order to maintain the efficacy and prestige of the legal system, which supports (D).

132
Q

In England the burden of history weighs heavily
on common law, that unwritten code of time-honored
laws derived largely from English judicial custom and
precedent. Students of contemporary British law are
frequently required to study medieval cases, to
interpret archaic Latin maxims, or to confront
doctrinal principles whose validity is based solely on
their being part of the “timeless reason” of the
English legal tradition. Centuries-old custom serves as
the basis both for the divisions of law school subject
matter and for much of the terminology of legal
redress. Connected not only with legal history but
also with the cultural history of the English people,
common law cannot properly be understood without
taking a long historical view.
Yet the academic study of jurisprudence has
seldom treated common law as a constantly evolving
phenomenon rooted in history; those interpretive
theories that do acknowledge the antiquity of
common law ignore the practical contemporary
significance of its historical forms. The reasons for
this omission are partly theoretical and partly
political. In theoretical terms, modern jurisprudence
has consistently treated law as a unified system of
rules that can be studied at any given moment in time
as a logical whole. The notion of jurisprudence as a
system of norms or principles deemphasizes history
in favor of the coherence of a system. In this view,
the past of the system is conceived as no more than
the continuous succession of its states of presence. In
political terms, believing in the logic of law is a
necessary part of believing in its fairness; even if
history shows the legal tradition to be far from
unitary and seldom logical, the prestige of the legal
institution requires that jurisprudence treat the
tradition as if it were, in essence, the application of
known rules to objectively determined facts. To
suggest otherwise would be dispiriting for the student
and demoralizing for the public.
Legal historian Peter Goodrich has argued,
however, that common law is most fruitfully studied
as a continually developing tradition rather than as a
set of rules. Taking his cue from the study of
literature, Goodrich sees common law as a sort of
literary text, with history and tradition serving as the
text’s narrative development. To study the common
law historically, says Goodrich, is to study a text in
which fiction is as influential as analysis, perception
as significant as rule, and the play of memory as
strong as the logic of argument. The concept of
tradition, for Goodrich, implies not only the
preservation and transmission of existing forms, but
also the continuous rewriting of those forms to adapt
them to contemporary legal circumstances.

  1. It can be inferred from the passage that Peter Goodrich
    would be most likely to agree with which one of the
    following statements concerning common law?
    (A) Common law is more fruitfully studied as a
    relic of the history of the English people than
    as a legal code.
    (B) The “text” of common law has degenerated
    from an early stage of clarity to a current
    state of incoherence.
    (C) Without the public’s belief in the justness of
    common law, the legal system cannot be
    perpetuated.
    (D) While rich in literary significance, the “text” of
    common law has only a very limited
    applicability to modern life.
    (E) The common law “text” inherited by future
    generations will differ from the one currently
    in use.
A

Correct Answer: E

Passage Analysis

E Extract Infer

Peter Goodrich’s theory is discussed in the third paragraph.

A. No. He claims that it should be seen as a continually developing tradition, not a relic.

B. No. He never claims that it is now incoherent.

C. No. The public’s beliefs concerning common law are discussed in the second paragraph.

D. No. He believes that it has applicability to modern life. This suggests that it has only a very limited applicability.

E. Yes. He argues that it is a continually developing tradition and that we have to rewrite some of it to adapt common law to contemporary legal circumstances.

133
Q

In England the burden of history weighs heavily
on common law, that unwritten code of time-honored
laws derived largely from English judicial custom and
precedent. Students of contemporary British law are
frequently required to study medieval cases, to
interpret archaic Latin maxims, or to confront
doctrinal principles whose validity is based solely on
their being part of the “timeless reason” of the
English legal tradition. Centuries-old custom serves as
the basis both for the divisions of law school subject
matter and for much of the terminology of legal
redress. Connected not only with legal history but
also with the cultural history of the English people,
common law cannot properly be understood without
taking a long historical view.
Yet the academic study of jurisprudence has
seldom treated common law as a constantly evolving
phenomenon rooted in history; those interpretive
theories that do acknowledge the antiquity of
common law ignore the practical contemporary
significance of its historical forms. The reasons for
this omission are partly theoretical and partly
political. In theoretical terms, modern jurisprudence
has consistently treated law as a unified system of
rules that can be studied at any given moment in time
as a logical whole. The notion of jurisprudence as a
system of norms or principles deemphasizes history
in favor of the coherence of a system. In this view,
the past of the system is conceived as no more than
the continuous succession of its states of presence. In
political terms, believing in the logic of law is a
necessary part of believing in its fairness; even if
history shows the legal tradition to be far from
unitary and seldom logical, the prestige of the legal
institution requires that jurisprudence treat the
tradition as if it were, in essence, the application of
known rules to objectively determined facts. To
suggest otherwise would be dispiriting for the student
and demoralizing for the public.
Legal historian Peter Goodrich has argued,
however, that common law is most fruitfully studied
as a continually developing tradition rather than as a
set of rules. Taking his cue from the study of
literature, Goodrich sees common law as a sort of
literary text, with history and tradition serving as the
text’s narrative development. To study the common
law historically, says Goodrich, is to study a text in
which fiction is as influential as analysis, perception
as significant as rule, and the play of memory as
strong as the logic of argument. The concept of
tradition, for Goodrich, implies not only the
preservation and transmission of existing forms, but
also the continuous rewriting of those forms to adapt
them to contemporary legal circumstances.

  1. The passage states that students of British law are
    frequently required to study
    (A) histories of English politics
    (B) episodes of litigation from the Middle Ages
    (C) treatises on political philosophy
    (D) histories of ancient Roman jurisprudence
    (E) essays on narrative development
A

Correct Answer: B

Passage Analysis

B Extract Fact

The first paragraph of the passage discusses what students of British law are frequently required to study.

A. No. This is not mentioned in the first paragraph.

B. Yes. The passage mentions this in line 5.

C. No. Political philosophy is not mentioned in the first paragraph.

D. No. While the first paragraph does mention archaic Latin maxims, there is no reference anywhere in the passage to histories of ancient Roman jurisprudence.

E. No. Narrative development is mentioned, in a different context, in the final paragraph.

133
Q

In England the burden of history weighs heavily
on common law, that unwritten code of time-honored
laws derived largely from English judicial custom and
precedent. Students of contemporary British law are
frequently required to study medieval cases, to
interpret archaic Latin maxims, or to confront
doctrinal principles whose validity is based solely on
their being part of the “timeless reason” of the
English legal tradition. Centuries-old custom serves as
the basis both for the divisions of law school subject
matter and for much of the terminology of legal
redress. Connected not only with legal history but
also with the cultural history of the English people,
common law cannot properly be understood without
taking a long historical view.
Yet the academic study of jurisprudence has
seldom treated common law as a constantly evolving
phenomenon rooted in history; those interpretive
theories that do acknowledge the antiquity of
common law ignore the practical contemporary
significance of its historical forms. The reasons for
this omission are partly theoretical and partly
political. In theoretical terms, modern jurisprudence
has consistently treated law as a unified system of
rules that can be studied at any given moment in time
as a logical whole. The notion of jurisprudence as a
system of norms or principles deemphasizes history
in favor of the coherence of a system. In this view,
the past of the system is conceived as no more than
the continuous succession of its states of presence. In
political terms, believing in the logic of law is a
necessary part of believing in its fairness; even if
history shows the legal tradition to be far from
unitary and seldom logical, the prestige of the legal
institution requires that jurisprudence treat the
tradition as if it were, in essence, the application of
known rules to objectively determined facts. To
suggest otherwise would be dispiriting for the student
and demoralizing for the public.
Legal historian Peter Goodrich has argued,
however, that common law is most fruitfully studied
as a continually developing tradition rather than as a
set of rules. Taking his cue from the study of
literature, Goodrich sees common law as a sort of
literary text, with history and tradition serving as the
text’s narrative development. To study the common
law historically, says Goodrich, is to study a text in
which fiction is as influential as analysis, perception
as significant as rule, and the play of memory as
strong as the logic of argument. The concept of
tradition, for Goodrich, implies not only the
preservation and transmission of existing forms, but
also the continuous rewriting of those forms to adapt
them to contemporary legal circumstances.

  1. Which one of the following best describes the
    author’s opinion of most modern academic theories
    of common law?
    (A) They are overly detailed and thus stultifying to
    both the student and the public.
    (B) They lack an essential dimension that would
    increase their accuracy.
    (C) They overemphasize the practical aspects of
    the common law at the expense of the
    theoretical.
    (D) They excuse students of the law from the study
    of important legal disputes of the past.
    (E) They routinely treat the study of the law as an
    art rather than as a science.
A

Correct Answer: B

Passage Analysis

B Extract Infer

The author criticizes modern academic theories of common law, so the credited response will reflect this.

A. No. The author never claims that most modern academic theories of common law are boring.

B. Yes. The author criticizes them for viewing common law not as evolving and based in history but instead as static.

C. No. The author never claims that practical factors take precedence over theoretical factors in modern academic theories of common law.

D. No. Students have to study past legal disputes, according to the first paragraph. The problem is that they are not taught that these disputes may well influence and affect modern legal disputes.

E. No. The author never claims that modern academic theories of common law treat the study of the law as an art.

134
Q

In England the burden of history weighs heavily
on common law, that unwritten code of time-honored
laws derived largely from English judicial custom and
precedent. Students of contemporary British law are
frequently required to study medieval cases, to
interpret archaic Latin maxims, or to confront
doctrinal principles whose validity is based solely on
their being part of the “timeless reason” of the
English legal tradition. Centuries-old custom serves as
the basis both for the divisions of law school subject
matter and for much of the terminology of legal
redress. Connected not only with legal history but
also with the cultural history of the English people,
common law cannot properly be understood without
taking a long historical view.
Yet the academic study of jurisprudence has
seldom treated common law as a constantly evolving
phenomenon rooted in history; those interpretive
theories that do acknowledge the antiquity of
common law ignore the practical contemporary
significance of its historical forms. The reasons for
this omission are partly theoretical and partly
political. In theoretical terms, modern jurisprudence
has consistently treated law as a unified system of
rules that can be studied at any given moment in time
as a logical whole. The notion of jurisprudence as a
system of norms or principles deemphasizes history
in favor of the coherence of a system. In this view,
the past of the system is conceived as no more than
the continuous succession of its states of presence. In
political terms, believing in the logic of law is a
necessary part of believing in its fairness; even if
history shows the legal tradition to be far from
unitary and seldom logical, the prestige of the legal
institution requires that jurisprudence treat the
tradition as if it were, in essence, the application of
known rules to objectively determined facts. To
suggest otherwise would be dispiriting for the student
and demoralizing for the public.
Legal historian Peter Goodrich has argued,
however, that common law is most fruitfully studied
as a continually developing tradition rather than as a
set of rules. Taking his cue from the study of
literature, Goodrich sees common law as a sort of
literary text, with history and tradition serving as the
text’s narrative development. To study the common
law historically, says Goodrich, is to study a text in
which fiction is as influential as analysis, perception
as significant as rule, and the play of memory as
strong as the logic of argument. The concept of
tradition, for Goodrich, implies not only the
preservation and transmission of existing forms, but
also the continuous rewriting of those forms to adapt
them to contemporary legal circumstances.

  1. The primary purpose of the passage is to
    (A) explain a paradoxical situation and discuss a
    new view of the situation
    (B) supply a chronological summary of the history
    of an idea
    (C) trace the ideas of an influential theorist and
    evaluate the theorist’s ongoing work
    (D) contrast the legal theories of past eras with
    those of today and suggest how these theories
    should be studied
    (E) advocate a traditional school of thought while
    criticizing a new trend
A

Correct Answer: A

Passage Analysis

A Big Picture

The author criticizes modern theories of common law and discusses a new interpretation. The credited response will reflect this.

A. Yes. Common law is recognized as coming from history and is the basis for legal decisions, but modern theories of common law don’t address its history or this history’s relevance to modern law. In the third paragraph, the author introduces Peter Goodrich’s new view of the situation.

B. No. The passage is not just a summary. It is, in part, a criticism.

C. No. The only theorist specifically mentioned is Peter Goodrich, and his ideas are not traced through his career.

D. No. The legal theories discussed are all modern theories.

E. No. The passage’s purpose is the opposite of this.

135
Q

The passages discuss relationships between business
interests and university research.
Passage A
As university researchers working in a “gift
economy” dedicated to collegial sharing of ideas, we
have long been insulated from market pressures. The
recent tendency to treat research findings as
commodities, tradable for cash, threatens this tradition
and the role of research as a public good.
The nurseries for new ideas are traditionally
universities, which provide an environment uniquely
suited to the painstaking testing and revision of
theories. Unfortunately, the market process and values
governing commodity exchange are ill suited to the
cultivation and management of new ideas. With their
shareholders impatient for quick returns, businesses
are averse to wide-ranging experimentation. And, what
is even more important, few commercial enterprises
contain the range of expertise needed to handle the
replacement of shattered theoretical frameworks.
Further, since entrepreneurs usually have little
affinity for adventure of the intellectual sort, they can
buy research and bury its products, hiding knowledge
useful to society or to their competitors. The growth of
industrial biotechnology, for example, has been
accompanied by a reduction in the free sharing of
research methods and results—a high price to pay for
the undoubted benefits of new drugs and therapies.
Important new experimental results once led
university scientists to rush down the hall and share
their excitement with colleagues. When instead the
rush is to patent lawyers and venture capitalists, I
worry about the long-term future of scientific
discovery.
Passage B
The fruits of pure science were once considered
primarily a public good, available for society as a
whole. The argument for this view was that most of
these benefits were produced through government
support of universities, and thus no individual was
entitled to restrict access to them.
Today, however, the critical role of science in the
modern “information economy” means that what was
previously seen as a public good is being transformed
into a market commodity. For example, by exploiting
the information that basic research has accumulated
about the detailed structures of cells and genes, the
biotechnology industry can derive profitable
pharmaceuticals or medical screening technologies. In
this context, assertion of legal claims to “intellectual
property”—not just in commercial products but in the
underlying scientific knowledge—becomes crucial.
Previously, the distinction between a scientific
“discovery” (which could not be patented) and a
technical “invention” (which could) defined the limits
of industry’s ability to patent something. Today,
however, the speed with which scientific discoveries
can be turned into products and the large profits

LSAT Preptest 53 Reading Comprehension Passage 3 Analysis.

A

LSAT Preptest 53 Reading Comprehension Passage 3 Analysis.

Passage A

The author criticizes the recent tendency to treat research findings as commodities, claiming that this threatens the tradition of collegial sharing of ideas and the role of research as a public good. This is because the market process and values governing commodity exchange are ill suited to the development and revision of new ideas. Businesses are too focused on short-term gains and, because entrepreneurs aren’t usually interested in intellectual pursuit for its own sake, they are more likely to buy research and prevent its dissemination to other researchers, thereby tamping down the exchange of ideas.

Passage B

This author is much less critical of the change in the role of science to a market commodity. This passage documents the change and what it means to science. As a result of this change, the assertion of legal claims to intellectual property becomes essential. Also, what counts as discovery—not patentable—and invention—patentable—has become blurred. The author then mentions that industry claims that it should be able to claim as its property any discovery made by those whose research it has supported.

136
Q

Passage A
As university researchers working in a “gift
economy” dedicated to collegial sharing of ideas, we
have long been insulated from market pressures. The
recent tendency to treat research findings as
commodities, tradable for cash, threatens this tradition
and the role of research as a public good.
The nurseries for new ideas are traditionally
universities, which provide an environment uniquely
suited to the painstaking testing and revision of
theories. Unfortunately, the market process and values
governing commodity exchange are ill suited to the
cultivation and management of new ideas. With their
shareholders impatient for quick returns, businesses
are averse to wide-ranging experimentation. And, what
is even more important, few commercial enterprises
contain the range of expertise needed to handle the
replacement of shattered theoretical frameworks.
Further, since entrepreneurs usually have little
affinity for adventure of the intellectual sort, they can
buy research and bury its products, hiding knowledge
useful to society or to their competitors. The growth of
industrial biotechnology, for example, has been
accompanied by a reduction in the free sharing of
research methods and results—a high price to pay for
the undoubted benefits of new drugs and therapies.
Important new experimental results once led
university scientists to rush down the hall and share
their excitement with colleagues. When instead the
rush is to patent lawyers and venture capitalists, I
worry about the long-term future of scientific
discovery.
Passage B
The fruits of pure science were once considered
primarily a public good, available for society as a
whole. The argument for this view was that most of
these benefits were produced through government
support of universities, and thus no individual was
entitled to restrict access to them.
Today, however, the critical role of science in the
modern “information economy” means that what was
previously seen as a public good is being transformed
into a market commodity. For example, by exploiting
the information that basic research has accumulated
about the detailed structures of cells and genes, the
biotechnology industry can derive profitable
pharmaceuticals or medical screening technologies. In
this context, assertion of legal claims to “intellectual
property”—not just in commercial products but in the
underlying scientific knowledge—becomes crucial.
Previously, the distinction between a scientific
“discovery” (which could not be patented) and a
technical “invention” (which could) defined the limits
of industry’s ability to patent something. Today,
however, the speed with which scientific discoveries
can be turned into products and the large profits

  1. Both passages place in opposition the members of
    which one of the following pairs?
    (A) commercially successful research and
    commercially unsuccessful research
    (B) research methods and research results
    (C) a marketable commodity and a public good
    (D) a discovery and an invention
    (E) scientific research and other types of inquiry
A

Correct Answer: C

Passage Analysis

C Extract Fact

A. No. Neither passage discusses commercially unsuccessful research explicitly.

B. No. These are not placed in opposition in either passage.

C. Yes. Passage A does this near line 5 and passage B does this in the first sentence of the second paragraph.

D. No. Only passage B discusses discovery versus invention.

E. No. Both passages are focused on scientific research and not on other types of inquiry.

136
Q

Passage A
As university researchers working in a “gift
economy” dedicated to collegial sharing of ideas, we
have long been insulated from market pressures. The
recent tendency to treat research findings as
commodities, tradable for cash, threatens this tradition
and the role of research as a public good.
The nurseries for new ideas are traditionally
universities, which provide an environment uniquely
suited to the painstaking testing and revision of
theories. Unfortunately, the market process and values
governing commodity exchange are ill suited to the
cultivation and management of new ideas. With their
shareholders impatient for quick returns, businesses
are averse to wide-ranging experimentation. And, what
is even more important, few commercial enterprises
contain the range of expertise needed to handle the
replacement of shattered theoretical frameworks.
Further, since entrepreneurs usually have little
affinity for adventure of the intellectual sort, they can
buy research and bury its products, hiding knowledge
useful to society or to their competitors. The growth of
industrial biotechnology, for example, has been
accompanied by a reduction in the free sharing of
research methods and results—a high price to pay for
the undoubted benefits of new drugs and therapies.
Important new experimental results once led
university scientists to rush down the hall and share
their excitement with colleagues. When instead the
rush is to patent lawyers and venture capitalists, I
worry about the long-term future of scientific
discovery.
Passage B
The fruits of pure science were once considered
primarily a public good, available for society as a
whole. The argument for this view was that most of
these benefits were produced through government
support of universities, and thus no individual was
entitled to restrict access to them.
Today, however, the critical role of science in the
modern “information economy” means that what was
previously seen as a public good is being transformed
into a market commodity. For example, by exploiting
the information that basic research has accumulated
about the detailed structures of cells and genes, the
biotechnology industry can derive profitable
pharmaceuticals or medical screening technologies. In
this context, assertion of legal claims to “intellectual
property”—not just in commercial products but in the
underlying scientific knowledge—becomes crucial.
Previously, the distinction between a scientific
“discovery” (which could not be patented) and a
technical “invention” (which could) defined the limits
of industry’s ability to patent something. Today,
however, the speed with which scientific discoveries
can be turned into products and the large profits

  1. Which one of the following is discussed in passage B
    but not in passage A?
    (A) the blurring of the legal distinction between
    discovery and invention
    (B) the general effects of the market on the
    exchange of scientific knowledge
    (C) the role of scientific research in supplying
    public goods
    (D) new pharmaceuticals that result from
    industrial research
    (E) industry’s practice of restricting access to
    research findings
A

Correct Answer: A

Passage Analysis

A Extract Fact

A. Yes. Passage A does not discuss the legal aspects of the change in the role of science to a market commodity, while passage B discusses this just after line 55.

B. No. Both passages discuss the effects of the market on the exchange of scientific knowledge.

C. No. Both passages discuss this.

D. No. Neither passage discusses new pharmaceuticals that result from industrial research. Passage B mentions pharmaceuticals but not in the context of industrial research.

E. No. Both passages discuss industry’s practice of restricting access to research findings.

137
Q

Passage A
As university researchers working in a “gift
economy” dedicated to collegial sharing of ideas, we
have long been insulated from market pressures. The
recent tendency to treat research findings as
commodities, tradable for cash, threatens this tradition
and the role of research as a public good.
The nurseries for new ideas are traditionally
universities, which provide an environment uniquely
suited to the painstaking testing and revision of
theories. Unfortunately, the market process and values
governing commodity exchange are ill suited to the
cultivation and management of new ideas. With their
shareholders impatient for quick returns, businesses
are averse to wide-ranging experimentation. And, what
is even more important, few commercial enterprises
contain the range of expertise needed to handle the
replacement of shattered theoretical frameworks.
Further, since entrepreneurs usually have little
affinity for adventure of the intellectual sort, they can
buy research and bury its products, hiding knowledge
useful to society or to their competitors. The growth of
industrial biotechnology, for example, has been
accompanied by a reduction in the free sharing of
research methods and results—a high price to pay for
the undoubted benefits of new drugs and therapies.
Important new experimental results once led
university scientists to rush down the hall and share
their excitement with colleagues. When instead the
rush is to patent lawyers and venture capitalists, I
worry about the long-term future of scientific
discovery.
Passage B
The fruits of pure science were once considered
primarily a public good, available for society as a
whole. The argument for this view was that most of
these benefits were produced through government
support of universities, and thus no individual was
entitled to restrict access to them.
Today, however, the critical role of science in the
modern “information economy” means that what was
previously seen as a public good is being transformed
into a market commodity. For example, by exploiting
the information that basic research has accumulated
about the detailed structures of cells and genes, the
biotechnology industry can derive profitable
pharmaceuticals or medical screening technologies. In
this context, assertion of legal claims to “intellectual
property”—not just in commercial products but in the
underlying scientific knowledge—becomes crucial.
Previously, the distinction between a scientific
“discovery” (which could not be patented) and a
technical “invention” (which could) defined the limits
of industry’s ability to patent something. Today,
however, the speed with which scientific discoveries
can be turned into products and the large profits

  1. Both passages refer to which one of the following?
    (A) theoretical frameworks
    (B) venture capitalists
    (C) physics and chemistry
    (D) industrial biotechnology
    (E) shareholders
A

Correct Answer: D

Passage Analysis

D Extract Fact

A. No. Only passage A refers to theoretical frameworks.

B. No. Only passage A refers to venture capitalists.

C. No. Passage A does not refer to either physics or chemistry.

D. Yes. Passage A refers to this in line 22 and passage B refers to this in line 44.

E. No. Only passage A refers to shareholders.

138
Q

Passage A
As university researchers working in a “gift
economy” dedicated to collegial sharing of ideas, we
have long been insulated from market pressures. The
recent tendency to treat research findings as
commodities, tradable for cash, threatens this tradition
and the role of research as a public good.
The nurseries for new ideas are traditionally
universities, which provide an environment uniquely
suited to the painstaking testing and revision of
theories. Unfortunately, the market process and values
governing commodity exchange are ill suited to the
cultivation and management of new ideas. With their
shareholders impatient for quick returns, businesses
are averse to wide-ranging experimentation. And, what
is even more important, few commercial enterprises
contain the range of expertise needed to handle the
replacement of shattered theoretical frameworks.
Further, since entrepreneurs usually have little
affinity for adventure of the intellectual sort, they can
buy research and bury its products, hiding knowledge
useful to society or to their competitors. The growth of
industrial biotechnology, for example, has been
accompanied by a reduction in the free sharing of
research methods and results—a high price to pay for
the undoubted benefits of new drugs and therapies.
Important new experimental results once led
university scientists to rush down the hall and share
their excitement with colleagues. When instead the
rush is to patent lawyers and venture capitalists, I
worry about the long-term future of scientific
discovery.
Passage B
The fruits of pure science were once considered
primarily a public good, available for society as a
whole. The argument for this view was that most of
these benefits were produced through government
support of universities, and thus no individual was
entitled to restrict access to them.
Today, however, the critical role of science in the
modern “information economy” means that what was
previously seen as a public good is being transformed
into a market commodity. For example, by exploiting
the information that basic research has accumulated
about the detailed structures of cells and genes, the
biotechnology industry can derive profitable
pharmaceuticals or medical screening technologies. In
this context, assertion of legal claims to “intellectual
property”—not just in commercial products but in the
underlying scientific knowledge—becomes crucial.
Previously, the distinction between a scientific
“discovery” (which could not be patented) and a
technical “invention” (which could) defined the limits
of industry’s ability to patent something. Today,
however, the speed with which scientific discoveries
can be turned into products and the large profits

It can be inferred from the passages that the authors
believe that the increased constraint on access to
scientific information and ideas arises from
(A) the enormous increase in the volume of
scientific knowledge that is being generated
(B) the desire of individual researchers to receive
credit for their discoveries
(C) the striving of commercial enterprises to gain
a competitive advantage in the market
(D) moral reservations about the social impact of
some scientific research
(E) a drastic reduction in government funding for
university research

A

Correct Answer: C

Passage Analysis

C Extract Infer

A. No. Neither passage mentions the enormous increase in the volume of scientific knowledge being generated.

B. No. Neither passage discusses the desires of the individual researchers.

C. Yes. Passage A implies this in the third paragraph, while passage B implies this in the second paragraph.

D. No. Neither passage discusses moral reservations.

E. No. Neither passage states that there has been a drastic reduction in government funding.

139
Q

Passage A
As university researchers working in a “gift
economy” dedicated to collegial sharing of ideas, we
have long been insulated from market pressures. The
recent tendency to treat research findings as
commodities, tradable for cash, threatens this tradition
and the role of research as a public good.
The nurseries for new ideas are traditionally
universities, which provide an environment uniquely
suited to the painstaking testing and revision of
theories. Unfortunately, the market process and values
governing commodity exchange are ill suited to the
cultivation and management of new ideas. With their
shareholders impatient for quick returns, businesses
are averse to wide-ranging experimentation. And, what
is even more important, few commercial enterprises
contain the range of expertise needed to handle the
replacement of shattered theoretical frameworks.
Further, since entrepreneurs usually have little
affinity for adventure of the intellectual sort, they can
buy research and bury its products, hiding knowledge
useful to society or to their competitors. The growth of
industrial biotechnology, for example, has been
accompanied by a reduction in the free sharing of
research methods and results—a high price to pay for
the undoubted benefits of new drugs and therapies.
Important new experimental results once led
university scientists to rush down the hall and share
their excitement with colleagues. When instead the
rush is to patent lawyers and venture capitalists, I
worry about the long-term future of scientific
discovery.
Passage B
The fruits of pure science were once considered
primarily a public good, available for society as a
whole. The argument for this view was that most of
these benefits were produced through government
support of universities, and thus no individual was
entitled to restrict access to them.
Today, however, the critical role of science in the
modern “information economy” means that what was
previously seen as a public good is being transformed
into a market commodity. For example, by exploiting
the information that basic research has accumulated
about the detailed structures of cells and genes, the
biotechnology industry can derive profitable
pharmaceuticals or medical screening technologies. In
this context, assertion of legal claims to “intellectual
property”—not just in commercial products but in the
underlying scientific knowledge—becomes crucial.
Previously, the distinction between a scientific
“discovery” (which could not be patented) and a
technical “invention” (which could) defined the limits
of industry’s ability to patent something. Today,
however, the speed with which scientific discoveries
can be turned into products and the large profits

  1. Which one of the following statements is most
    strongly supported by both passages?
    (A) Many scientific researchers who previously
    worked in universities have begun to work in
    the biotechnology industry.
    (B) Private biotechnology companies have
    invalidly patented the basic research findings
    of university researchers.
    (C) Because of the nature of current scientific
    research, patent authorities no longer
    consider the distinction between discoveries
    and inventions to be clear-cut.
    (D) In the past, scientists working in industry had
    free access to the results of basic research
    conducted in universities.
    (E) Government-funded research in universities
    has traditionally been motivated by the goals
    of private industry
A

Correct Answer: D

Passage Analysis

D Extract Infer

A. No. Neither passage implies that many scientific researchers have been leaving universities to work in industry.

B. No. Passage A does not discuss patents and passage B mentions the issues concerning patents without making a judgment.

C. No. This subject is discussed only in passage B.

D. Yes. Both passages contrast the new limitations on access to the results of basic research conducted in universities to how it was in the past.

E. No. Both passages claim that the goals of private industry have only recently become a major motivation for research.

140
Q

Sometimes there is no more effective means of
controlling an agricultural pest than giving free rein
to its natural predators. A case in point is the
cyclamen mite, a pest whose population can be
effectively controlled by a predatory mite of the
genus Typhlodromus. Cyclamen mites infest
strawberry plants; they typically establish themselves
in a strawberry field shortly after planting, but their
populations do not reach significantly damaging
levels until the plants’ second year. Typhlodromus
mites usually invade the strawberry fields during the
second year, rapidly subdue the cyclamen mite
populations, and keep them from reaching
significantly damaging levels.
Typhlodromus owes its effectiveness as a predator
to several factors in addition to its voracious appetite.
Its population can increase as rapidly as that of its
prey. Both species reproduce by parthenogenesis—a
mode of reproduction in which unfertilized eggs
develop into fertile females. Cyclamen mites lay three
eggs per day over the four or five days of their
reproductive life span; Typhlodromus lay two or three
eggs per day for eight to ten days. Seasonal
synchrony of Typhlodromus reproduction with the
growth of prey populations and ability to survive at
low prey densities also contribute to the predatory
efficiency of Typhlodromus. During winter, when
cyclamen mite populations dwindle to a few
individuals hidden in the crevices and folds of leaves
in the crowns of the strawberry plants, the predatory
mites subsist on the honeydew produced by aphids
and white flies. They do not reproduce except when
they are feeding on the cyclamen mites. These
features, which make Typhlodromus well-suited for
exploiting the seasonal rises and falls of its prey, are
common among predators that control prey
populations.
Greenhouse experiments have verified the
importance of Typhlodromus predation for keeping
cyclamen mites in check. One group of strawberry
plants was stocked with both predator and prey mites;
a second group was kept predator-free by regular
application of parathion, an insecticide that kills the
predatory species but does not affect the cyclamen
mite. Throughout the study, populations of cyclamen
mites remained low in plots shared with
Typhlodromus, but their infestation attained
significantly damaging proportions on predator-free
plants.
Applying parathion in this instance is a clear case
in which using a pesticide would do far more harm
than good to an agricultural enterprise. The results
were similar in field plantings of strawberries, where
cyclamen mites also reached damaging levels when
predators were eliminated by parathion, but they did
not attain such levels in untreated plots. When
cyclamen mite populations began to increase in an
untreated planting, the predator populations quickly
responded to reduce the outbreak. On average,
cyclamen mites were about 25 times more abundant
in the absence of predators than in their presence.

  1. Which one of the following most accurately expresses
    the main point of the passage?
    (A) Control of agricultural pests is most effectively
    and safely accomplished without the use of
    pesticides, because these pesticides can kill
    predators that also control the pests.
    (B) Experimental verification is essential in
    demonstrating the effectiveness of natural
    controls of agricultural pests.
    (C) The relationship between Typhlodromus and
    cyclamen mites demonstrates how natural
    predation can keep a population of
    agricultural pests in check.
    (D) Predation by Typhlodromus is essential for the
    control of cyclamen mite populations in
    strawberry fields.
    (E) Similarity in mode and timing of reproduction
    is what enables Typhlodromus effectively to
    control populations of cyclamen mites in
    fields of strawberry plants
A

Correct Answer: C

Passage Analysis

C Big Picture

A. No. This is too strong. The passage claims that there are some instances in which pesticides cause more harm but doesn’t claim this about all pesticides in all situations.

B. No. The passage does provide experimental verification but never claims that this is essential.

C. Yes. The passage informs the reader that natural predation can be more effective against agricultural pests and uses the relationship between Typhlodromus and cyclamen mites as an example of this effectiveness.

D. No. The passage uses the relationship between Typhlodromus and cyclamen mites as an example to illustrate its point. This relationship is not the main point of the passage.

E. No. This is too specific. This is discussed only in the second paragraph.

141
Q

Sometimes there is no more effective means of
controlling an agricultural pest than giving free rein
to its natural predators. A case in point is the
cyclamen mite, a pest whose population can be
effectively controlled by a predatory mite of the
genus Typhlodromus. Cyclamen mites infest
strawberry plants; they typically establish themselves
in a strawberry field shortly after planting, but their
populations do not reach significantly damaging
levels until the plants’ second year. Typhlodromus
mites usually invade the strawberry fields during the
second year, rapidly subdue the cyclamen mite
populations, and keep them from reaching
significantly damaging levels.
Typhlodromus owes its effectiveness as a predator
to several factors in addition to its voracious appetite.
Its population can increase as rapidly as that of its
prey. Both species reproduce by parthenogenesis—a
mode of reproduction in which unfertilized eggs
develop into fertile females. Cyclamen mites lay three
eggs per day over the four or five days of their
reproductive life span; Typhlodromus lay two or three
eggs per day for eight to ten days. Seasonal
synchrony of Typhlodromus reproduction with the
growth of prey populations and ability to survive at
low prey densities also contribute to the predatory
efficiency of Typhlodromus. During winter, when
cyclamen mite populations dwindle to a few
individuals hidden in the crevices and folds of leaves
in the crowns of the strawberry plants, the predatory
mites subsist on the honeydew produced by aphids
and white flies. They do not reproduce except when
they are feeding on the cyclamen mites. These
features, which make Typhlodromus well-suited for
exploiting the seasonal rises and falls of its prey, are
common among predators that control prey
populations.
Greenhouse experiments have verified the
importance of Typhlodromus predation for keeping
cyclamen mites in check. One group of strawberry
plants was stocked with both predator and prey mites;
a second group was kept predator-free by regular
application of parathion, an insecticide that kills the
predatory species but does not affect the cyclamen
mite. Throughout the study, populations of cyclamen
mites remained low in plots shared with
Typhlodromus, but their infestation attained
significantly damaging proportions on predator-free
plants.
Applying parathion in this instance is a clear case
in which using a pesticide would do far more harm
than good to an agricultural enterprise. The results
were similar in field plantings of strawberries, where
cyclamen mites also reached damaging levels when
predators were eliminated by parathion, but they did
not attain such levels in untreated plots. When
cyclamen mite populations began to increase in an
untreated planting, the predator populations quickly
responded to reduce the outbreak. On average,
cyclamen mites were about 25 times more abundant
in the absence of predators than in their presence.

  1. Which one of the following most accurately expresses
    the main point of the passage?
    (A) Control of agricultural pests is most effectively
    and safely accomplished without the use of
    pesticides, because these pesticides can kill
    predators that also control the pests.
    (B) Experimental verification is essential in
    demonstrating the effectiveness of natural
    controls of agricultural pests.
    (C) The relationship between Typhlodromus and
    cyclamen mites demonstrates how natural
    predation can keep a population of
    agricultural pests in check.
    (D) Predation by Typhlodromus is essential for the
    control of cyclamen mite populations in
    strawberry fields.
    (E) Similarity in mode and timing of reproduction
    is what enables Typhlodromus effectively to
    control populations of cyclamen mites in
    fields of strawberry plants.
A

Correct Answer: C

Passage Analysis

C Big Picture

A. No. This is too strong. The passage claims that there are some instances in which pesticides cause more harm but doesn’t claim this about all pesticides in all situations.

B. No. The passage does provide experimental verification but never claims that this is essential.

C. Yes. The passage informs the reader that natural predation can be more effective against agricultural pests and uses the relationship between Typhlodromus and cyclamen mites as an example of this effectiveness.

D. No. The passage uses the relationship between Typhlodromus and cyclamen mites as an example to illustrate its point. This relationship is not the main point of the passage.

E. No. This is too specific. This is discussed only in the second paragraph.

142
Q

Sometimes there is no more effective means of
controlling an agricultural pest than giving free rein
to its natural predators. A case in point is the
cyclamen mite, a pest whose population can be
effectively controlled by a predatory mite of the
genus Typhlodromus. Cyclamen mites infest
strawberry plants; they typically establish themselves
in a strawberry field shortly after planting, but their
populations do not reach significantly damaging
levels until the plants’ second year. Typhlodromus
mites usually invade the strawberry fields during the
second year, rapidly subdue the cyclamen mite
populations, and keep them from reaching
significantly damaging levels.
Typhlodromus owes its effectiveness as a predator
to several factors in addition to its voracious appetite.
Its population can increase as rapidly as that of its
prey. Both species reproduce by parthenogenesis—a
mode of reproduction in which unfertilized eggs
develop into fertile females. Cyclamen mites lay three
eggs per day over the four or five days of their
reproductive life span; Typhlodromus lay two or three
eggs per day for eight to ten days. Seasonal
synchrony of Typhlodromus reproduction with the
growth of prey populations and ability to survive at
low prey densities also contribute to the predatory
efficiency of Typhlodromus. During winter, when
cyclamen mite populations dwindle to a few
individuals hidden in the crevices and folds of leaves
in the crowns of the strawberry plants, the predatory
mites subsist on the honeydew produced by aphids
and white flies. They do not reproduce except when
they are feeding on the cyclamen mites. These
features, which make Typhlodromus well-suited for
exploiting the seasonal rises and falls of its prey, are
common among predators that control prey
populations.
Greenhouse experiments have verified the
importance of Typhlodromus predation for keeping
cyclamen mites in check. One group of strawberry
plants was stocked with both predator and prey mites;
a second group was kept predator-free by regular
application of parathion, an insecticide that kills the
predatory species but does not affect the cyclamen
mite. Throughout the study, populations of cyclamen
mites remained low in plots shared with
Typhlodromus, but their infestation attained
significantly damaging proportions on predator-free
plants.
Applying parathion in this instance is a clear case
in which using a pesticide would do far more harm
than good to an agricultural enterprise. The results
were similar in field plantings of strawberries, where
cyclamen mites also reached damaging levels when
predators were eliminated by parathion, but they did
not attain such levels in untreated plots. When
cyclamen mite populations began to increase in an
untreated planting, the predator populations quickly
responded to reduce the outbreak. On average,
cyclamen mites were about 25 times more abundant
in the absence of predators than in their presence.

  1. Based on the passage, the author would probably
    hold that which one of the following principles is
    fundamental to long-term predatory control of
    agricultural pests?
    (A) The reproduction of the predator population
    should be synchronized with that of the prey
    population, so that the number of predators
    surges just prior to a surge in prey numbers.
    (B) The effectiveness of the predatory relationship
    should be experimentally demonstrable in
    greenhouse as well as field applications.
    (C) The prey population should be able to survive
    in times of low crop productivity, so that the
    predator population will not decrease to very
    low levels.
    (D) The predator population’s level of
    consumption of the prey species should be
    responsive to variations in the size of the prey
    population.
    (E) The predator population should be vulnerable
    only to pesticides to which the prey
    population is also vulnerable.
A

Correct Answer: D

Passage Analysis

D Extract Infer

The passage discusses the particulars of control of agricultural pests in the second paragraph.

A. No. The passage talks only about seasonal synchrony and doesn’t state that the number of predators should surge just prior to a surge in the number of prey.

B. No. The passage mentions demonstration of effectiveness in an experiment but does not claim that this is essential.

C. No. The passage claims that the predator population is able to survive even when the prey population is dormant or has dropped to very low levels.

D. Yes. This is exactly what the passage claims in the first part of the second paragraph.

E. No. The passage is not advocating pesticides as fundamental to long-term predatory control of agricultural pests.

143
Q

Sometimes there is no more effective means of
controlling an agricultural pest than giving free rein
to its natural predators. A case in point is the
cyclamen mite, a pest whose population can be
effectively controlled by a predatory mite of the
genus Typhlodromus. Cyclamen mites infest
strawberry plants; they typically establish themselves
in a strawberry field shortly after planting, but their
populations do not reach significantly damaging
levels until the plants’ second year. Typhlodromus
mites usually invade the strawberry fields during the
second year, rapidly subdue the cyclamen mite
populations, and keep them from reaching
significantly damaging levels.
Typhlodromus owes its effectiveness as a predator
to several factors in addition to its voracious appetite.
Its population can increase as rapidly as that of its
prey. Both species reproduce by parthenogenesis—a
mode of reproduction in which unfertilized eggs
develop into fertile females. Cyclamen mites lay three
eggs per day over the four or five days of their
reproductive life span; Typhlodromus lay two or three
eggs per day for eight to ten days. Seasonal
synchrony of Typhlodromus reproduction with the
growth of prey populations and ability to survive at
low prey densities also contribute to the predatory
efficiency of Typhlodromus. During winter, when
cyclamen mite populations dwindle to a few
individuals hidden in the crevices and folds of leaves
in the crowns of the strawberry plants, the predatory
mites subsist on the honeydew produced by aphids
and white flies. They do not reproduce except when
they are feeding on the cyclamen mites. These
features, which make Typhlodromus well-suited for
exploiting the seasonal rises and falls of its prey, are
common among predators that control prey
populations.
Greenhouse experiments have verified the
importance of Typhlodromus predation for keeping
cyclamen mites in check. One group of strawberry
plants was stocked with both predator and prey mites;
a second group was kept predator-free by regular
application of parathion, an insecticide that kills the
predatory species but does not affect the cyclamen
mite. Throughout the study, populations of cyclamen
mites remained low in plots shared with
Typhlodromus, but their infestation attained
significantly damaging proportions on predator-free
plants.
Applying parathion in this instance is a clear case
in which using a pesticide would do far more harm
than good to an agricultural enterprise. The results
were similar in field plantings of strawberries, where
cyclamen mites also reached damaging levels when
predators were eliminated by parathion, but they did
not attain such levels in untreated plots. When
cyclamen mite populations began to increase in an
untreated planting, the predator populations quickly
responded to reduce the outbreak. On average,
cyclamen mites were about 25 times more abundant
in the absence of predators than in their presence.

  1. Which one of the following is mentioned in the
    passage as a factor contributing to the effectiveness of
    Typhlodromus as a predator?
    (A) its ability to withstand most insecticides except
    parathion
    (B) its lack of natural predators in strawberry
    fields
    (C) its ability to live in different climates in
    different geographic regions
    (D) its constant food supply in cyclamen mite
    populations
    (E) its ability to survive when few prey are
    available
A

Correct Answer: E

Passage Analysis

E Extract Fact

The passage discusses this topic in the second paragraph.

A. No. The passage does not discuss Typhlodromus’s ability to withstand most insecticides.

B. No. The passage does not discuss the natural predators of Typhlodromus.

C. No. The passage does not claim that Typhlodromus can live in different climates and regions.

D. No. The passage acknowledges that Typhlodromus does not always have a constant supply of cyclamen mites.

E. Yes. The passage claims that Typhlodromus’s ability to subsist on the honeydew produced by aphids and white flies when there aren’t many cyclamen mites is a feature that makes Typhlodromus effective as a predator.

144
Q

Sometimes there is no more effective means of
controlling an agricultural pest than giving free rein
to its natural predators. A case in point is the
cyclamen mite, a pest whose population can be
effectively controlled by a predatory mite of the
genus Typhlodromus. Cyclamen mites infest
strawberry plants; they typically establish themselves
in a strawberry field shortly after planting, but their
populations do not reach significantly damaging
levels until the plants’ second year. Typhlodromus
mites usually invade the strawberry fields during the
second year, rapidly subdue the cyclamen mite
populations, and keep them from reaching
significantly damaging levels.
Typhlodromus owes its effectiveness as a predator
to several factors in addition to its voracious appetite.
Its population can increase as rapidly as that of its
prey. Both species reproduce by parthenogenesis—a
mode of reproduction in which unfertilized eggs
develop into fertile females. Cyclamen mites lay three
eggs per day over the four or five days of their
reproductive life span; Typhlodromus lay two or three
eggs per day for eight to ten days. Seasonal
synchrony of Typhlodromus reproduction with the
growth of prey populations and ability to survive at
low prey densities also contribute to the predatory
efficiency of Typhlodromus. During winter, when
cyclamen mite populations dwindle to a few
individuals hidden in the crevices and folds of leaves
in the crowns of the strawberry plants, the predatory
mites subsist on the honeydew produced by aphids
and white flies. They do not reproduce except when
they are feeding on the cyclamen mites. These
features, which make Typhlodromus well-suited for
exploiting the seasonal rises and falls of its prey, are
common among predators that control prey
populations.
Greenhouse experiments have verified the
importance of Typhlodromus predation for keeping
cyclamen mites in check. One group of strawberry
plants was stocked with both predator and prey mites;
a second group was kept predator-free by regular
application of parathion, an insecticide that kills the
predatory species but does not affect the cyclamen
mite. Throughout the study, populations of cyclamen
mites remained low in plots shared with
Typhlodromus, but their infestation attained
significantly damaging proportions on predator-free
plants.
Applying parathion in this instance is a clear case
in which using a pesticide would do far more harm
than good to an agricultural enterprise. The results
were similar in field plantings of strawberries, where
cyclamen mites also reached damaging levels when
predators were eliminated by parathion, but they did
not attain such levels in untreated plots. When
cyclamen mite populations began to increase in an
untreated planting, the predator populations quickly
responded to reduce the outbreak. On average,
cyclamen mites were about 25 times more abundant
in the absence of predators than in their presence.

  1. Suppose that pesticide X drastically slows the
    reproductive rate of cyclamen mites and has no other
    direct effect on cyclamen mites or Typhlodromus.
    Based on the information in the passage, which one
    of the following would most likely have occurred if,
    in the experiments mentioned in the passage,
    pesticide X had been used instead of parathion, with
    all other conditions affecting the experiments
    remaining the same?
    (A) In both treated and untreated plots inhabited
    by both Typhlodromus and cyclamen mites,
    the latter would have been effectively
    controlled.
    (B) Cyclamen mite populations in all treated plots
    from which Typhlodromus was absent would
    have been substantially lower than in
    untreated plots inhabited by both kinds of
    mites.
    (C) In the treated plots, slowed reproduction in
    cyclamen mites would have led to a loss of
    reproductive synchrony between
    Typhlodromus and cyclamen mites.
    (D) In the treated plots, Typhlodromus populations
    would have decreased temporarily and would
    have eventually increased.
    (E) In the treated plots, cyclamen mite
    populations would have reached significantly
    damaging levels more slowly, but would have
    remained at those levels longer, than in
    untreated plots.
A

Correct Answer: A

Passage Analysis

A RC Reasoning

A. Yes. Parathion killed the predator species but didn’t affect the cyclamen mites. Since this pesticide does not affect the predator species, and Typhlodromus preys on cyclamen mites, the mites should have been controlled in both treated and untreated plots.

B. No. If there were no Typhlodromus, then the cyclamen mite populations of plots in which Typhlodromus was absent would likely have been higher than those of the plots in which Typhlodromus was preying on the cyclamen mites.

C. No. We don’t know this because the key to Typhlodromus’s reproductive cycle is that it is faster than that of the cyclamen mites.

D. No. The pesticide does not have any direct effect on Typhlodromus.

E. No. We don’t know that they would have remained at damaging levels longer in the treated plots.

145
Q

Sometimes there is no more effective means of
controlling an agricultural pest than giving free rein
to its natural predators. A case in point is the
cyclamen mite, a pest whose population can be
effectively controlled by a predatory mite of the
genus Typhlodromus. Cyclamen mites infest
strawberry plants; they typically establish themselves
in a strawberry field shortly after planting, but their
populations do not reach significantly damaging
levels until the plants’ second year. Typhlodromus
mites usually invade the strawberry fields during the
second year, rapidly subdue the cyclamen mite
populations, and keep them from reaching
significantly damaging levels.
Typhlodromus owes its effectiveness as a predator
to several factors in addition to its voracious appetite.
Its population can increase as rapidly as that of its
prey. Both species reproduce by parthenogenesis—a
mode of reproduction in which unfertilized eggs
develop into fertile females. Cyclamen mites lay three
eggs per day over the four or five days of their
reproductive life span; Typhlodromus lay two or three
eggs per day for eight to ten days. Seasonal
synchrony of Typhlodromus reproduction with the
growth of prey populations and ability to survive at
low prey densities also contribute to the predatory
efficiency of Typhlodromus. During winter, when
cyclamen mite populations dwindle to a few
individuals hidden in the crevices and folds of leaves
in the crowns of the strawberry plants, the predatory
mites subsist on the honeydew produced by aphids
and white flies. They do not reproduce except when
they are feeding on the cyclamen mites. These
features, which make Typhlodromus well-suited for
exploiting the seasonal rises and falls of its prey, are
common among predators that control prey
populations.
Greenhouse experiments have verified the
importance of Typhlodromus predation for keeping
cyclamen mites in check. One group of strawberry
plants was stocked with both predator and prey mites;
a second group was kept predator-free by regular
application of parathion, an insecticide that kills the
predatory species but does not affect the cyclamen
mite. Throughout the study, populations of cyclamen
mites remained low in plots shared with
Typhlodromus, but their infestation attained
significantly damaging proportions on predator-free
plants.
Applying parathion in this instance is a clear case
in which using a pesticide would do far more harm
than good to an agricultural enterprise. The results
were similar in field plantings of strawberries, where
cyclamen mites also reached damaging levels when
predators were eliminated by parathion, but they did
not attain such levels in untreated plots. When
cyclamen mite populations began to increase in an
untreated planting, the predator populations quickly
responded to reduce the outbreak. On average,
cyclamen mites were about 25 times more abundant
in the absence of predators than in their presence.

  1. It can be inferred from the passage that the author
    would be most likely to agree with which one of the
    following statements about the use of predators to
    control pest populations?
    (A) If the use of predators to control cyclamen
    mite populations fails, then parathion should
    be used to control these populations.
    (B) Until the effects of the predators on beneficial
    insects that live in strawberry fields are
    assessed, such predators should be used with
    caution to control cyclamen mite
    populations.
    (C) Insecticides should be used to control certain
    pest populations in fields of crops only if the
    use of natural predators has proven
    inadequate.
    (D) If an insecticide can effectively control pest
    populations as well as predator populations,
    then it should be used instead of predators to
    control pest populations.
    (E) Predators generally control pest populations
    more effectively than pesticides because they
    do not harm the crops that their prey feed on.
A

Correct Answer: C

Passage Analysis

C Extract Infer

A. No. The passage never advocates the use of parathion.

B. No. The passage does not discuss the potential impact of predators on beneficial insects.

C. Yes. The passage advocates the use of predators over pesticides when the predators are effective.

D. No. The passage advocates the use of predators over pesticides when the predators are effective.

E. No. The passage does not claim that pesticides generally harm the crops.

146
Q

Sometimes there is no more effective means of
controlling an agricultural pest than giving free rein
to its natural predators. A case in point is the
cyclamen mite, a pest whose population can be
effectively controlled by a predatory mite of the
genus Typhlodromus. Cyclamen mites infest
strawberry plants; they typically establish themselves
in a strawberry field shortly after planting, but their
populations do not reach significantly damaging
levels until the plants’ second year. Typhlodromus
mites usually invade the strawberry fields during the
second year, rapidly subdue the cyclamen mite
populations, and keep them from reaching
significantly damaging levels.
Typhlodromus owes its effectiveness as a predator
to several factors in addition to its voracious appetite.
Its population can increase as rapidly as that of its
prey. Both species reproduce by parthenogenesis—a
mode of reproduction in which unfertilized eggs
develop into fertile females. Cyclamen mites lay three
eggs per day over the four or five days of their
reproductive life span; Typhlodromus lay two or three
eggs per day for eight to ten days. Seasonal
synchrony of Typhlodromus reproduction with the
growth of prey populations and ability to survive at
low prey densities also contribute to the predatory
efficiency of Typhlodromus. During winter, when
cyclamen mite populations dwindle to a few
individuals hidden in the crevices and folds of leaves
in the crowns of the strawberry plants, the predatory
mites subsist on the honeydew produced by aphids
and white flies. They do not reproduce except when
they are feeding on the cyclamen mites. These
features, which make Typhlodromus well-suited for
exploiting the seasonal rises and falls of its prey, are
common among predators that control prey
populations.
Greenhouse experiments have verified the
importance of Typhlodromus predation for keeping
cyclamen mites in check. One group of strawberry
plants was stocked with both predator and prey mites;
a second group was kept predator-free by regular
application of parathion, an insecticide that kills the
predatory species but does not affect the cyclamen
mite. Throughout the study, populations of cyclamen
mites remained low in plots shared with
Typhlodromus, but their infestation attained
significantly damaging proportions on predator-free
plants.
Applying parathion in this instance is a clear case
in which using a pesticide would do far more harm
than good to an agricultural enterprise. The results
were similar in field plantings of strawberries, where
cyclamen mites also reached damaging levels when
predators were eliminated by parathion, but they did
not attain such levels in untreated plots. When
cyclamen mite populations began to increase in an
untreated planting, the predator populations quickly
responded to reduce the outbreak. On average,
cyclamen mites were about 25 times more abundant
in the absence of predators than in their presence.

  1. The author mentions the egg-laying ability of each
    kind of mite (lines 20–23) primarily in order to
    support which one of the following claims?
    (A) Mites that reproduce by parthenogenesis do so
    at approximately equal rates.
    (B) Predatory mites typically have a longer
    reproductive life span than do cyclamen
    mites.
    (C) Typhlodromus can lay their eggs in synchrony
    with cyclamen mites.
    (D) Typhlodromus can reproduce at least as quickly
    as cyclamen mites.
    (E) The egg-laying rate of Typhlodromus is slower
    in the presence of cyclamen mites than it is in
    their absence.
A

Correct Answer: D

Passage Analysis

D Structure

The author mentions the egg-laying ability of each kind of mite in order to support the claim that Typhlodromus can reproduce at least as quickly as, if not more quickly than, cyclamen mites, which supports (D).

147
Q

Sometimes there is no more effective means of
controlling an agricultural pest than giving free rein
to its natural predators. A case in point is the
cyclamen mite, a pest whose population can be
effectively controlled by a predatory mite of the
genus Typhlodromus. Cyclamen mites infest
strawberry plants; they typically establish themselves
in a strawberry field shortly after planting, but their
populations do not reach significantly damaging
levels until the plants’ second year. Typhlodromus
mites usually invade the strawberry fields during the
second year, rapidly subdue the cyclamen mite
populations, and keep them from reaching
significantly damaging levels.
Typhlodromus owes its effectiveness as a predator
to several factors in addition to its voracious appetite.
Its population can increase as rapidly as that of its
prey. Both species reproduce by parthenogenesis—a
mode of reproduction in which unfertilized eggs
develop into fertile females. Cyclamen mites lay three
eggs per day over the four or five days of their
reproductive life span; Typhlodromus lay two or three
eggs per day for eight to ten days. Seasonal
synchrony of Typhlodromus reproduction with the
growth of prey populations and ability to survive at
low prey densities also contribute to the predatory
efficiency of Typhlodromus. During winter, when
cyclamen mite populations dwindle to a few
individuals hidden in the crevices and folds of leaves
in the crowns of the strawberry plants, the predatory
mites subsist on the honeydew produced by aphids
and white flies. They do not reproduce except when
they are feeding on the cyclamen mites. These
features, which make Typhlodromus well-suited for
exploiting the seasonal rises and falls of its prey, are
common among predators that control prey
populations.
Greenhouse experiments have verified the
importance of Typhlodromus predation for keeping
cyclamen mites in check. One group of strawberry
plants was stocked with both predator and prey mites;
a second group was kept predator-free by regular
application of parathion, an insecticide that kills the
predatory species but does not affect the cyclamen
mite. Throughout the study, populations of cyclamen
mites remained low in plots shared with
Typhlodromus, but their infestation attained
significantly damaging proportions on predator-free
plants.
Applying parathion in this instance is a clear case
in which using a pesticide would do far more harm
than good to an agricultural enterprise. The results
were similar in field plantings of strawberries, where
cyclamen mites also reached damaging levels when
predators were eliminated by parathion, but they did
not attain such levels in untreated plots. When
cyclamen mite populations began to increase in an
untreated planting, the predator populations quickly
responded to reduce the outbreak. On average,
cyclamen mites were about 25 times more abundant
in the absence of predators than in their presence.

  1. Which one of the following would, if true, most
    strengthen the author’s position regarding the
    practical applicability of the information about
    predatory mites presented in the passage?
    (A) The individual Typhlodromus mites that have
    the longest reproductive life spans typically
    also lay the greatest number of eggs per day.
    (B) The insecticides that are typically used for
    mite control on strawberry plants kill both
    predatory and nonpredatory species of mites.
    (C) In areas in which strawberry plants become
    infested by cyclamen mites, winters tend to be
    short and relatively mild.
    (D) Typhlodromus are sometimes preyed upon by
    another species of mites that is highly
    susceptible to parathion.
    (E) Typhlodromus easily tolerate the same range of
    climatic conditions that strawberry plants do.
A

Correct Answer: E

Passage Analysis

E RC Reasoning

The author discusses the practical applications in the third and fourth paragraphs.

A. No. This doesn’t mention the strawberry plants discussed in the third paragraph.

B. No. This doesn’t clearly strengthen his position; we don’t know what effect killing both predatory and non-predatory mites would have.

C. No. The passage doesn’t mention which climates are preferable to Typhlodromus.

D. No. The author is not advocating the use of parathion.

E. Yes. If Typhlodromus easily tolerates the same range of climatic conditions that strawberry plants do, then their use in controlling the cyclamen mite population would be as effective in all regions where strawberries are grown.

148
Q

Sometimes there is no more effective means of
controlling an agricultural pest than giving free rein
to its natural predators. A case in point is the
cyclamen mite, a pest whose population can be
effectively controlled by a predatory mite of the
genus Typhlodromus. Cyclamen mites infest
strawberry plants; they typically establish themselves
in a strawberry field shortly after planting, but their
populations do not reach significantly damaging
levels until the plants’ second year. Typhlodromus
mites usually invade the strawberry fields during the
second year, rapidly subdue the cyclamen mite
populations, and keep them from reaching
significantly damaging levels.
Typhlodromus owes its effectiveness as a predator
to several factors in addition to its voracious appetite.
Its population can increase as rapidly as that of its
prey. Both species reproduce by parthenogenesis—a
mode of reproduction in which unfertilized eggs
develop into fertile females. Cyclamen mites lay three
eggs per day over the four or five days of their
reproductive life span; Typhlodromus lay two or three
eggs per day for eight to ten days. Seasonal
synchrony of Typhlodromus reproduction with the
growth of prey populations and ability to survive at
low prey densities also contribute to the predatory
efficiency of Typhlodromus. During winter, when
cyclamen mite populations dwindle to a few
individuals hidden in the crevices and folds of leaves
in the crowns of the strawberry plants, the predatory
mites subsist on the honeydew produced by aphids
and white flies. They do not reproduce except when
they are feeding on the cyclamen mites. These
features, which make Typhlodromus well-suited for
exploiting the seasonal rises and falls of its prey, are
common among predators that control prey
populations.
Greenhouse experiments have verified the
importance of Typhlodromus predation for keeping
cyclamen mites in check. One group of strawberry
plants was stocked with both predator and prey mites;
a second group was kept predator-free by regular
application of parathion, an insecticide that kills the
predatory species but does not affect the cyclamen
mite. Throughout the study, populations of cyclamen
mites remained low in plots shared with
Typhlodromus, but their infestation attained
significantly damaging proportions on predator-free
plants.
Applying parathion in this instance is a clear case
in which using a pesticide would do far more harm
than good to an agricultural enterprise. The results
were similar in field plantings of strawberries, where
cyclamen mites also reached damaging levels when
predators were eliminated by parathion, but they did
not attain such levels in untreated plots. When
cyclamen mite populations began to increase in an
untreated planting, the predator populations quickly
responded to reduce the outbreak. On average,
cyclamen mites were about 25 times more abundant
in the absence of predators than in their presence.

  1. Information in the passage most strongly supports
    which one of the following statements?
    (A) Strawberry crops can support populations of
    both cyclamen mites and Typhlodromus mites
    without significant damage to those crops.
    (B) For control of cyclamen mites by another mite
    species to be effective, it is crucial that the two
    species have the same mode of reproduction.
    (C) Factors that make Typhlodromus effective
    against cyclamen mites also make it effective
    against certain other pests of strawberry
    plants.
    (D) When Typhlodromus is relied on to control
    cyclamen mites in strawberry crops,
    pesticides may be necessary to prevent
    significant damage during the first year.
    (E) Strawberry growers have unintentionally
    caused cyclamen mites to become a serious
    crop pest by the indiscriminate use of
    parathion.
A

Correct Answer: A

Passage Analysis

A Extract Infer

A. Yes. That’s exactly what the passage claims in the third paragraph when it states that, in the strawberry fields where there were both cyclamen mites and Typhlodromus, the cyclamen mites did not reach damaging levels.

B. No. The passage does not discuss the possibility of cyclamen mites being controlled by another mite species.

C. No. We know from the passage only about its effectiveness in controlling cyclamen mites.

D. No. The passage does not claim that pesticides would be necessary when Typhlodromus is relied on to control cyclamen mites in strawberry crops. In the first paragraph, we are told that cyclamen mites do not reach significantly damaging levels until the crops’ second year, which suggests that pesticide use before that time would be unnecessary.

E. No. We don’t know that strawberry growers have used parathion indiscriminately.

149
Q

The Internet is a system of computer networks
that allows individuals and organizations to
communicate freely with other Internet users
throughout the world. As a result, an astonishing
variety of information is able to flow unimpeded
across national and other political borders, presenting
serious difficulties for traditional approaches to
legislation and law enforcement, to which such
borders are crucial.
Control over physical space and the objects
located in it is a defining attribute of sovereignty.
Lawmaking presupposes some mechanism for
enforcement, i.e., the ability to control violations. But
jurisdictions cannot control the information and
transactions flowing across their borders via the
Internet. For example, a government might seek to
intercept transmissions that propagate the kinds of
consumer fraud that it regulates within its jurisdiction.
But the volume of electronic communications
crossing its territorial boundaries is too great to allow
for effective control over individual transmissions. In
order to deny its citizens access to specific materials,
a government would thus have to prevent them from
using the Internet altogether. Such a draconian
measure would almost certainly be extremely
unpopular, since most affected citizens would
probably feel that the benefits of using the Internet
decidedly outweigh the risks.
One legal domain that is especially sensitive to
geographical considerations is that governing
trademarks. There is no global registration of
trademarks; international protection requires
registration in each country. Moreover, within a
country, the same name can sometimes be used
proprietarily by businesses of different kinds in the
same locality, or by businesses of the same kind in
different localities, on the grounds that use of the
trademark by one such business does not affect the
others. But with the advent of the Internet, a business
name can be displayed in such a way as to be
accessible from any computer connected to the
Internet anywhere in the world. Should such a display
advertising a restaurant in Norway be deemed to
infringe a trademark in Brazil just because it can be
accessed freely from Brazil? It is not clear that any
particular country’s trademark authorities possess, or
should possess, jurisdiction over such displays.
Otherwise, any use of a trademark on the Internet
could be subject to the jurisdiction of every country
simultaneously.
The Internet also gives rise to situations in which
regulation is needed but cannot be provided within
the existing framework. For example, electronic
communications, which may pass through many
different territorial jurisdictions, pose perplexing new
questions about the nature and adequacy of privacy
protections. Should French officials have lawful
access to messages traveling via the Internet from
Canada to Japan? This is just one among many
questions that collectively challenge the notion that
the Internet can be effectively controlled by the
existing system of territorial jurisdictions.

  1. The author mentions French officials in connection
    with messages traveling between Canada and Japan
    (lines 57–59) primarily to
    (A) emphasize that the Internet allows data to be
    made available to users worldwide
    (B) illustrate the range of languages that might be
    used on the Internet
    (C) provide an example of a regulatory problem
    arising when an electronic communication
    intended for a particular destination passes
    through intermediate jurisdictions
    (D) show why any use of a trademark on the
    Internet could be subject to the jurisdiction of
    every country simultaneously
    (E) highlight the kind of international cooperation
    that made the Internet possible
A

C Structure

This paragraph is examining the need for regulation to determine what rights nations have concerning Internet transmissions passing through their borders.

A. No. This does not address the purpose of the paragraph.

B. No. This does not address the purpose of the paragraph.

C. Yes. This is a hypothetical example given so the reader may consider an unresolved regulatory question involving international Internet transmissions.

D. No. This paragraph has nothing to do with trademarks.

E. No. The origins of the Internet are never discussed.

149
Q

The Internet is a system of computer networks
that allows individuals and organizations to
communicate freely with other Internet users
throughout the world. As a result, an astonishing
variety of information is able to flow unimpeded
across national and other political borders, presenting
serious difficulties for traditional approaches to
legislation and law enforcement, to which such
borders are crucial.
Control over physical space and the objects
located in it is a defining attribute of sovereignty.
Lawmaking presupposes some mechanism for
enforcement, i.e., the ability to control violations. But
jurisdictions cannot control the information and
transactions flowing across their borders via the
Internet. For example, a government might seek to
intercept transmissions that propagate the kinds of
consumer fraud that it regulates within its jurisdiction.
But the volume of electronic communications
crossing its territorial boundaries is too great to allow
for effective control over individual transmissions. In
order to deny its citizens access to specific materials,
a government would thus have to prevent them from
using the Internet altogether. Such a draconian
measure would almost certainly be extremely
unpopular, since most affected citizens would
probably feel that the benefits of using the Internet
decidedly outweigh the risks.
One legal domain that is especially sensitive to
geographical considerations is that governing
trademarks. There is no global registration of
trademarks; international protection requires
registration in each country. Moreover, within a
country, the same name can sometimes be used
proprietarily by businesses of different kinds in the
same locality, or by businesses of the same kind in
different localities, on the grounds that use of the
trademark by one such business does not affect the
others. But with the advent of the Internet, a business
name can be displayed in such a way as to be
accessible from any computer connected to the
Internet anywhere in the world. Should such a display
advertising a restaurant in Norway be deemed to
infringe a trademark in Brazil just because it can be
accessed freely from Brazil? It is not clear that any
particular country’s trademark authorities possess, or
should possess, jurisdiction over such displays.
Otherwise, any use of a trademark on the Internet
could be subject to the jurisdiction of every country
simultaneously.
The Internet also gives rise to situations in which
regulation is needed but cannot be provided within
the existing framework. For example, electronic
communications, which may pass through many
different territorial jurisdictions, pose perplexing new
questions about the nature and adequacy of privacy
protections. Should French officials have lawful
access to messages traveling via the Internet from
Canada to Japan? This is just one among many
questions that collectively challenge the notion that
the Internet can be effectively controlled by the
existing system of territorial jurisdictions.

  1. Which one of the following most accurately expresses
    the main point of the passage?
    (A) The high-volume, global nature of activity on
    the Internet undermines the feasibility of
    controlling it through legal frameworks that
    presuppose geographic boundaries.
    (B) The system of Internet communications
    simultaneously promotes and weakens the
    power of national governments to control
    their citizens’ speech and financial
    transactions.
    (C) People value the benefits of their participation
    on the Internet so highly that they would
    strongly oppose any government efforts to
    regulate their Internet activity.
    (D) Internet communications are responsible for a
    substantial increase in the volume and
    severity of global crime.
    (E) Current Internet usage and its future
    expansion pose a clear threat to the internal
    political stability of many nations.
A

LSAT Preptest 54 Reading Comprehension Passage 1 Anylysis.

This passage addresses problems created for “traditional legislation and law enforcement” by the international flow of information on the Internet. The first paragraph defines the Internet and states the thesis. The second paragraph explains that a government can enforce control only over something contained within its national borders and it would be forced to enact a restrictive, unpopular policy of denying its citizens access to the Internet if it wanted to prevent or monitor all transmissions. The third paragraph examines the specific difficulties of trying to enforce trademark laws on the Internet. The fourth paragraph discusses the need for regulation regarding Internet transmissions that travel through several jurisdictions.

A Big Picture

A. Yes. It identifies that certain aspects of the Internet make it problematic for existing legal enforcement and regulation.

B. No. The passage does not make reference to weakening a government’s power over any sort of financial transactions.

C. No. This addresses only one sentence at the end of the second paragraph.

D. No. The passage never refers to global crime.

E. No. The passage never addresses the stability of many nations.

150
Q

The Internet is a system of computer networks
that allows individuals and organizations to
communicate freely with other Internet users
throughout the world. As a result, an astonishing
variety of information is able to flow unimpeded
across national and other political borders, presenting
serious difficulties for traditional approaches to
legislation and law enforcement, to which such
borders are crucial.
Control over physical space and the objects
located in it is a defining attribute of sovereignty.
Lawmaking presupposes some mechanism for
enforcement, i.e., the ability to control violations. But
jurisdictions cannot control the information and
transactions flowing across their borders via the
Internet. For example, a government might seek to
intercept transmissions that propagate the kinds of
consumer fraud that it regulates within its jurisdiction.
But the volume of electronic communications
crossing its territorial boundaries is too great to allow
for effective control over individual transmissions. In
order to deny its citizens access to specific materials,
a government would thus have to prevent them from
using the Internet altogether. Such a draconian
measure would almost certainly be extremely
unpopular, since most affected citizens would
probably feel that the benefits of using the Internet
decidedly outweigh the risks.
One legal domain that is especially sensitive to
geographical considerations is that governing
trademarks. There is no global registration of
trademarks; international protection requires
registration in each country. Moreover, within a
country, the same name can sometimes be used
proprietarily by businesses of different kinds in the
same locality, or by businesses of the same kind in
different localities, on the grounds that use of the
trademark by one such business does not affect the
others. But with the advent of the Internet, a business
name can be displayed in such a way as to be
accessible from any computer connected to the
Internet anywhere in the world. Should such a display
advertising a restaurant in Norway be deemed to
infringe a trademark in Brazil just because it can be
accessed freely from Brazil? It is not clear that any
particular country’s trademark authorities possess, or
should possess, jurisdiction over such displays.
Otherwise, any use of a trademark on the Internet
could be subject to the jurisdiction of every country
simultaneously.
The Internet also gives rise to situations in which
regulation is needed but cannot be provided within
the existing framework. For example, electronic
communications, which may pass through many
different territorial jurisdictions, pose perplexing new
questions about the nature and adequacy of privacy
protections. Should French officials have lawful
access to messages traveling via the Internet from
Canada to Japan? This is just one among many
questions that collectively challenge the notion that
the Internet can be effectively controlled by the
existing system of territorial jurisdictions.

  1. According to the passage, which one of the following
    is an essential property of political sovereignty?
    (A) control over business enterprises operating
    across territorial boundaries
    (B) authority over communicative exchanges
    occurring within a specified jurisdiction
    (C) power to regulate trademarks throughout a
    circumscribed geographic region
    (D) control over the entities included within a
    designated physical space
    (E) authority over all commercial transactions
    involving any of its citizens
A

D Extract Fact

The first sentence of the second paragraph states the defining attribute of sovereignty.

A. No. Control over business enterprises is not stated.

B. No. Authority over communicative exchanges is not stated.

C. No. Power to regulate trademarks is not stated.

D. Yes. This is almost the passage’s wording verbatim.

E. No. Authority over all commercial transactions is not stated.

151
Q

The Internet is a system of computer networks
that allows individuals and organizations to
communicate freely with other Internet users
throughout the world. As a result, an astonishing
variety of information is able to flow unimpeded
across national and other political borders, presenting
serious difficulties for traditional approaches to
legislation and law enforcement, to which such
borders are crucial.
Control over physical space and the objects
located in it is a defining attribute of sovereignty.
Lawmaking presupposes some mechanism for
enforcement, i.e., the ability to control violations. But
jurisdictions cannot control the information and
transactions flowing across their borders via the
Internet. For example, a government might seek to
intercept transmissions that propagate the kinds of
consumer fraud that it regulates within its jurisdiction.
But the volume of electronic communications
crossing its territorial boundaries is too great to allow
for effective control over individual transmissions. In
order to deny its citizens access to specific materials,
a government would thus have to prevent them from
using the Internet altogether. Such a draconian
measure would almost certainly be extremely
unpopular, since most affected citizens would
probably feel that the benefits of using the Internet
decidedly outweigh the risks.
One legal domain that is especially sensitive to
geographical considerations is that governing
trademarks. There is no global registration of
trademarks; international protection requires
registration in each country. Moreover, within a
country, the same name can sometimes be used
proprietarily by businesses of different kinds in the
same locality, or by businesses of the same kind in
different localities, on the grounds that use of the
trademark by one such business does not affect the
others. But with the advent of the Internet, a business
name can be displayed in such a way as to be
accessible from any computer connected to the
Internet anywhere in the world. Should such a display
advertising a restaurant in Norway be deemed to
infringe a trademark in Brazil just because it can be
accessed freely from Brazil? It is not clear that any
particular country’s trademark authorities possess, or
should possess, jurisdiction over such displays.
Otherwise, any use of a trademark on the Internet
could be subject to the jurisdiction of every country
simultaneously.
The Internet also gives rise to situations in which
regulation is needed but cannot be provided within
the existing framework. For example, electronic
communications, which may pass through many
different territorial jurisdictions, pose perplexing new
questions about the nature and adequacy of privacy
protections. Should French officials have lawful
access to messages traveling via the Internet from
Canada to Japan? This is just one among many
questions that collectively challenge the notion that
the Internet can be effectively controlled by the
existing system of territorial jurisdictions.

  1. Which one of the following words employed by the
    author in the second paragraph is most indicative of
    the author’s attitude toward any hypothetical
    measure a government might enact to deny its
    citizens access to the Internet?
    (A) benefits
    (B) decidedly
    (C) unpopular
    (D) draconian
    (E) risks
A

Correct Answer: D

Passage Analysis

D Extract Infer

A. No. The context in which this word is used suggests no attitude.

B. No. This relates to the attitude of the “affected citizens.”

C. No. This relates to the attitude of the “affected citizens.”

D. Yes. By using “draconian,” the author suggests such a measure would be unthinkably restrictive.

E. No. The context in which this word is used suggests no attitude.

151
Q

The Internet is a system of computer networks
that allows individuals and organizations to
communicate freely with other Internet users
throughout the world. As a result, an astonishing
variety of information is able to flow unimpeded
across national and other political borders, presenting
serious difficulties for traditional approaches to
legislation and law enforcement, to which such
borders are crucial.
Control over physical space and the objects
located in it is a defining attribute of sovereignty.
Lawmaking presupposes some mechanism for
enforcement, i.e., the ability to control violations. But
jurisdictions cannot control the information and
transactions flowing across their borders via the
Internet. For example, a government might seek to
intercept transmissions that propagate the kinds of
consumer fraud that it regulates within its jurisdiction.
But the volume of electronic communications
crossing its territorial boundaries is too great to allow
for effective control over individual transmissions. In
order to deny its citizens access to specific materials,
a government would thus have to prevent them from
using the Internet altogether. Such a draconian
measure would almost certainly be extremely
unpopular, since most affected citizens would
probably feel that the benefits of using the Internet
decidedly outweigh the risks.
One legal domain that is especially sensitive to
geographical considerations is that governing
trademarks. There is no global registration of
trademarks; international protection requires
registration in each country. Moreover, within a
country, the same name can sometimes be used
proprietarily by businesses of different kinds in the
same locality, or by businesses of the same kind in
different localities, on the grounds that use of the
trademark by one such business does not affect the
others. But with the advent of the Internet, a business
name can be displayed in such a way as to be
accessible from any computer connected to the
Internet anywhere in the world. Should such a display
advertising a restaurant in Norway be deemed to
infringe a trademark in Brazil just because it can be
accessed freely from Brazil? It is not clear that any
particular country’s trademark authorities possess, or
should possess, jurisdiction over such displays.
Otherwise, any use of a trademark on the Internet
could be subject to the jurisdiction of every country
simultaneously.
The Internet also gives rise to situations in which
regulation is needed but cannot be provided within
the existing framework. For example, electronic
communications, which may pass through many
different territorial jurisdictions, pose perplexing new
questions about the nature and adequacy of privacy
protections. Should French officials have lawful
access to messages traveling via the Internet from
Canada to Japan? This is just one among many
questions that collectively challenge the notion that
the Internet can be effectively controlled by the
existing system of territorial jurisdictions.

A
152
Q
A
152
Q
A
153
Q
A
154
Q
A
155
Q
A
156
Q
A
157
Q
A
158
Q
A
159
Q
A
160
Q
A
161
Q
A
162
Q
A
163
Q
A
164
Q
A
165
Q
A
166
Q
A
167
Q
A
168
Q
A
169
Q
A
170
Q
A
170
Q
A
171
Q
A
172
Q
A
173
Q
A
174
Q
A
175
Q
A
176
Q
A
176
Q
A
177
Q
A
178
Q
A
179
Q
A
180
Q
A
181
Q
A
181
Q
A
182
Q
A
183
Q
A
184
Q
A
185
Q
A
186
Q
A
187
Q
A
188
Q
A
189
Q
A
190
Q
A
190
Q
A
191
Q
A
192
Q
A
193
Q
A
194
Q
A
195
Q
A
196
Q
A
197
Q
A
198
Q
A
198
Q
A
199
Q
A
199
Q
A
199
Q
A
200
Q
A
200
Q
A
200
Q
A
201
Q
A
202
Q
A
203
Q
A
204
Q
A
205
Q
A
205
Q
A
205
Q
A
206
Q
A
207
Q
A
207
Q
A
208
Q
A
209
Q
A
210
Q
A
211
Q
A
212
Q
A
213
Q
A
214
Q
A
215
Q
A
216
Q
A
217
Q
A
218
Q
A
219
Q
A
220
Q
A
220
Q
A
221
Q
A